2 - Topnotch Anatomy SuperExam

December 19, 2017 | Author: ckmeesha | Category: Liver, Vein, Kidney, Aorta, Arm
Share Embed Donate


Short Description

anatomy...

Description

TOPNOTCH MEDICAL BOARD PREP ANATOMY SUPEREXAM For inquiries visit www.topnotchboardprep.com.ph or email us at [email protected] DEAR TOPNOTCH FRIENDS: PLEASE FOLLOW THESE INSTRUCTIONS: 1. These questions are previous diagnostic, midterm, and finals exams of Topnotch, almost all of them made by Topnotch Board Exam Topnotchers. 2. Answer this Topnotch Superexam seriously 100-items at a time. Cover the “Explanations” Column. Do not immediately look at the answers from the answer key. That’s not the correct way of answering sample exams. You need to treat these MCQs as exercises and not as handouts. 3. Time yourself. 1.5 hours per 100-item block. 4. After answering each 100-item block, refer to the Topnotch Answer Key for the correct answers. Please be careful of “frameshift mutations” when checking your answers – check every 10 items. (the format of the answer key was designed for you to practice against “frameshift mutations”) 5. The Topnotch Superexams are EXERCISES for the actual med boards. They will not appear verbatim in your future exams. More than knowing what’s the correct answer, it’s more important for you to: a. Know why the other choices are wrong b. Know why the other choices were included in the first place c. Know the explanation to the correct answer 6. Sharpen your mind by answering the Topnotch Superexams. Most of these questions based on past feedback are more difficult than the actual questions in the med boards. In these exams made by Board Exam Topnotchers, if you’re getting a score of 60/100 , that’s already a good score. More than 80/100 is outstanding. Item QUESTION EXPLANATION AUTHOR TOPNOTCH # EXAM 1 Patient had improperly fitted axillary crutch and suffered The affected nerve with improperly fitted crutches KRISTEL DIAGNOSTIC injury to a branch of the brachial plexus. Which is the most is the radial nerve. Yes, the first impulse for this TANHUI EXAM - likely finding? question is to think axillary nerve is the answer (TOP 3 - AUG MARCH A. Weakness of arm abduction because it’s an axillary crutch. BUT THAT’S 2015 MED 2016 B. Hand of benediction WRONG. The correct answer is radial nerve! BOARDS; C. Ape hand TOPNOTCH D. Cyclist palsy The axillary nerve, with the posterior humeral MD FROM LA E. Weakness in extending the forearm and wristdrop circumflex artery actually courses much higher SALLE) posteriorly via the quadrangular space. The radial nerve, along with the deep brachial artery courses posteriorly via the triangular interval and is more susceptible to injury from the axilla. Weakness of arm abduction - axillary nerve Hand of benediction – Median nerve Ape hand – median nerve Cyclist palsy – ulnar nerve Weakness in extending the forearm – radial nerve Source: Topnotch handout and pearls in anatomy

2

Which of the following is not involved in the transmission of weight while standing? A. Sacrum B. Pelvis C. Femur D. Tibia E. Fibula

The fibula is just a bone for anchoring muscles. In fact, it’s so insignificant, you can use it for bone grafting! The part used for grafting is the middle third because this contains the nutrient artery. Source: Moore Clinically oriented anatomy 4th ed Chapter 5 (Ebook)

KRISTEL TANHUI (TOP 3 - AUG 2015 MED BOARDS; TOPNOTCH MD FROM LA SALLE)

DIAGNOSTIC EXAM - MARCH 2016

3

Which is the equivalent layer of the Deep investing fascia of the abdomen in the perineum? A. Colles’ fascia B. Gallaudet’s fascia C. Scarpa’s fascia D. Dartos fascia E. Buck’s fascia

I can’t fit a table here but the ideal way to memorize the following is to put them in a table side by side: Abdomen (superficial to deep) Camper’s fascia (fatty superficial layer) Scarpa’s fascia (membranous superficial layer) Deep investing fascia Perineum (superficial to deep) Fatty superficial layer Colles’ fascia (membranous superficial layer) Gallaudet’s fascia/investing fascia Penis (superficial to deep) Dartos muscle Dartos fascia Buck’s fascia Source: Moore Clinically oriented anatomy 4th ed Chapter 2 and 3 (Ebook)

KRISTEL TANHUI (TOP 3 - AUG 2015 MED BOARDS; TOPNOTCH MD FROM LA SALLE)

DIAGNOSTIC EXAM - MARCH 2016

TOPNOTCH MEDICAL BOARD PREP ANATOMY SUPEREXAM Page 1 of 94 For inquiries visit www.topnotchboardprep.com.ph or email us at [email protected]

TOPNOTCH MEDICAL BOARD PREP ANATOMY SUPEREXAM For inquiries visit www.topnotchboardprep.com.ph or email us at [email protected] Item # 4

QUESTION

EXPLANATION

AUTHOR

TOPNOTCH EXAM DIAGNOSTIC EXAM - MARCH 2016

Which of the following is false of the Greater pelvis? A. Obstetric significance B. Lies above the linea terminales C. Pelvis major D. False pelvis E. None of the above

Lesser pelvis (true pelvis, pelvis minor) – below the arcuate line, obstetric significance Source: Moore Clinically oriented anatomy 4th ed Chapter 3 (Ebook)

KRISTEL TANHUI (TOP 3 - AUG 2015 MED BOARDS; TOPNOTCH MD FROM LA SALLE)

5

What is the shape of the left adrenal gland? A. Triangular B. Kidney C. Crescent D. Oval E. Round

Left crescent, Right triangle For those who are math geeks, my mnemonic is “right triangle” (a triangle with a right angle in it) Source: Moore Clinically oriented anatomy 4th ed Chapter 2 (Ebook)

KRISTEL TANHUI (TOP 3 - AUG 2015 MED BOARDS; TOPNOTCH MD FROM LA SALLE)

DIAGNOSTIC EXAM - MARCH 2016

6

Which of the following cartilages is pyramidal in shape? A. Epiglottis B. Arytenoid C. Corniculate D. Cuneiform E. Thyroid

Arytenoid: 3 sided pyramid Apex superiorly – bears corniculate cartilage and attaches to aryepiglottic fold Vocal process anteriorly – posterior attachement for vocal cord Muscular process laterally – attachment for posterior and lateral cricoarytenoid muscle Base: cricoarytenoid joint Source: Moore Clinically oriented anatomy 4th ed Chapter 8 (Ebook)

KRISTEL TANHUI (TOP 3 - AUG 2015 MED BOARDS; TOPNOTCH MD FROM LA SALLE)

DIAGNOSTIC EXAM - MARCH 2016

7

Which of the following is not true of the quadrangular membrane? A. Extends from the lateral aspects of the arytenoid and epiglottic cartilage B. The aryepiglottic ligament/fold forms the free superior margin C. The vestibular ligament/fold forms the free inferior margin D. The vestibular fold forms the false vocal cord E. None of the above

The vestibular ligament forms the inferior border of the quadrangular membrane. Source: Moore Clinically oriented anatomy 4th ed Chapter 8 (Ebook)

KRISTEL TANHUI (TOP 3 - AUG 2015 MED BOARDS; TOPNOTCH MD FROM LA SALLE)

DIAGNOSTIC EXAM - MARCH 2016

8

What do you call the junction where the nasal bones and frontal bones meet? A. Inion B. Rhinion C. Glabella D. Nasion E. Pterion

SIMILAR TO PREVIOUS BOARD EXAM CONCEPT/PRINCIPLE. Source: Moore Clinically oriented anatomy 4th ed Chapter 7 (Ebook)

KRISTEL TANHUI (TOP 3 - AUG 2015 MED BOARDS; TOPNOTCH MD FROM LA SALLE)

DIAGNOSTIC EXAM - MARCH 2016

9

FSH is secreted by which type of cell in the pituitary gland? A. Acidophils B. Basophils C. Chromophobes D. Supraoptic nuclei E. Paraventricular nuclei

Acidophils – Growth hormone, prolactin Basophils – FSH, LH, TSH, ACTH Supraoptic nuclei – Vasopressin mainly Paraventricular nuclei – Oxytocin mainly Source: Topnotch

KRISTEL TANHUI (TOP 3 - AUG 2015 MED BOARDS; TOPNOTCH MD FROM LA SALLE)

DIAGNOSTIC EXAM - MARCH 2016

10

A trauma patient suffers a tear at the junction of the SVC and the right atrium. This tear would likely damage the: A. SA node B. AV node C. AV bundle D. Right bundle branch E. Left bundle branch

The SA node is located in the right atrium at the junction where it is joined by the SVC. Source: Moore Clinically oriented anatomy 4th ed Chapter 1 (Ebook

KRISTEL TANHUI (TOP 3 - AUG 2015 MED BOARDS; TOPNOTCH MD FROM LA SALLE)

DIAGNOSTIC EXAM - MARCH 2016

11

The lowest level of the lung on end expiration at the midaxillary line is? A. 6th intercostal space B. 7th intercostal space C. 8th intercostal space D. 9th intercostal space E. 10th intercostal space

Lungs – 6-8-10 Pleura – 8-10-12 Source: Topnotch handout on anatomy

KRISTEL TANHUI (TOP 3 - AUG 2015 MED BOARDS; TOPNOTCH MD FROM LA SALLE)

DIAGNOSTIC EXAM - MARCH 2016

TOPNOTCH MEDICAL BOARD PREP ANATOMY SUPEREXAM Page 2 of 94 For inquiries visit www.topnotchboardprep.com.ph or email us at [email protected]

TOPNOTCH MEDICAL BOARD PREP ANATOMY SUPEREXAM For inquiries visit www.topnotchboardprep.com.ph or email us at [email protected] Item # 12

QUESTION

EXPLANATION

AUTHOR

TOPNOTCH EXAM DIAGNOSTIC EXAM - MARCH 2016

While doing a spinal tap, which describes the most accurate sequence of layers from the skin to the subarachnoid space? A. Skin, interspinous ligament, dura mater, subarachnoid space B. Skin, supraspinous ligament, interspinous ligament, posterior longitudinal ligament, dura mater, subarachnoid space C. Skin, supraspinous ligament, interspinous ligament, dura mater, subarachnoid space D. Skin supraspinous ligament, intertransverse ligament, arachnoid space, subarachnoid space E. Skin, interspinous ligament, anterior longitudinal ligament, dura mater, subarachnoid space

Source: Topnotch handout on anatomy

KRISTEL TANHUI (TOP 3 - AUG 2015 MED BOARDS; TOPNOTCH MD FROM LA SALLE)

13

A 5-year-old male presented with painless testicular mass. If the mass proved to be a malignancy, the earliest affected lymph nodes would be: A. Lumbar (aortic) lymph nodes B. Inferior mesenteric nodes C. Deep inguinal nodes D. Common iliac nodes E. Superficial inguinal nodes

Tumor cells from the gonads that metastasize through the lymphatics metastasize to the lumbar nodes initially. Remember that the origin of the gonadal arteries is the aorta, therefore the lymphatics will also course similarly. Source: Moore Clinically oriented anatomy 4th ed Chapter 2 (Ebook)

KRISTEL TANHUI (TOP 3 - AUG 2015 MED BOARDS; TOPNOTCH MD FROM LA SALLE)

DIAGNOSTIC EXAM - MARCH 2016

14

A surgeon is about to make an incision through the fundus of the stomach, which of the following requires clamping to prevent bleeding? A. Right gastric artery B. Left gastric artery C. Right gastroomental artery D. Left gastroomental artery E. Short gastric artery

Lesser curvature – right and left gastric artery Greater curvature – right and left gastroomental artery Fundus – short gastric artery Source: Moore Clinically oriented anatomy 4th ed Chapter 2 (Ebook)

KRISTEL TANHUI (TOP 3 - AUG 2015 MED BOARDS; TOPNOTCH MD FROM LA SALLE)

DIAGNOSTIC EXAM - MARCH 2016

15

A surgeon is about to perform cholecystectomy. The gallbladder can be localized in its fossa between which 2 anatomical lobes? A. Quadrate and left lobes B. Quadrate and caudate lobes C. Right and quadrate lobes D. Caudate and right lobes E. Caudate and left lobes

Source: Moore Clinically oriented anatomy 4th ed Chapter 2 (Ebook)

KRISTEL TANHUI (TOP 3 - AUG 2015 MED BOARDS; TOPNOTCH MD FROM LA SALLE)

DIAGNOSTIC EXAM - MARCH 2016

16

The right pulmonary artery: A. Are 2 in number B. Are 4 in number C. Courses posterior to the ascending aorta and the SVC D. B and C E. None of the above

There are 2 pulmonary arteries – 1 right and 1 left. There are 4 pulmonary veins. The right pulmonary artery courses posterior to the ascending aorta and SVC! Source: Moore Clinically oriented anatomy 4th ed Chapter 1 (Ebook)

KRISTEL TANHUI (TOP 3 - AUG 2015 MED BOARDS; TOPNOTCH MD FROM LA SALLE)

DIAGNOSTIC EXAM - MARCH 2016

17

What happens in May-Thurner syndrome? A. The aorta compresses the IVC B. The left common iliac vein is compressed by the left common iliac artery C. The right common iliac vein is compressed by the right common iliac artery D. The left common iliac vein is compressed by the right common iliac artery E. The right common iliac vein is compressed by the left common iliac artery

This condition predisposes to Deep vein thrombosis.

KRISTEL TANHUI (TOP 3 - AUG 2015 MED BOARDS; TOPNOTCH MD FROM LA SALLE)

DIAGNOSTIC EXAM - MARCH 2016

18

Which of the following is a syndesmosis? A. Interosseus membrane B. Gomphosis C. Distal tibiofibular joint D. A and B E. All of the above

Source: Topnotch handout on anatomy

KRISTEL TANHUI (TOP 3 - AUG 2015 MED BOARDS; TOPNOTCH MD FROM LA SALLE)

DIAGNOSTIC EXAM - MARCH 2016

TOPNOTCH MEDICAL BOARD PREP ANATOMY SUPEREXAM Page 3 of 94 For inquiries visit www.topnotchboardprep.com.ph or email us at [email protected]

TOPNOTCH MEDICAL BOARD PREP ANATOMY SUPEREXAM For inquiries visit www.topnotchboardprep.com.ph or email us at [email protected] Item # 19

QUESTION

EXPLANATION

AUTHOR

TOPNOTCH EXAM DIAGNOSTIC EXAM - MARCH 2016

Kiesselbach’s plexus is vascular anastomosis on the anterior part of the nose. Juan dela Cruz damaged this plexus while picking his nose. Which of the following is least likely to contribute to the epistaxis? A. Ophthalmic artery B. Maxillary artery C. Mandibular artery D. Facial artery E. Ethmoidal artery

Arterial supply Sphenopalatine artery (Maxillary artery) Anterior and posterior ethmoidal artery (ophthalmic artery branch) Greater palatine artery (Maxillary artery) Superior labial artery and lateral nasal branches of the facial artery Source: Moore Clinically oriented anatomy 4th ed Chapter 7 (Ebook)

KRISTEL TANHUI (TOP 3 - AUG 2015 MED BOARDS; TOPNOTCH MD FROM LA SALLE)

20

Which of the following is has a brush border lining its lumen? A. Small intestine B. Proximal convoluted tubule C. Distal convoluted tubule D. A and B E. All of the above

The brush border differentiates the proximal from the distal convoluted tubule. Source: Topnotch handout on anatomy

KRISTEL TANHUI (TOP 3 - AUG 2015 MED BOARDS; TOPNOTCH MD FROM LA SALLE)

DIAGNOSTIC EXAM - MARCH 2016

21

The inner ear is housed within which bone? A. Temporal B. Sphenoid C. Ethmoid D. Maxillary E. Zygomatic

SIMILAR TO PREVIOUS BOARD EXAM CONCEPT/PRINCIPLE. A few ENT questions were present in our Anatomy exam, and were generally tougher than the anatomy questions.

MIDTERM 1 EXAM - MARCH 2016

22

Which type of pelvis presents with a larger AP diameter compared to the transverse diameter? A. Gynecoid B. Android C. Anthropoid D. Platypelloid E. Arthropod

SIMILAR TO PREVIOUS BOARD EXAM CONCEPT/PRINCIPLE. Anthropoid=AP diameter>transverse; Platypelloid ("flatypelloid")=transverse diameter>AP diameter. Both came out in 2 seaparate questions during our Anatomy exam.

23

Correct boundaries of the greater sciatic foramen except: A. anterolaterally by the greater sciatic notch B. posteromedially by the sacrotuberous ligament C. inferiorly by the sacrospinous ligament and ischial spine D. inferolaterally by the piriformis muscle E. superiorly by the anterior sacroilliac ligament

SIMILAR TO PREVIOUS BOARD EXAM CONCEPT/PRINCIPLE. The piriformis muscle divides the greater sciatic foramen into a suprapiriform and infrapiriform foramen. There were about 3 questions about the greater sciatic notch in our Anatomy exam. Expect a few difficult questions about pelvic anatamy

24

Secretes surfactant in the lung: A. Type I alveolar B. Type II alveolar cells C. Kulchitsky cells D. Clara cells E. B, C, D are correct.

SIMILAR TO PREVIOUS BOARD EXAM CONCEPT/PRINCIPLE. The few histology questions asked during our anatomy exam were generally easy.

25

A 63 year-old male smoker develops an apical sulcus tumor that compresses the brachial plexus and cervical stellate ganglion. This may lead to: A. Pancoast syndrome B. Superior vena cava syndrome C. Horner's syndrome D. Thoracic outlet syndrome E. Thoracic inlet syndrome

SIMILAR TO PREVIOUS BOARD EXAM CONCEPT/PRINCIPLE. Thoracic outlet and inlet syndrome generally pertain to the same thing. Horner syndrome involves the cervical sympathetic trunk. 2 questions about this topic in our Anatomy exam.

LESTER BRYAN CO (TOP 10 - AUG 2015 MED BOARDS; TOPNOTCH MD FROM UST) LESTER BRYAN CO (TOP 10 - AUG 2015 MED BOARDS; TOPNOTCH MD FROM UST) LESTER BRYAN CO (TOP 10 - AUG 2015 MED BOARDS; TOPNOTCH MD FROM UST) LESTER BRYAN CO (TOP 10 - AUG 2015 MED BOARDS; TOPNOTCH MD FROM UST) LESTER BRYAN CO (TOP 10 - AUG 2015 MED BOARDS; TOPNOTCH MD FROM UST)

26

To pass a needle into the pleural space in the midaxillary line, the following structures will have to be pierced except? A. Internal intercostals B. Levatores costarum C. External intercostals D. Parietal pleura E. Innermost intercostals



MIDTERM 1 EXAM - MARCH 2016

27

On percussing the anterior chest of a patient, you find the right margin of the heart to lie 5 cm to right of the edge of the sternum. Which chamber of the heart is likely to be enlarged? A. left ventricle B. left atrium C. right ventricle D. right atrium E. both ventricles

The right border of the heart is formed by the right atrium. Inferior border is mostly the right ventricle with the apex of the left ventricle

LESTER BRYAN CO (TOP 10 - AUG 2015 MED BOARDS; TOPNOTCH MD FROM UST) LESTER BRYAN CO (TOP 10 - AUG 2015 MED BOARDS; TOPNOTCH MD FROM UST)

TOPNOTCH MEDICAL BOARD PREP ANATOMY SUPEREXAM Page 4 of 94 For inquiries visit www.topnotchboardprep.com.ph or email us at [email protected]

MIDTERM 1 EXAM - MARCH 2016

MIDTERM 1 EXAM - MARCH 2016

MIDTERM 1 EXAM - MARCH 2016

MIDTERM 1 EXAM - MARCH 2016

MIDTERM 1 EXAM - MARCH 2016

TOPNOTCH MEDICAL BOARD PREP ANATOMY SUPEREXAM For inquiries visit www.topnotchboardprep.com.ph or email us at [email protected] Item # 28

QUESTION

EXPLANATION

AUTHOR LESTER BRYAN CO (TOP 10 - AUG 2015 MED BOARDS; TOPNOTCH MD FROM UST) LESTER BRYAN CO (TOP 10 - AUG 2015 MED BOARDS; TOPNOTCH MD FROM UST) LESTER BRYAN CO (TOP 10 - AUG 2015 MED BOARDS; TOPNOTCH MD FROM UST) LESTER BRYAN CO (TOP 10 - AUG 2015 MED BOARDS; TOPNOTCH MD FROM UST)

TOPNOTCH EXAM MIDTERM 1 EXAM - MARCH 2016

The following structures form the walls of the inguinal canal except? A. Conjoint tendon B. Aponeurosis of external obliique muscle C. Internal oblique muscle D. Lacunar ligament E. Fascia transversalis



29

To pass a needle into the cavity of the tunica vaginalis in the scrotum, the following structures have to be pierced except? A. Dartos muscle B. Colles' fascia C. Tunica albuginea D. Internal spermatic fascia E. Cremasteric fascia

SIMILAR TO PREVIOUS BOARD EXAM CONCEPT/PRINCIPLE. The tunica albuginea is the fibrous covering of the testis. There were also other questions about Colles's and Scarpa's fascia.

30

The hilum of the right kidney contains the following important structures except? A. Renal pelvis B. Tributaries of right renal vein C. Sympathetic nerve fibers D. Part of the right suprarenal gland E. Branches of the right renal artery

The right suprarenal adrenal gland does not extend downward to the hilum of the right kidney.

31

A 53-year-old man has difficulty with breathing through his nose. On examination, his physician finds that he has swelling of the mucous membranes of the superior nasal meatus. Which opening of the paranasal sinuses is most likely plugged? A. Middle ethmoidal sinus B. Maxillary sinus C. Posterior ethmoidal sinus D. Anterior ethmoidal sinus E. Frontal sinus

The posterior ethmoidal sinus opens into the superior nasal meatus. The maxillary, frontal, and anterior and middle ethmoidal sinuses drain into the middle nasal meatus.

32

An oncologist is reviewing a CT scan of a 74-year-old man with newly diagnosed hepatocellular carcinoma. He locates the affected quadrate lobe of the liver that: A. Lies between the IVC and ligamentum venosum B. Receives blood from the right hepatic artery C. Drains bile into the left hepatic duct D. Is a medial superior segment E. Is functionally a part of the right lobe

The quadrate lobe of the liver drains bile into the left hepatic duct and receives blood from the left hepatic artery. It lies between the gallbladder fossa and the ligamentum teres hepatic, is a medial inferior segment, and is a part of the left lobe.

LESTER BRYAN CO (TOP 10 - AUG 2015 MED BOARDS; TOPNOTCH MD FROM UST)

MIDTERM 1 EXAM - MARCH 2016

33

In a patient with portal hypertension, which of the following veins is most likely to be dilated? A. Right colic vein B. Inferior epigastric vein C. Inferior phrenic vein D. Suprarenal vein E. Ovarian vein

The right colic vein belongs to the portal venous system and empties into the superior mesenteric vein, which joins the splenic vein to form the portal vein. The inferior epigastric, inferior phrenic, suprarenal, and ovarian veins belong to the systemic (or caval) venous system and drain directly or indirectly into the IVC.

MIDTERM 1 EXAM - MARCH 2016

34

A 2-year-old boy presents with pain in his groin that has been increasing over the past few weeks. He is found to have a degenerative malformation of the transversalis fascia during development. Which of the following structures on the anterior abdominal wall is likely defective? A. Superficial inguinal ring B. Deep inguinal ring C. Inguinal ligament D. Sac of a direct inguinal hernia E. Anterior wall of the inguinal canal

The deep inguinal ring lies in the transversalis fascia, just lateral to the inferior epigastric vessels. The superficial inguinal ring is in the aponeurosis of the external oblique muscle. The inguinal ligament and the anterior wall of the inguinal canal are formed by the aponeurosis of the external oblique muscle. The sac of a direct inguinal hernia is formed by the peritoneum.

LESTER BRYAN CO (TOP 10 - AUG 2015 MED BOARDS; TOPNOTCH MD FROM UST) LESTER BRYAN CO (TOP 10 - AUG 2015 MED BOARDS; TOPNOTCH MD FROM UST)

35

A pediatric surgeon is resecting a possible malignant mass from the liver of a neonate with cerebral palsy. The surgeon divides the round ligament of the liver during surgery. A fibrous remnant of which of the following fetal vessels is severed? A. Ductus venosus B. Ductus arteriosus C. Left umbilical vein D. Right umbilical vein E. Umbilical artery

The left umbilical vein becomes the round ligament of the liver after birth. The right umbilical vein does not leave a fibrous remnant because it degenerates during the early embryonic period. The ductus venosus forms the ligamentum venosum; the ductus arteriosus forms the ligamentum arteriosum; the umbilical artery forms the medial umbilical ligament.

LESTER BRYAN CO (TOP 10 - AUG 2015 MED BOARDS; TOPNOTCH MD FROM UST)

MIDTERM 1 EXAM - MARCH 2016

TOPNOTCH MEDICAL BOARD PREP ANATOMY SUPEREXAM Page 5 of 94 For inquiries visit www.topnotchboardprep.com.ph or email us at [email protected]

MIDTERM 1 EXAM - MARCH 2016

MIDTERM 1 EXAM - MARCH 2016

MIDTERM 1 EXAM - MARCH 2016

MIDTERM 1 EXAM - MARCH 2016

TOPNOTCH MEDICAL BOARD PREP ANATOMY SUPEREXAM For inquiries visit www.topnotchboardprep.com.ph or email us at [email protected] Item # 36

QUESTION

EXPLANATION

AUTHOR

TOPNOTCH EXAM MIDTERM 1 EXAM - MARCH 2016

A 33-year-old patient is suffering from a sudden occlusion at The first two posterior intercostal arteries are the origin of the descending (thoracic) aorta. This condition branches of the highest (superior) intercostal would most likely decrease blood flow in which of the artery of the costocervical trunk; the remaining following intercostal arteries? nine branches are from the thoracic aorta. The A. Upper six anterior internal thoracic artery gives off the upper six B. All of the posterior anterior intercostal arteries and is divided into the C. Upper two posterior superior epigastric and musculophrenic arteries, D. Lower anterior which gives off anterior intercostal arteries in the E. Lower six posterior 7th, 8th, and 9th intercostal spaces and ends in the 10th intercostal space where it anastomoses with the deep circumflex iliac artery.

LESTER BRYAN CO (TOP 10 - AUG 2015 MED BOARDS; TOPNOTCH MD FROM UST)

37

The following statements concerning the left suprarenal gland are correct except? A. The gland extends along the medial border of the left kidney from the upper pole to the hilus. B. The left suprarenal vein drains into the left renal vein. C. The gland is separated from the left kidney by perirenal fat. D. The gland lies behind the lesser sac. E. The medulla is innervated by post-ganglionic sympathetic nerve fibers

The medulla is innervated by preganglionic sympathetic fibers.

LESTER BRYAN CO (TOP 10 - AUG 2015 MED BOARDS; TOPNOTCH MD FROM UST)

MIDTERM 1 EXAM - MARCH 2016

38

A thoracic surgeon removed the right middle lobar bronchus along with lung tissue from a 57-year-old heavy smoker with lung cancer. Which of the following bronchopulmonary segments must contain cancerous tissues? A. Medial and lateral B. Anterior and posterior C. Anterior basal and medial basal D. Anterior basal and posterior basal E. Lateral basal and posterior basal

The right middle lobar (secondary) bronchus leads to the medial and lateral bronchopulmonary segments. The right superior lobar bronchus divides into the superior, posterior, and anterior segmental (tertiary) bronchi. The right inferior lobar bronchus has the anterior, lateral, posterior, and anterior segmental bronchi.

LESTER BRYAN CO (TOP 10 - AUG 2015 MED BOARDS; TOPNOTCH MD FROM UST)

MIDTERM 1 EXAM - MARCH 2016

39

A 19-year-old man came to the emergency department, and his angiogram exhibited that he was bleeding from the vein that is accompanied by the posterior interventricular artery. Which of the following veins is most likely to be ruptured? A. Great cardiac vein B. Middle cardiac vein C. Anterior cardiac vein D. Small cardiac vein E. Oblique veins of the left atrium

The middle cardiac vein ascends in the posterior interventricular groove, accompanied by the posterior interventricular branch of the right coronary artery. The great cardiac vein is accompanied by the anterior interventricular artery, the anterior cardiac vein drains directly into the right atrium, and the small cardiac vein is accompanied by the marginal artery.

LESTER BRYAN CO (TOP 10 - AUG 2015 MED BOARDS; TOPNOTCH MD FROM UST)

MIDTERM 1 EXAM - MARCH 2016

40

After an automobile accident, a back muscle that forms the boundaries of the triangle of auscultation and the lumbar triangle receives no blood. Which of the following muscles might be ischemic? A. Levator scapulae B. Rhomboid minor C. Latissimus dorsi D. Trapezius E. Splenius capitis

LESTER BRYAN CO (TOP 10 - AUG 2015 MED BOARDS; TOPNOTCH MD FROM UST)

MIDTERM 1 EXAM - MARCH 2016

41

The isthmus of the thyroid gland lies anterior to the A. Cricoid cartilage B. Thyroid cartilage C. 2-3 tracheal rings D. 4-5 tracheal rings E. NOTA

The latissimus dorsi forms boundaries of the auscultation and lumbar triangles and receives blood from the thoracodorsal artery. The levator scapulae, rhomboid minor, and splenius capitis muscles do not form boundaries of these two triangles. The trapezius muscle forms a boundary of the auscultation triangle but not the lumbar triangle. The levator scapulae, rhomboid minor, and trapezius muscles receive blood from the transverse cervical artery. The splenius capitis muscle receives blood from the occipital and transverse cervical arteries.

MIDTERM 2 EXAM - MARCH 2016

42

The nasal septum is comprised of the following, except: A. Septal cartilage B. Cribiform plate of ethmoid C. Vomer D. Perpendicular plate of ethmoid E. NOTA



43

The structure that drains into the inferior nasal meatus A. Nasolacrimal duct B. Anterior ethmoid sinus C. Frontal sinus D. Maxillary sinus E. NOTA

all other choices drain to the middle nasal meatus

GEORGE MICHAEL SOSUAN (TOP 5 - AUG 2015 MED BOARDS; TOPNOTCH MD FROM UST) GEORGE MICHAEL SOSUAN (TOP 5 - AUG 2015 MED BOARDS; TOPNOTCH MD FROM UST) GEORGE MICHAEL SOSUAN (TOP 5 - AUG 2015 MED BOARDS; TOPNOTCH MD FROM UST)

TOPNOTCH MEDICAL BOARD PREP ANATOMY SUPEREXAM Page 6 of 94 For inquiries visit www.topnotchboardprep.com.ph or email us at [email protected]

MIDTERM 2 EXAM - MARCH 2016

MIDTERM 2 EXAM - MARCH 2016

TOPNOTCH MEDICAL BOARD PREP ANATOMY SUPEREXAM For inquiries visit www.topnotchboardprep.com.ph or email us at [email protected] Item # 44

QUESTION

EXPLANATION

Aqueous humor is produced by the A. Choroid B. Ciliary process C. Cornea D. Retina E. Iris

The aqueous humor is produced by the ciliary process of the uveal tract and drained into the anterior chamber angle.

45

Origins of the arterial supply to the rectum, except A. Superior mesenteric artery B. Inferior mesenteric artery C. Internal iliac artery D. Internal pudendal artery E. NOTA

Superior rectal artery from the inferior mesenteric; Middle rectal artery from the internal iliac artery; inferior rectal artery from the internal pudendal artery from the internal iliac artery

46

Blood supply ot the liver includes A. 25% from portal vein B. 75% from hepatic artery C. 75% from portal vein D. 20% from the celiac artery E. Both A and B

75% from the portal vein and 25%from the hepatoc artery

47

Most common used vein in the antecubittal fossa for phlebotomy A. Median cubittal vein B. Brachial vein C. Basilic vein D. Cephalic vein E. AOTA



48

Divides the axillary artery into three parts A. Teres major B. Teres minor C. Scalene medius D. Pectoralis major E. Pectoralis minor



49

Support of the uterus is provided by the following, except: A. Uterosacral ligament B. Cardinal ligament C. Ileococcygeus D. Puborectalis E. Broad ligament

The other choices comprise the endopelvic fascia

50

The left primary bronchus A. Has one eparterial and one hyparterial bronchi B. Is narrower and shorter than the right C. Has the aorta arching over it D. Has a more vertical direction than the right E. NOTA



51

A fluid found in the membranous labyrinth A. Perilymph B. Corticolymph C. Blood D. Endolymph E. NOTA

Endolymph is the fluid contained in the membranous labyrinth of the inner ear and rich in potassium.

52

A pair of opening on the lateral wall of the oral cavity A. Ducts of Rivinius B. Wharton's duct C. Stensen's duct D. Boath A and B E. AOTA

The Stensen's duct or parotid duct opens opposite the 2nd upper molar.

AUTHOR GEORGE MICHAEL SOSUAN (TOP 5 - AUG 2015 MED BOARDS; TOPNOTCH MD FROM UST) GEORGE MICHAEL SOSUAN (TOP 5 - AUG 2015 MED BOARDS; TOPNOTCH MD FROM UST) GEORGE MICHAEL SOSUAN (TOP 5 - AUG 2015 MED BOARDS; TOPNOTCH MD FROM UST) GEORGE MICHAEL SOSUAN (TOP 5 - AUG 2015 MED BOARDS; TOPNOTCH MD FROM UST) GEORGE MICHAEL SOSUAN (TOP 5 - AUG 2015 MED BOARDS; TOPNOTCH MD FROM UST) GEORGE MICHAEL SOSUAN (TOP 5 - AUG 2015 MED BOARDS; TOPNOTCH MD FROM UST) GEORGE MICHAEL SOSUAN (TOP 5 - AUG 2015 MED BOARDS; TOPNOTCH MD FROM UST) GEORGE MICHAEL SOSUAN (TOP 5 - AUG 2015 MED BOARDS; TOPNOTCH MD FROM UST) GEORGE MICHAEL SOSUAN (TOP 5 - AUG 2015 MED BOARDS; TOPNOTCH MD FROM UST)

TOPNOTCH MEDICAL BOARD PREP ANATOMY SUPEREXAM Page 7 of 94 For inquiries visit www.topnotchboardprep.com.ph or email us at [email protected]

TOPNOTCH EXAM MIDTERM 2 EXAM - MARCH 2016

MIDTERM 2 EXAM - MARCH 2016

MIDTERM 2 EXAM - MARCH 2016

MIDTERM 2 EXAM - MARCH 2016

MIDTERM 2 EXAM - MARCH 2016

MIDTERM 2 EXAM - MARCH 2016

MIDTERM 2 EXAM - MARCH 2016

MIDTERM 2 EXAM - MARCH 2016

MIDTERM 2 EXAM - MARCH 2016

TOPNOTCH MEDICAL BOARD PREP ANATOMY SUPEREXAM For inquiries visit www.topnotchboardprep.com.ph or email us at [email protected] Item # 53

QUESTION

EXPLANATION

The bulk of muscles that make up the floor of the oral cavity A. Genioglossus B. Stylohyoid C. Mylohyoid D. Digastric E. NOTA

The mylohyoid forms the floor of the oral cavity and the roof of the submandibular triangle.

54

The Stensen's duct pierces this structure to open into the oral cavity A. Buccinator B. Masster C. Mentalis D. Zygoma E. NOTA

The Stensen's duct or parotid duct crosses the masseter to pierce the buccinator to open into the oral cavity.

55

Spongiocytes are located in this layer A. Zona glomerulosa B. Zona fasciculata C. Zona reticularis D. Adrenal medulla E. AOTA

A spongiocyte is a cell in the zona fasciculata of the adrenal cortex containing lipid droplets that show pronounced vacuolization.

56

Origin of blood supply of the suprarenal glands A. Inferior phrenic artery B. Renal artery C. Abdominal aorta D. Both B and C E. AOTA

The suprarenal glands has three origins of blood supply.

57

Most common direction of hip dislocation A. Anterior B. Posterior C. Superior D. Inferior E. Antero-inferior

90% of the hip dislocation occurs posteriorly.

58

Crutch paralysis injures this nerve A. Axillary nerve B. Radial nerve C. Median nerve D. Musculocutaneous nerve E. Ulnar nerve

Crutch paralysis is a form of paralysis which can occur when the radial nerve, is under constant pressure, such as by the use of a crutch.

59

Forms the hood of the clitoris A. Frenulum B. Labia majora C. Prepuce D. Labia minora E. NOTA



60

The ovarian vessels are enclosed by this ligament A. True ovarian ligament B. Suspensory ligament C. Round ligament D. Cardinal ligament E. NOTA

The suspensory ligament of the ovary, also infundibulopelvic ligament, is a fold of peritoneum that extends out from the ovary to the wall of the pelvis and encloses the ovarian vessels.

61

The lens of the eye is made up of what epithelial layer? A. Simple squamous B. Simple columnar C.Simple cuboidal D. Stratified squamous non keratinizing E. Pseudostratified columnar

Other parts of the eye: The corneal epithelium is made up of stratified squaomous. The corneal endothelium is made up of simple squamous.

AUTHOR GEORGE MICHAEL SOSUAN (TOP 5 - AUG 2015 MED BOARDS; TOPNOTCH MD FROM UST) GEORGE MICHAEL SOSUAN (TOP 5 - AUG 2015 MED BOARDS; TOPNOTCH MD FROM UST) GEORGE MICHAEL SOSUAN (TOP 5 - AUG 2015 MED BOARDS; TOPNOTCH MD FROM UST) GEORGE MICHAEL SOSUAN (TOP 5 - AUG 2015 MED BOARDS; TOPNOTCH MD FROM UST) GEORGE MICHAEL SOSUAN (TOP 5 - AUG 2015 MED BOARDS; TOPNOTCH MD FROM UST) GEORGE MICHAEL SOSUAN (TOP 5 - AUG 2015 MED BOARDS; TOPNOTCH MD FROM UST) GEORGE MICHAEL SOSUAN (TOP 5 - AUG 2015 MED BOARDS; TOPNOTCH MD FROM UST) GEORGE MICHAEL SOSUAN (TOP 5 - AUG 2015 MED BOARDS; TOPNOTCH MD FROM UST) JAN CHRISTIAN FELICIANO (TOP 2 - AUG 2015 MED BOARDS; TOPNOTCH MD FROM UST)

TOPNOTCH MEDICAL BOARD PREP ANATOMY SUPEREXAM Page 8 of 94 For inquiries visit www.topnotchboardprep.com.ph or email us at [email protected]

TOPNOTCH EXAM MIDTERM 2 EXAM - MARCH 2016

MIDTERM 2 EXAM - MARCH 2016

MIDTERM 2 EXAM - MARCH 2016

MIDTERM 2 EXAM - MARCH 2016

MIDTERM 2 EXAM - MARCH 2016

MIDTERM 2 EXAM - MARCH 2016

MIDTERM 2 EXAM - MARCH 2016

MIDTERM 2 EXAM - MARCH 2016

MIDTERM 3 EXAM - MARCH 2016

TOPNOTCH MEDICAL BOARD PREP ANATOMY SUPEREXAM For inquiries visit www.topnotchboardprep.com.ph or email us at [email protected] Item # 62

QUESTION

EXPLANATION

AUTHOR

TOPNOTCH EXAM MIDTERM 3 EXAM - MARCH 2016

Frey's syndrome commonly termed as crocodile tears is a condition wherein beads of perspiration appear on the skin when the patient eats due to penetrating injury to the parotid glands. This is due to faulty regeneration of the auriculotemporal nerve which is a branch of what nerve/ganglion? A. Trigeminal nerve B. Facial nerve C. Glossopharyngeal nerve D. Vagus nerve E. Sueprior cervical ganglion

The Auriculotemporal branch of the trigeminal nerve carries parasympathetic fibers to the sweat glands of the scalp and the parotid salivary gland. As a result of severance and inappropriate regeneration, the parasympathetic nerve fibers may switch course, resulting in "gustatory sweating" or sweating in the anticipation of eating, instead of the normal salivatory response.

JAN CHRISTIAN FELICIANO (TOP 2 - AUG 2015 MED BOARDS; TOPNOTCH MD FROM UST)

63

90% of epistaxis occurs in the Keiseelbach's plexus or Little's triangle. 4 arteries contribute to this plexus. Which artery does NOT contribute? A. Anterior ethmoidal artery of ophthalmic artery B. Sphenopalatine artery of maxillary artery C. Greater palatine artery of maxillary artery D. Dorsal nasal artery of ophthalmic artery E. Superior labial artery of facial artery

SIMILAR TO PREVIOUS BOARD EXAM CONCEPT/PRINCIPLE. The dorsal nasal artery is the termminal branch of the opthalmic artery and supplies the dorsum and root of the nose. All the other 4 arteries make up the Keisselbach's plexus

JAN CHRISTIAN FELICIANO (TOP 2 - AUG 2015 MED BOARDS; TOPNOTCH MD FROM UST)

MIDTERM 3 EXAM - MARCH 2016

64

When thyroid follicular cells are stimulated by TSH and fiiled with colloid they assume what configuration? A. Transitional B. Tall columnar C. Simple cuboidal D. Stratified squamous non keratinizing E. No specific configuration

SIMILAR TO PREVIOUS BOARD EXAM CONCEPT/PRINCIPLE. When inactive thyroid follicles are simple cuboidal but assume a tall columnar configuration when stimulated by TSH.

MIDTERM 3 EXAM - MARCH 2016

65

All of the ff nerves arises from the posterior cord of the brachial plexus EXCEPT? A. Subscapular B. Long thoracic C. Thoracodorsal D. Axillary E. Radial

For branches of the posterior cord. Remember the mnemonic STAR. Subscapular nerve, Thoracodorsal, Axillary and Radial. The long throacic nerve arises from the C5-C7 roots of the plexus not the cord.

66

The dorsalis pedis pulse can be located on what landmark? A. Lateral to the flexor hallucis longus and medial to flexor hallucis longus B .Lateral to the extensor hallucis brevis and medial to extnesor hallucis brevis C. Lateral to the extensor hallucis longus and medial to extnesor hallucis longus D. Lateral to the flexor hallucis brevis and medial to flexor hallucis brevis E. Behind the inguinal ligament between the ASIS and symphysis pubis

Choice C should have been: Lateral to the extensor hallucis longus and medial to extensor digitorum longus . SIMILAR TO PREVIOUS BOARD EXAM CONCEPT/PRINCIPLE. The dorsalis pedis artery pulse can be palpated readily lateral to the extensor hallucis longus tendon and medially to the extensor digitorum longus tendon on the dorsal surface of the foot. Choice E refers to the femoral artery

JAN CHRISTIAN FELICIANO (TOP 2 - AUG 2015 MED BOARDS; TOPNOTCH MD FROM UST) JAN CHRISTIAN FELICIANO (TOP 2 - AUG 2015 MED BOARDS; TOPNOTCH MD FROM UST) JAN CHRISTIAN FELICIANO (TOP 2 - AUG 2015 MED BOARDS; TOPNOTCH MD FROM UST)

67

What cell prodcues the hormone that activates bone resorption and increases blood calcium level? A. Follicular cells B. Parafolllicular cells C. Oxyntic cells D. Oxyphil cells E. Chief cells

SIMILAR TO PREVIOUS BOARD EXAM CONCEPT/PRINCIPLE. The questions refes to Parathormone or PTH secreted by the chief cells of the parathyroid gland. Parafollicular cellsof the thyroid gland secrete calcitonin. Oxyphil cells although part of the parathyroids have unknown functions. Oxyntic cells is in the stomach.

MIDTERM 3 EXAM - MARCH 2016

68

All of the ff are contents of the spermatic cord EXCEPT? A. Testicular artery B. Processus vaginalis C. Deferential artery D. Ilioinguinal nerve E. Pampiniform plexus

SIMILAR TO PREVIOUS BOARD EXAM CONCEPT/PRINCIPLE The ilioinguinal nerve psses through the inguinal canal but lies outside the spermatic cord. All the other structures plus the vas deferens, cremasteric artery and genital branch of the genitofemoral nerve are contents.

69

What structure is termed as the false vocal cords? A. Quadrangular membrane B. Vestibular folds C. Aryepiglottic folds D. Vocal folds E. Rima glottidis

SIMILAR TO PREVIOUS BOARD EXAM CONCEPT/PRINCIPLE. The vestibular fold is a fixed fold on each side of the larynx and is termed as the false vocal cords. While the mobile vocal folds immediately below it are the true vocal cords. The gap b/w the true vocal cord is termed as the rima glottidis or glottis. The quadrangular membrane extends between the epiglottis and arytenoid cartilage and its inferior margin forms the vestibular folds.

JAN CHRISTIAN FELICIANO (TOP 2 - AUG 2015 MED BOARDS; TOPNOTCH MD FROM UST) JAN CHRISTIAN FELICIANO (TOP 2 - AUG 2015 MED BOARDS; TOPNOTCH MD FROM UST) JAN CHRISTIAN FELICIANO (TOP 2 - AUG 2015 MED BOARDS; TOPNOTCH MD FROM UST)

TOPNOTCH MEDICAL BOARD PREP ANATOMY SUPEREXAM Page 9 of 94 For inquiries visit www.topnotchboardprep.com.ph or email us at [email protected]

MIDTERM 3 EXAM - MARCH 2016

MIDTERM 3 EXAM - MARCH 2016

MIDTERM 3 EXAM - MARCH 2016

MIDTERM 3 EXAM - MARCH 2016

TOPNOTCH MEDICAL BOARD PREP ANATOMY SUPEREXAM For inquiries visit www.topnotchboardprep.com.ph or email us at [email protected] Item # 70

QUESTION

EXPLANATION

AUTHOR

The medial wall of the bony orbit is formed by the ff bones EXCEPT? A. Frontal process of the maxilla B. Lacrimal bone C. Orbital plate of the maxilla D. Body of sphenoid E. Orbital plate of the ethmoid

SIMILAR TO PREVIOUS BOARD EXAM CONCEPT/PRINCIPLE. The orbital plate of the maxilla forms the floor of the orbit. The 4 other choices forms the medial wall of the orbit. The orbital plate of the frontal bone forms the roof of the orbit while the zygomatic and the greater wing of the sphenoid forms the lateral wall.

71

What rotator cuff muscle is a lateral rotator of the shoulder and supplied by a trunk of the brachial plexus? A. Supraspinatus B. Infraspinatus C. Teres Minor D.Teres Major E. Subscapularis

There are 2 lateral rotators of the shoulder: infraspinatus supplied by suprascapular nerve from the trunk of the brachial plexus and the teres minor supplied by the axillary nerve from the posteriro cord.

72

Which spinal nerves innervates the main muscle of respiration? A. C2 C3 C4 B. C3 C4 C5 C. C4 C5 C6 D. C5 C6 C7 E. C6 C7 T1

The question is refers to the phrenic nerve. Remember: C3 4 5 keeps the diaphragm alive.

73

What is the correct sequence of the CSF pathway? A .Lateral ventricle- Foramen of Magendie- 3rd ventricle- Cerebral aqueduct- 4th ventricle- Foramen of Monroe Arachnoid villi B. 3rd ventricle- Foramen of Magendie- Latreral ventricle- Cerebral aqueduct- 4th ventricle- Foramen of Monroe Arachnoid villi C. 3rd ventricle- Foramen of Monroe- Lateral ventricle- Cerebral aqueduct- 4th ventricle- Foramen of Magendie Arachnoid villi D. Lateral ventricle- Foramen of Monroe- 3rd ventricle- Cerebral aqueduct- 4th ventricle- Foramen of Magendie Arachnoid villi E. Lateral ventricle- Cerebral aqueduct- 3rd ventricle- Foramen of Monroe- 4th ventricle- Foramen of Magendie Arachnoid villi The outer hair cells of the organ of Corti are attached inferiorly to basilar membrane and superiorly to what membrane? A. Reissner's membrane B. Shrapnel's membrane C. Vestibular membrane D. Tympanic membrane E. Tectorial membrane

Knowthe CSF pathway by heart.. Lateral ventricle- Foramen of Monroe- 3rd ventricle- Cerebral aqueduct- 4th ventricle- Foramen of Lushka and Magendie -Subarachnoid space-Arachnoid villi

SIMILAR TO PREVIOUS BOARD EXAM CONCEPT/PRINCIPLE. The Reissner's aka the Vestibualr membrane is the roof of the scala media while the floor is the basilar membrane. The tectorial membrane overlies the outer hair cells of the organ of Corti and during acoustic stimulation stimulates the inner hair cells through fluid coupling.

JAN CHRISTIAN FELICIANO (TOP 2 - AUG 2015 MED BOARDS; TOPNOTCH MD FROM UST)

MIDTERM 3 EXAM - MARCH 2016

75

What cranial foramina and structure passing through it is correctly paired? A. Superior orbital fissure- opthalmic artery B. Foramen spinosum- Middle meningeal artery C. Foramen rotundum- Mandibular nerve D. Internal accoustic meatus- Glossopharyngeal nerve E. Jugular foramen- Facial nerve

Ophthalmic artery passes through the optic canal. The maxillary nerve passes through the formaen rotundum while the Mandibular nerve passes through the foramen ovale. GP nerve passes through jugular foramen while the facial nerve passes through the internal accoustic meatus

MIDTERM 3 EXAM - MARCH 2016

76

Which statement regarding the lungs is correct? A.The left lung is divided by the oblique and horizontal fissure into three lobes B. Each lung has an apex which projects upward into the neck 2.5 inches above the clavicle C. The pulmonary lobe is the anatomical and functional unit of the lung D. All the lymph from all parts of the lungs leaves the hilum and drains into the tracheobronchial nodes and into the bronchomediastinal trunks E. None of the above

The right lung is divided by the oblique and horizontal fissure into three lobes. The apex is 1 inch or 2.5 cm above the clavicle. The bornchopulmonary segment and not the lobe is the anatomical and functional unit of the lung.

JAN CHRISTIAN FELICIANO (TOP 2 - AUG 2015 MED BOARDS; TOPNOTCH MD FROM UST) JAN CHRISTIAN FELICIANO (TOP 2 - AUG 2015 MED BOARDS; TOPNOTCH MD FROM UST)

77

What lobe of the prostate is primarily affected in prostate adenocarcinoma? A. Anterior lobe B. Lateral lobe C. Posterior lobe D. Middle lobe E. Inferior lobe

Lateral and middle lobe (periurethral zone) is affected in BPH while posterior lobe (peripheral zone) is affected in prostate adenoCA

JAN CHRISTIAN FELICIANO (TOP 2 - AUG 2015 MED BOARDS; TOPNOTCH MD FROM UST)

MIDTERM 3 EXAM - MARCH 2016

74

JAN CHRISTIAN FELICIANO (TOP 2 - AUG 2015 MED BOARDS; TOPNOTCH MD FROM UST) JAN CHRISTIAN FELICIANO (TOP 2 - AUG 2015 MED BOARDS; TOPNOTCH MD FROM UST) JAN CHRISTIAN FELICIANO (TOP 2 - AUG 2015 MED BOARDS; TOPNOTCH MD FROM UST) JAN CHRISTIAN FELICIANO (TOP 2 - AUG 2015 MED BOARDS; TOPNOTCH MD FROM UST)

TOPNOTCH EXAM MIDTERM 3 EXAM - MARCH 2016

TOPNOTCH MEDICAL BOARD PREP ANATOMY SUPEREXAM Page 10 of 94 For inquiries visit www.topnotchboardprep.com.ph or email us at [email protected]

MIDTERM 3 EXAM - MARCH 2016

MIDTERM 3 EXAM - MARCH 2016

MIDTERM 3 EXAM - MARCH 2016

MIDTERM 3 EXAM - MARCH 2016

TOPNOTCH MEDICAL BOARD PREP ANATOMY SUPEREXAM For inquiries visit www.topnotchboardprep.com.ph or email us at [email protected] Item # 78

QUESTION

EXPLANATION

AUTHOR

TOPNOTCH EXAM MIDTERM 3 EXAM - MARCH 2016

What statement regarding the liver is true? A. The quadrate and caudate lobes are a functional part of the left lobe of the liver B. Divided into a large right lobe and small left lobe by the coronary ligament C. The first liver segment is regarded as the quadrate lobe and 4th segment is the caudate lobe D. Half of blood supply is from the portal vein and the other half is from the hepatic artery E. None of the above

Experiments have shown that the quadrate and caudate lobes are a functional part of the left lobe of the liver. It is divided into a large right lobe and small left lobe by the falciform ligament. The first liver segment is regarded as the caudate lobe and 4th segment is the quadrate lobe. Blood supply is 70% portal vein and 30% hepatic vein.

JAN CHRISTIAN FELICIANO (TOP 2 - AUG 2015 MED BOARDS; TOPNOTCH MD FROM UST)

79

Which of the ff nerve does not pass through the greater sciatic foramen? A. Superior gluteal nerve B. Inferior gluteal nerve C. Sciatic nerve D. Obturator nerve E. Pudendal nerve

JAN CHRISTIAN FELICIANO (TOP 2 - AUG 2015 MED BOARDS; TOPNOTCH MD FROM UST)

MIDTERM 3 EXAM - MARCH 2016

80

Which among the ff parts of the ear is part of the bony labyrynth? A. Utricle B. Saccule C. Semicircular canal D. Cohclear duct E. Auditory ossicles

SIMILAR TO PREVIOUS BOARD EXAM CONCEPT/PRINCIPLE. The ff structures passes thru the greater sciatic foramen: Sciatic Nerve, Superior Gluteal Nerve, Inferior Gluteal Nerve, Pudendal Nerve, Posterior, Femoral Cutaneous Nerve, Nerve to Quadratus Femoris, Nerve to Obturator Internus, Superior Gluteal Artery & Vein, Inferior Gluteal Artery & vein, Internal Pudendal Artery & vein and the Piriformis muscle. Please review the contents of the lesser sciatic foramen as well. SIMILAR TO PREVIOUS BOARD EXAM CONCEPT/PRINCIPLE. The inner ear is divided into the bony labyrynth (external) and membranous labyrnth (internal). Bony: Vestibule, Semicircular canal, cochlea. Membranous: Utricle, saccule, cochlear duct, semicircular duct

MIDTERM 3 EXAM - MARCH 2016

81

1. Which of the following does NOT play a role in parasitic infections? a. IL - 6 b. IL - 4 c. IgE d. IL - 5 e. none of the above



82

2. Which of the following structures does not pass through both greater and lesser sciatic foramen? a. Internal pudendal artery b. pudendal nerve c. obturator internus nerve d. obturator nerve e. none of the above

83

3. If the aryepiglottic fold forms the superior border, which of the following is the inferior border of the quadrangular membrane? a. Vocal ligament b. Vestibular ligament c. Median thyrohyoid ligament d. Cricotracheal ligament e. Cricoid cartilage 4. Which of the following cartilages is found in the quadrangular membrane? a. Corniculate cartilage b. Cuneiform cartilage c. Epiglottis d. Thyroid cartilage e. Arytenoid cartilage

obturator nerve arises from the lumbar plexus (L2 – L4) and emerges on the medial border of the psoas muscle within the abdomen. It runs forward on the lateral wall of the pelvis to reach the upper part of the obturator foramen where it divides into anterior and posterior divisions. Snells’ Clinical anatomy by regions 9th edition p. 465 The thickened inferior margin forms the vestibular ligament and the vestibular ligaments form the interior of the vestibular folds. Snells’ Clinical anatomy by regions 9th edition p. 647

JAN CHRISTIAN FELICIANO (TOP 2 - AUG 2015 MED BOARDS; TOPNOTCH MD FROM UST) ANDREW TIU (TOP 1 - AUG 2015 MED BOARDS; TOPNOTCH MD FROM CIM) ANDREW TIU (TOP 1 - AUG 2015 MED BOARDS; TOPNOTCH MD FROM CIM) ANDREW TIU (TOP 1 - AUG 2015 MED BOARDS; TOPNOTCH MD FROM CIM) ANDREW TIU (TOP 1 - AUG 2015 MED BOARDS; TOPNOTCH MD FROM CIM) ANDREW TIU (TOP 1 - AUG 2015 MED BOARDS; TOPNOTCH MD FROM CIM) ANDREW TIU (TOP 1 - AUG 2015 MED BOARDS; TOPNOTCH MD FROM CIM)

84

These two small rod shaped cartilages are found in the aryepiglottic folds and serve to strengthen them. Snells’ Clinical anatomy by regions 9th edition p. 646

85

5. Which of the following is found superior to the arytenoid cartilages? a. Corniculate cartilage b. Cuneiform cartilage c. Epiglottis d. Thyroid cartilage e. Arytenoid cartilage

Snells’ Clinical anatomy by regions 9th edition p. 646

86

6. Which of the following is a membranous labyrinth? a. Vestibule b. Semicircular canals c. Cochlea d. Utricle e. None of the above

The membranous labyrinth is lodged within the bony labyrinth. It is filed with endolymph and surrounded by perilymph. It consists of utricle and saccule which are lodged in the bony vestibule; three semicircular ducts, which lie within the bony semicircular canals; and the duct of the cochlea, which lies within the bony cochlea. All these structures freely communicate with each other. Snells’ Clinical anatomy by regions 9th edition p. 569

TOPNOTCH MEDICAL BOARD PREP ANATOMY SUPEREXAM Page 11 of 94 For inquiries visit www.topnotchboardprep.com.ph or email us at [email protected]

FINAL EXAM - MARCH 2016

FINAL EXAM - MARCH 2016

FINAL EXAM - MARCH 2016

FINAL EXAM - MARCH 2016

FINAL EXAM - MARCH 2016

FINAL EXAM - MARCH 2016

TOPNOTCH MEDICAL BOARD PREP ANATOMY SUPEREXAM For inquiries visit www.topnotchboardprep.com.ph or email us at [email protected] Item # 87

QUESTION 7. Which of the following is responsible for depression of the mandible? a. Temporalis b. Lateral pterygoid c. Medial pterygoid d. Masseter e. None of the above

EXPLANATION Contraction of the lateral pterygoids pulls forward the neck of the mandible and articular disc so that the latter moves onto the articular tubercle. The forward movement of the disc is limited by the tension of the fibroelastic tissue, which tethers the disc to the temporal bone posteriorly. Snells’ Clinical anatomy by regions 9th edition p. 572 Snells’ Clinical anatomy by regions 9th edition p. 569

88

8. Which of the following divides the cochlear canal into scala vestibule and scala tympani? a. Basilar membrane b. Reissner’s membrane c. Tectorial membrane d. Vestibular membrane e. Spiral ligament

89

9. Which of the following membranes vibrates to code acoustic information into nerve impulses and is responsible for amplitude discrimination? a. Basilar membrane b. Reissner’s membrane c. Tectorial membrane d. Vestibular membrane e. Spiral ligament

90

10. Which of the following landmarks signify the intersection between the frontal bone and 2 nasal bones? a. Nasion b. Rhinion c. Pterion d. Glabella e. Asterion

91

11. Which of the following lies above the pelvic inlet? a. true pelvis b. false pelvis c. pelvic cavity d. lesser pelvis e. none of the above

SNell's CLinical anatomy for Medical students 5th edition p. 277

92

12. Which of the following types of pelvis is described with a greater anteroposterior diameter, straight walls, small subpubic arch, and large sacrosciatic notches? a. android b. anthropoid c. gynecoid d. platypelloid e. none of the above 13. Which of the following is a branch from the aorta? a. inferior rectal artery b. superior vesical artery c. superior rectal artery d. vaginal artery e. all of the above

A - heart shaped with triangular anterior segment C - normal female pelvis D - transversely wide with flattened anteroposterior diameter SNell's CLinical anatomy for Medical students 5th edition p. 299

14. A 5 year old male comes to you for epistaxis. Which of the following does NOT contribute to the blood supply in the Kiesselbach area? a. ophthalmic artery b. sphenopalatine artery c. greater palatine artery d. superior nasal artery e. superior labial artery 15. Which of the following is NOT a globulin? a. transferrin b. fibrinogen c. Beta2 - macroglobulin d. fibronectin e. immunoglobulin

The following contributes: anterior ethmoidal artery from ophthalmic artery sphenopalatine artery from terminal branch of maxillary artery greater palatine artery from maxillary artery septal branch of superior labial artery from facial artery

16. Which of the following develops from a reticular cell? a. eosinophil b. platelet c. RBC d. neutrophil e. basophil

Junqueira's Basic Histology 12th edition p. 209

93

94

95

96

Hair cells are attached through the phalangeal cells to the basilar membrane, which vibrates during sound reception. The stereocilia of these hair cells are in turn attached to the tectorial membrane which also vibrates. However, the tectorial membrane and the basilar membrane are hinged at different points. Thus, a shearing effect occurs between the basilar membrane ( and the cells attached to it) and the tectorial membrane when sound vibrations impinge on the ear. Histology 7th edition. Pawlina. P. 952 Rhinion refers to the lower end of the suture between the nasal bones. Wikipedia (cant find it in snells)

this confusing question came out. Best answer is still superior rectal artery, a branch of the inferior mesenteric artery. SNell's CLinical anatomy for Medical students 5th edition p. 291

Junqueira's Basic Histology 12th edition p. 203

AUTHOR ANDREW TIU (TOP 1 - AUG 2015 MED BOARDS; TOPNOTCH MD FROM CIM) ANDREW TIU (TOP 1 - AUG 2015 MED BOARDS; TOPNOTCH MD FROM CIM) ANDREW TIU (TOP 1 - AUG 2015 MED BOARDS; TOPNOTCH MD FROM CIM)

ANDREW TIU (TOP 1 - AUG 2015 MED BOARDS; TOPNOTCH MD FROM CIM) ANDREW TIU (TOP 1 - AUG 2015 MED BOARDS; TOPNOTCH MD FROM CIM) ANDREW TIU (TOP 1 - AUG 2015 MED BOARDS; TOPNOTCH MD FROM CIM) ANDREW TIU (TOP 1 - AUG 2015 MED BOARDS; TOPNOTCH MD FROM CIM) ANDREW TIU (TOP 1 - AUG 2015 MED BOARDS; TOPNOTCH MD FROM CIM) ANDREW TIU (TOP 1 - AUG 2015 MED BOARDS; TOPNOTCH MD FROM CIM) ANDREW TIU (TOP 1 - AUG 2015 MED BOARDS; TOPNOTCH MD FROM CIM)

TOPNOTCH MEDICAL BOARD PREP ANATOMY SUPEREXAM Page 12 of 94 For inquiries visit www.topnotchboardprep.com.ph or email us at [email protected]

TOPNOTCH EXAM FINAL EXAM - MARCH 2016

FINAL EXAM - MARCH 2016

FINAL EXAM - MARCH 2016

FINAL EXAM - MARCH 2016

FINAL EXAM - MARCH 2016

FINAL EXAM - MARCH 2016

FINAL EXAM - MARCH 2016

FINAL EXAM - MARCH 2016

FINAL EXAM - MARCH 2016

FINAL EXAM - MARCH 2016

TOPNOTCH MEDICAL BOARD PREP ANATOMY SUPEREXAM For inquiries visit www.topnotchboardprep.com.ph or email us at [email protected] Item # 97

98

99

QUESTION 17. Which of the following zones of the adrenal cortex has long cords of large polyhedral cells separated by fenestrated sinusoidal capillaries? a. zona glomerulosa b. zona fasciculata c. zona reticularis d. zona pellucida e. zona medulla 18. The skin is not the largest organ of the body as it is considered an organ system. Part of its functions is protection of the body by triggering an immune response. One of the cells that play a role in alerting the immune system is bone marrow derived and capable of binding, processing, and presenting antigens to T lymphocytes. Which of the following layers of the skin is this cell predominantly located? a. stratum corneum b. stratum lucidum c. stratum germinativum d. stratum granulosum e. stratum spinosum 19. Which of following supporting cells of the CNS is derived from neural tube and functions in the repair processes? a. oligodendrocyte b. neurolemmocyte c. astrocyte d. ependymal cell e. microglia

EXPLANATION

AUTHOR

The cells are most densely filled with cytoplasmic lipid droplets and as a result of lipid dissolution during tissue preparation, often appear vacuolated or spongy. Junqueira's Basic Histology 12th edition p. 356

Junqueira's Basic Histology 12th edition p. 320

oligodendrocyte - neural tube - CNS - myelin production neurolemmocyte - neural crest - PNS - myelin production ependymal cell - neural tube - CNS - lines the cavities microglia - bone marrow - CNS - immune related activity Junqueira's Basic Histology 12th edition p. 151 http://radiopaedia.org/articles/ureter

ANDREW TIU (TOP 1 - AUG 2015 MED BOARDS; TOPNOTCH MD FROM CIM) ANDREW TIU (TOP 1 - AUG 2015 MED BOARDS; TOPNOTCH MD FROM CIM)

TOPNOTCH EXAM FINAL EXAM - MARCH 2016

FINAL EXAM - MARCH 2016

ANDREW TIU (TOP 1 - AUG 2015 MED BOARDS; TOPNOTCH MD FROM CIM)

FINAL EXAM - MARCH 2016

ANDREW TIU (TOP 1 - AUG 2015 MED BOARDS; TOPNOTCH MD FROM CIM) ANGELA PAULINE P. CALIMAGLOYOLA (TOP 8 - FEB 2015 MED BOARDS; TOPNOTCH MD FROM UST)

FINAL EXAM - MARCH 2016

100

20. At what level does the ureter turns forward and medially to insert into the posterolateral wall of the urinary bladder? a. pelvic brim b. coccyx c. ischial spine d. bifurcation of common iliac vessels e. sacroiliac joint

101

A 30 y/o female patient is brought to the emergency room in Page 11 of Topnotch Handout. To establish an respiratory distress. On PE she has an anterior neck mass, emergency airway you do a cricothyroidotomy. which relatives recounted that it recently suddenly increased Through the cricothyroid membrane located in size. You decided to create an emergency airway to retore inferior to the thyroid cartilage and superior to the respiration. At what level could you rapidly create an airway cricoid cartilage, the true vocal cords lie superior with minimum danger of hemorrhage and damage to the to it. vocal cords? A. Through the third tracheal ring B. Just above the jugular notch C. Just above the thyroid cartilage D. Just below the thyroid cartilage E. Between the 3rd and 4th tracheal ring

102

During thyroidectomy of the patient in the above scenario, Page 11 of Topnotch Handout. The External the surgeon blindly clamped the superior thyroid artery and laryngeal nerve may be injured during ligation of ligates it. After 24 hours post-op the patient was noted to the superior thyroid artery. The cricothyroid is have a change in vocal quality. Which of the following supplied by the External laryngeal nerve. The muscles may be affected due to an injury to its innervation other muscles of phonation are supplied by the during the ligation of the superior thyroid artery? Recurrent laryngeal nerve. A. Thyroarytenoid B. Lateral cricoarytenoid C. Posterior cricoarytenoid D. Cricothyroid E. All of the above

ANGELA PAULINE P. CALIMAGLOYOLA (TOP 8 - FEB 2015 MED BOARDS; TOPNOTCH MD FROM UST)

DIAGNOSTIC EXAM - AUG 2015

103

A term newborn was noted to have grunting and retractions. SIMILAR TO PREVIOUS BOARD EXAM His chest xray showed a markedly elevated right CONCEPT/PRINCIPLE. Phrenic nerve palsy will be hemidiaphragm with essentially clear lung fields. PE showed seen as elevation of diagphram on chest xray. a globular abdomen. The most likely diagnosis is: A. Phrenic nerve palsy B. Congenital diaphragmatic hernia C. Pulmonary hypoplasia D. TTN E. PPHN

ANGELA PAULINE P. CALIMAGLOYOLA (TOP 8 - FEB 2015 MED BOARDS; TOPNOTCH MD FROM UST)

DIAGNOSTIC EXAM - AUG 2015



TOPNOTCH MEDICAL BOARD PREP ANATOMY SUPEREXAM Page 13 of 94 For inquiries visit www.topnotchboardprep.com.ph or email us at [email protected]

DIAGNOSTIC EXAM - AUG 2015

TOPNOTCH MEDICAL BOARD PREP ANATOMY SUPEREXAM For inquiries visit www.topnotchboardprep.com.ph or email us at [email protected] Item # 104

105

106

107

108

109

110

QUESTION

EXPLANATION

An 80 y/o male went to the OPD with complaints of low Page 1 and 3 of supplement Topnotch Handout. L5 back pain. Further tests showed a herniated disc root compression will manifest at the lateral compressing the L5 root. Where will the patient have surface of the leg and dorsum of the foot. dermatomal pain? A. Groin B. Posterior thigh C. Lateral part of the lower leg D. Medial aspect of the calf E. Big toe

AUTHOR ANGELA PAULINE P. CALIMAGLOYOLA (TOP 8 - FEB 2015 MED BOARDS; TOPNOTCH MD FROM UST)

A 29 y/o male presented at the ER due to abdominal pain. Page 10 and 11 of Topnotch Handout. The rule is ANGELA History revealed that the pain started in the periumbilical that pain originating from the midgut is generally PAULINE P. area. This was accompanied by anorexia. After 6 hours, the perceived in the periumbilical region. The midgut: CALIMAGpatient started vomiting and the pain became localized in the in the 5 mm embryo, it is seen beginning just LOYOLA right lower quadrant The organ involved is most likely a caudal to the entrance of the bile duct into the (TOP 8 - FEB derivative of the: duodenum and terminating at the beginning of the 2015 MED A. Foregut last third of the transverse colon (from anterior to BOARDS; B. Midgut posterior intestinal portals). It is suspended from TOPNOTCH C. Hindgut the dorsal abdominal wall by a short mesentery MD FROM D. All of the above and communicates with the yolk sac via the UST) E. None of the above vitelline duct. Its derivatives consist of the small intestines (except the first part of the duodenum to the common bile duct entrance); the cecum, the appendix, the ascending colon; and the right onehalf to two-thirds or proximal part of the transverse colon. It is supplied by the superior mesenteric artery and vagus nerve. The patient was then brought to the OR for emergency Page 1-2 of Day 3 Topnotch Handout. McBurney's ANGELA appendectomy, a McBurney's incision was done. This incision, is oblique beginning laterally from above PAULINE P. incision will pass through the following except: and ending medially and located at about oneCALIMAGA. External oblique third the distance along a line from the ASIS to the LOYOLA B. Internal oblique umbilicus. The skin and subcutaneous tissue are (TOP 8 - FEB C. Transversus abdominis incised down to the external oblique muscle, the 2015 MED D. Rectus abdominis underlying internal oblique and transversus BOARDS; E. None of the above abdominis are split and separated. TOPNOTCH MD FROM UST) A 10 month old male was diagnosed with Congenital megacolon. On barium studies which segment is considered normal: A. the dilated proximal colon B. the contracted distal colon C. the whole colon D. Both A and B E. None of the above

TOPNOTCH EXAM DIAGNOSTIC EXAM - AUG 2015

DIAGNOSTIC EXAM - AUG 2015

DIAGNOSTIC EXAM - AUG 2015

SIMILAR TO PREVIOUS BOARD EXAM CONCEPT/PRINCIPLE. Page 11 of Day 3 Topnotch Handout. In Hirscprungs disease/Congenital megacolon there is absence of autonomic ganglion cells in the myenteric plexus distal to the dilated segment of colon.

ANGELA DIAGNOSTIC PAULINE P. EXAM - AUG CALIMAG2015 LOYOLA (TOP 8 - FEB 2015 MED BOARDS; TOPNOTCH MD FROM UST) In a bilateral recurrent layngeal nerve injury the vocal cords SIMILAR TO PREVIOUS BOARD EXAM ANGELA DIAGNOSTIC assume what position? CONCEPT/PRINCIPLE. Bilateral recurrent PAULINE P. EXAM - AUG A. Midway between full abduction and adduction laryngeal nerve palsy: In this condition both cords CALIMAG2015 B. Paramedian assume a paramedian position compromising the LOYOLA C. Full Abduction airway. This commonly occurs following total (TOP 8 - FEB D. Full Adduction thyroidectomy or in thyroid malignancies. The 2015 MED E. Either C or D patient will commonly manifest with stridor. The BOARDS; voice will be near normal. TOPNOTCH MD FROM UST) As a surgeon who will perform a laparoscopic Page 9 of Day 3 Topnotch Handout. Liver is ANGELA DIAGNOSTIC cholecystectomy, you must know the boundaries of the superior, cystic duct is inferior, common hepatic PAULINE P. EXAM - AUG triangle of Calot which consists of: duct is medial. CALIMAG2015 A. Cystic duct, common hepatic duct, inferior border of the LOYOLA liver (TOP 8 - FEB B. Cystic duct, common hepatic duct, common bile duct 2015 MED C. Common bile duct, common hepatic duct, inferior margin BOARDS; of the liver TOPNOTCH D. Cystic duct, common bile duct, inferior border of the MD FROM liver UST) E. Cystic duct, common hepatic artery, inferior border of the liver

A 9 month old male was noted to have an inguinal mass. A SIMILAR TO PREVIOUS BOARD EXAM ANGELA DIAGNOSTIC diagnosis of cryptorchid testes was made. Which of the CONCEPT/PRINCIPLE. Page 22 of day 3 Topnotch PAULINE P. EXAM - AUG following controls the descent of testes into the scrotum: Handout. 26-28 weeks-the testes descended CALIMAG2015 A. Withdrawal of maternal estrogen retroperitoneally from the posterior abdominal LOYOLA B. Enlargement of fetal pelvis wall to the deep inguinal rings; due to (TOP 8 - FEB C. Androgens enlargement of fetal pelvis. 2-3 days descent into 2015 MED D. All of the above the scrotum; controlled by androgens. BOARDS; E. Both A and B TOPNOTCH MD FROM UST)

TOPNOTCH MEDICAL BOARD PREP ANATOMY SUPEREXAM Page 14 of 94 For inquiries visit www.topnotchboardprep.com.ph or email us at [email protected]

TOPNOTCH MEDICAL BOARD PREP ANATOMY SUPEREXAM For inquiries visit www.topnotchboardprep.com.ph or email us at [email protected] Item # 111

112

QUESTION

EXPLANATION

AUTHOR

TOPNOTCH EXAM DIAGNOSTIC EXAM - AUG 2015

A 23 y/o male arrived at the ER. He a suffered severe head Page 33 of supplement Topnotch Handout. The ANGELA trauma in a motorcycle accident. Radiographic studies of the mandibular branch/V3 of the trigeminal exits thru PAULINE P. head revealed a basilar skull fracture in the region of the the foramen ovale. It will manifest as Loss of CALIMAGforamen ovale. Which of the following functional losses general sensation in skin over mandible, LOYOLA would most likely be related to this injury? : mandibular teeth, tongue, paralysis of muscles of (TOP 8 - FEB A. Inability to abduct eye mastication; jaw deviation to injured side, 2015 MED B. Loss of general sensation in skin over maxilla Hypoacusis due to paralysis of the tensor tympani BOARDS; C. Ageusia muscle. B is related to maxillary branch of TOPNOTCH D. Loss of afferent limb of corneal reflex trigeminal/V2 which exits from the foramen MD FROM E. Hypoacusis rotundum, C is alteration or loss of taste realted to UST) facial nerve injury which exits from the internal auditory meatus, D loss of afferent limb of corneal reflex is due to injury to the Ophthalmic branch of the trigeminal/V1 which exits from the superior orbital fissure, Inability to abduct the eye is due to injury of the abducens which also exits from the superior orbital fissure. The physician was listening for breath sounds over the right Page 20 of Day 3 Topnotch Handout. The parietal ANGELA DIAGNOSTIC hemithorax and was concerned when no sounds were heard pleura reflection passes obliquely across the 8th PAULINE P. EXAM - AUG on the front of the chest wall at the level of the tenth rib in rib in MCL not the 10th. CALIMAG2015 the MCL, the following statements are correct, except: LOYOLA A. In a healthy individual, the lower border of the right lung (TOP 8 - FEB in the MCL in the midrespiratory position is at the level of 2015 MED the 6th rib B. No breath sounds are heard because the BOARDS; stethoscope was located over the liver TOPNOTCH C. The costodiaphragmatic recess is situated between the MD FROM lower border of the lung and the parietal pleura UST) D. The parietal pleura in the MCL crosses the tenth rib E. The lower margin of the lung crosses the tenth rib on the sides of the vertebral column

113

A nexus is an area of low electrical resistance and is involved Page 5 of Histology Topnotch Handout. Gap ANGELA in metabolic and electrical coupling of cells. These are found junctions or Nexus are found in nearly all tissues PAULINE P. in nearly all tissues except: except skeletal muscle. CALIMAGA. Osteocytes LOYOLA B. Skeletal muscle (TOP 8 - FEB C. Neurons 2015 MED D. Smooth muscle BOARDS; E. Cardiac muscle TOPNOTCH MD FROM UST)

DIAGNOSTIC EXAM - AUG 2015

114

Type IIa muscle fibers are also known as: Page 13 of Histology Topnotch Handout. Type IIa A. Red, slow, oxidative fibers are Red, fast, oxidative-glycolytic fibers. Type I are B. White, fast, glycolytic fibers Red, slow, oxidative fibers, while Type IIb are fast, C. Red, fast, oxidative-glycolytic fibers glycolytic fibers. D. White, slow, oxidative-glycolytic fibers E. Red, slow, oxidative-glycolytic fibers

DIAGNOSTIC EXAM - AUG 2015

115

ANGELA PAULINE P. CALIMAGLOYOLA (TOP 8 - FEB 2015 MED BOARDS; TOPNOTCH MD FROM UST) The liver has several functions including the storage of Page 24 of Histology Topnotch Handout. Ito ANGELA vitamin A and other fat soluble vitamins. This is specifically a cells/Stellate Cells/Lipocytes stores vitamin A and PAULINE P. function of the Ito Cells which are found in the: other fat soluble vitamins and are found in the CALIMAGA. Sinusoids space of disse or perisinusoidal space. LOYOLA B. Bile canaliculi (TOP 8 - FEB C. Principal parenchyma cell 2015 MED D. Space of disse BOARDS; E. Rokitansky aschoff sinus TOPNOTCH MD FROM UST)

TOPNOTCH MEDICAL BOARD PREP ANATOMY SUPEREXAM Page 15 of 94 For inquiries visit www.topnotchboardprep.com.ph or email us at [email protected]

DIAGNOSTIC EXAM - AUG 2015

TOPNOTCH MEDICAL BOARD PREP ANATOMY SUPEREXAM For inquiries visit www.topnotchboardprep.com.ph or email us at [email protected] Item # 116

117

118

119

120

121

QUESTION

EXPLANATION

AUTHOR

TOPNOTCH EXAM DIAGNOSTIC EXAM - AUG 2015

The normal anatomical position of the epididymis is SIMILAR TO PREVIOUS BOARD EXAM ANGELA ________________ in relation to the testes: CONCEPT/PRINCIPLE. The testes are located PAULINE P. A. Postero-lateral within the scrotum, with the epididymis situated CALIMAGB. Inferior on the posterolateral aspect of each testicle. LOYOLA C. Anterior Commonly, the left testicle lies lower than the (TOP 8 - FEB D. Superior right. They are suspended from the abdomen by 2015 MED E. Lateral the spermatic cord – collection of vessels, nerves BOARDS; and ducts that supply the testes. The epididymis is TOPNOTCH 5–10 mm thick and extends from the upper to the MD FROM caudal pole of the testis. The epididymis can be UST) divided in caput, corpus and cauda. A thin capsule and the serosa of the tunica vaginalis cover the epididymis. 8–10 efferent ducts transport the sperm from the rete testis into the ductus epididymidis. The epididymis consists mostly of the coiled epididymal duct, at the cauda of the epididymis the ductus deferens starts. Reference: http://www.ncbi.nlm.nih.gov/pubmed/16944482 J Clin Ultrasound. 2006 Oct;34(8):385-92. Normal and variant appearances of the adult epididymis and vas deferens on high-resolution sonography. Puttemans T1, Delvigne A, Murillo D. The prostatic segment of the male urethra is lined with this Page 27 of Histology Topnotch Handout. ANGELA DIAGNOSTIC type of epithelium: PAULINE P. EXAM - AUG A. Stratified columnar CALIMAG2015 B. Urothelium LOYOLA C. Pseudostratified columnar epithelium (TOP 8 - FEB D. Stratified squamous epithelium 2015 MED E. Pseudostratified squamous epithelium BOARDS; TOPNOTCH MD FROM UST) The anal canal above the dentate line is sensitive to: SIMILAR TO PREVIOUS BOARD EXAM ANGELA DIAGNOSTIC A. Pain CONCEPT/PRINCIPLE. Page 11 of Day 3 Topnotch PAULINE P. EXAM - AUG B. Temperature Handout. The anal canal segment above the CALIMAG2015 C. Touch dentate line is derived from the hindgut, supplied LOYOLA D. Pressure and drained by the superior rectal, sensitive to (TOP 8 - FEB E. Stretch stretch. 2015 MED BOARDS; TOPNOTCH MD FROM UST) Which of the following blood cells has the longest life span: SIMILAR TO PREVIOUS BOARD EXAM ANGELA DIAGNOSTIC A. RBC CONCEPT/PRINCIPLE. Page 10 of Histology PAULINE P. EXAM - AUG B. Neutrophils Topnotch Handout. ADULT RBC- 120 days, Fetal CALIMAG2015 C. Platelets RBC-90 days, Platelets-10 days, Reticulocytes- 1-2 LOYOLA D. Eosinophils days, Neutrophils- 1-4 days, Eosinophils-1-2 (TOP 8 - FEB E. Reticulocytes weeks, Basophils-Several months, Lymphocytes- 2015 MED hours to years, Monocytes- hours to years. BOARDS; TOPNOTCH MD FROM UST) A 56 y/o male came to the cardiac catheterization SIMILAR TO PREVIOUS BOARD EXAM ANGELA DIAGNOSTIC laboratory for suspected acute coronary syndrome. Coronary CONCEPT/PRINCIPLE. Page 17 of Day 2 Topnotch PAULINE P. EXAM - AUG angiography showed a 90% occlusion in the distal left Handout. In approximately 70% of the population, CALIMAG2015 circumflex. The patient had arrived at the hospital 24 hours the PDA originates from the right coronary artery; LOYOLA earlier with crescendo angina. He had no prior history of it is codominant in 20%, meaning both the right (TOP 8 - FEB coronary artery disease. His ECG showed Sinus rhythm at coronary artery and LCx feed the PDA; and 10% 2015 MED 75/min with acute myocardial injury of the posterior are left dominant, meaning the LCx alone supplies BOARDS; myocardium manifested as down-sloping ST-segment the PDA. TOPNOTCH depression in leads V1 to V5. The coronary artery that MD FROM supplies the PDA determines coronary “dominance.” In UST) approximately 70% of the population, the PDA originates from: A. Left circumflex artery B. Left Coronary artery C. Right Coronary artery D. Marginal artery E. Both A and C

The following layers of abdominal wall have scrotal derivatives except: A. External oblique B. Peritoneum C. Transversus abdominis D. Transversalis fascia E. Superficial fascia

Superficial fascia - Dartos muscle; External oblique - External spermatic fascia; Internal Oblique - Cremaster muscle; Transversalis fascia - Internal spermatic fascia; Peritoneum- Tunica vaginalis

LYNN DARYL FELICIANO VILLAMATE R, MD (TOP 5 - FEB 2015 MED BOARDS; TOPNOTCH MD FROM EAC)

TOPNOTCH MEDICAL BOARD PREP ANATOMY SUPEREXAM Page 16 of 94 For inquiries visit www.topnotchboardprep.com.ph or email us at [email protected]

MIDTERM 1 EXAM - AUG 2015

TOPNOTCH MEDICAL BOARD PREP ANATOMY SUPEREXAM For inquiries visit www.topnotchboardprep.com.ph or email us at [email protected] Item # 122

QUESTION

EXPLANATION

The following are parts of the greater omentum EXCEPT? A. Gastrohepatic B. Gastrocolic C. Gastrosplenic D. Gastrophrenic E. No exception

Gastrohepatic/hepatogastric (and hepatoduodenal) are parts of the lesser omentum.

123

The most common site of diverticulitis is the: A. Cecum B. Ascending colon C. Transverse colon D. Descending colon E. Sigmoid



124

A 28-year old male tennis player comes to the clinic and complained of pain when he opens the door or lift a glass. Which of the following muscles is least likely involved in this condition? A. Extensor carpi radialis brevis B. Extensor digitorum C. Extensor carpi ulnaris D. Supinator E. Extensor carpi radialis longus

This is a case of tennis elbow or lateral epicondylitis. Muscle attached to lateral epicondyle include the following: Anconeus muscle, the supinator, extensor carpi radialis brevis, extensor digitorum, extensor digiti, and extensor carpi ulnaris. ECRL is the exception. Its origin is the lateral supracondylar ridge of the humerus. This is a similar question asked in the boards.

125

The radial artery is the smaller of the terminal branches of the brachial artery. It begins at the level of: A. Lateral epicondyle of the humerus B. Head of the radius C. Radial neck D. Ulnar head E. Styloid process of the radius

The radial artery is the smaller of the terminal branches of brachial artery. It begins in the cubital fossa at the level of the neck of the radius and passes downward and laterally, beneath the bracioradialis muscle. In the middle third of its course, the superficial branch of the radial nerve lies on its lateral side. SIMILAR TO PREVIOUS BOARD EXAM CONCEPT

126

A 50 year old male patient was noted to have cyanosis and decreased sensation of left foot. You decided to assess the dorsalis pedis pulse. It is palpated: A. Medially to the extensor hallucis longus tendon B. Medially to the extensor digitorum longus tendon C. In front of the medial malleolus D. Behind the medial malleolus E. Behind the lateral malleolus

Dorsalis pedis artery can be palpated medially to the extensor digitorum longus tendon or laterally to the EHL tendon. Structures that pass in front of medial malleolus: Great saphenous vein and saphenous nerve. Behind the medial malleolus: Posterior tibial artery, tibial nerve, tibialis posterior tendon, FDL and FHL. Behind the lateral malleolus: Sural nerve and small saphenous vein. SIMILAR TO PREVIOUS BOARD EXAM CONCEPT

127

A 35 y/o patient suffered from a vehicular crash affecting his right kidney. On exploration, the first layer of the kidney to be encountered during surgery is the: A. Pararenal fat B. Perirenal fat C. Renal fascia D. Outer corte E. Fibrous capsule

128

129

AUTHOR LYNN DARYL FELICIANO VILLAMATE R, MD (TOP 5 - FEB 2015 MED BOARDS; TOPNOTCH MD FROM EAC) LYNN DARYL FELICIANO VILLAMATE R, MD (TOP 5 - FEB 2015 MED BOARDS; TOPNOTCH MD FROM EAC) LYNN DARYL FELICIANO VILLAMATE R, MD (TOP 5 - FEB 2015 MED BOARDS; TOPNOTCH MD FROM EAC)

TOPNOTCH EXAM MIDTERM 1 EXAM - AUG 2015

MIDTERM 1 EXAM - AUG 2015

MIDTERM 1 EXAM - AUG 2015

LYNN DARYL FELICIANO VILLAMATE R, MD (TOP 5 - FEB 2015 MED BOARDS; TOPNOTCH MD FROM EAC) LYNN DARYL FELICIANO VILLAMATE R, MD (TOP 5 - FEB 2015 MED BOARDS; TOPNOTCH MD FROM EAC)

MIDTERM 1 EXAM - AUG 2015

Layers of the kidney from outer to inner: Pararenal fat, renal fascia/Gerota's fascia, perirenal fat, fibrous/true capsule. Renal fascia encloses both the kidney and suprarenal glands. SIMILAR TO PREVIOUS BOARD EXAM CONCEPT

LYNN DARYL FELICIANO VILLAMATE R, MD (TOP 5 - FEB 2015 MED BOARDS; TOPNOTCH MD FROM EAC)

MIDTERM 1 EXAM - AUG 2015

The least dilatable portion of the male urethra: A. Prostatic urethra B. Membranous urethra C. Penile urethra D. Sphincter urethra E. None of the above

PROSTATIC URETHRA - widest and most dilatable portion; MEMBRANOUS - shortest and least dilatable; PENILE URETHRA - longest, EXTERNAL MEATUS - narrowest part; FOSSA TERMINALIS/NAVICULAR FOSSA - lies within the glans penis. SIMILAR TO PREVIOUS BOARD EXAM CONCEPT

MIDTERM 1 EXAM - AUG 2015

The most commonly injured portion of the ureter during gynecologic surgery? A. Abdominal portion B. Middle portion C. Pelvic portion D. Near the bifurcation of aorta E. As it crosses the pelvic brim

The ureter runs vertically downward behind and enters the pelvis by crossing the bifurcation of the common iliac artery in front of the sacroiliac joint. In vaginal hysterectomy, the ureter courses lateral to the uterine cervix. In oophorectomy, it lies medial to ovarian vessels. It also passes inferior to the uterine vessels. SIMILAR TO PREVIOUS BOARD EXAM CONCEPT

LYNN DARYL FELICIANO VILLAMATE R, MD (TOP 5 - FEB 2015 MED BOARDS; TOPNOTCH MD FROM EAC) LYNN DARYL FELICIANO VILLAMATE R, MD (TOP 5 - FEB 2015 MED BOARDS; TOPNOTCH MD FROM EAC)

TOPNOTCH MEDICAL BOARD PREP ANATOMY SUPEREXAM Page 17 of 94 For inquiries visit www.topnotchboardprep.com.ph or email us at [email protected]

MIDTERM 1 EXAM - AUG 2015

MIDTERM 1 EXAM - AUG 2015

TOPNOTCH MEDICAL BOARD PREP ANATOMY SUPEREXAM For inquiries visit www.topnotchboardprep.com.ph or email us at [email protected] Item # 130

QUESTION

EXPLANATION

AUTHOR

TOPNOTCH EXAM MIDTERM 1 EXAM - AUG 2015

Which of the following is true regarding safe subclavian vein catherization using the infraclavicular approach? A. Needle is inserted below the lower border of the clavicle at the junction of medial third and outer 2/3 of the clavicle and is pointed downward and posteriorly toward the middle suprasternal notch B. The needle is directed upward in the direction of the opposite nipple. C. The needle is directed downward in the direction of the opposite nipple. D. Needle is inserted below the lower border of the clavicle at the junction of medial third and outer 2/3 of the clavicle and is pointed upward and posteriorly toward the middle suprasternal notch E. None of the above.

Supraclavicular approach - Patient in Trendelenburg position with head turned to opposite side, the posterior border of clavicular origin of SCM is palpated. Needle is inserted and is directed downward in the direction of opposite nipple and enters the junction of IJV and subclavian vein. Infraclavicular approach - needle is inserted through the skin just below the lower border of th clavicle at the junction of the medial third and outer 2/3. Needle is pointed and upward and posteriorly toward the middle of the suprasternal notch. SIMILAR TO PREVIOUS BOARD EXAM CONCEPT.

LYNN DARYL FELICIANO VILLAMATE R, MD (TOP 5 - FEB 2015 MED BOARDS; TOPNOTCH MD FROM EAC)

131

Hyaline cartilage is composed of what type of collagen? A. Type I B. Type II C. Type III D. Type IV E. Type V

Type I - Fibrocartilage and elastic cartilage; Type II - Hyaline cartilage. SIMILAR TO PREVIOUS BOARD EXAM CONCEPT.

MIDTERM 1 EXAM - AUG 2015

132

The main glycosaminoglycan content of the bone is A. Chondroitin sulfate B. Hyaluronic acid C. Dermatan sulfate D. Heparan sulfate E. Keratan sulfate

Hyaluronic acid - synovial fluid, vitreous humor; Chrondoitin sulfate - none, cornea, skin; Dermatan sulftage - skin, tendon; Heparan sulfate - basal laminae, Keratan sulfate - cornea, nucleus pulposus, annulus fibosus

133

Patient presented with chronic cough and weight loss and was diagnosed with lung mass at the hilar area. Lymph metastases will first affect nodes at the: ? A. Bronchomediastinal lymph trunks B. Bronchopulmonary nodes C. Paratracheal D. Tracheobronchial nodes E. Parasternal

The superficial (subpleural) plexus lies beneath the visceral pleura and drains over the surface of the lungs toward the hilum, where the lymph vessels enter the bronchopulmonary nodes. All the lymph from the lung then leaves the hilum and drains into the tracheobronchial nodes and then into the bronchomediastinal lymph trunks. SIMILAR TO PREVIOUS BOARD EXAM CONCEPT.

LYNN DARYL FELICIANO VILLAMATE R, MD (TOP 5 - FEB 2015 MED BOARDS; TOPNOTCH MD FROM EAC) LYNN DARYL FELICIANO VILLAMATE R, MD (TOP 5 - FEB 2015 MED BOARDS; TOPNOTCH MD FROM EAC) LYNN DARYL FELICIANO VILLAMATE R, MD (TOP 5 - FEB 2015 MED BOARDS; TOPNOTCH MD FROM EAC)

134

A 50 year old woman was noted to have 2 cm breast cancer located in the nipple areolar complex of the left breast. Which of the following lymph nodes is most likely to be affected? A. Parasternal lymph nodes B. Inferior phrenic lymph nodes C. Axillary lymph nodes D. Subscapular lymph nodes E. Supraclavicular lymph nodes

Lymph from medial breast quadrants, drains to the parasternal lymph nodes or to the opposite breast while lymph from the inferior quadrants may pass deeply to the abdominal lymph nodes (subdiaphragmatic inferior phrenic lymph nodes). More than 75% of the lymph, especially from the lateral breast quadrants drains to the axillary lymph nodes, iniitially to the anterior or pectoral nodes for the most part.

LYNN DARYL FELICIANO VILLAMATE R, MD (TOP 5 - FEB 2015 MED BOARDS; TOPNOTCH MD FROM EAC)

MIDTERM 1 EXAM - AUG 2015

135

A 15-year-old male suffered from a sprained ankle due to excessive inversion of the foot. This would most likely demenstrated a torn of? A. Anterior talofibular ligament B. Posterior talofibular ligament C. Tendocalcaneus D. Deltoid ligament E. Long plantar ligament

Ankle is the most frequently injured major joint in the body. Ankle sprains (torn fibers of ligaments) are most common. A sprained ankle is nearly always an inversion injury. Lateral ligament is more frequently injured that medial ligament because it is much weaker. The anterior talofibular ligament, a part of lateral ligament is most vulnerable and most commonly torn during ankle sprains because it is much weaker. Severe eversion - deltoid or medial ligament. (Moore)

LYNN DARYL FELICIANO VILLAMATE R, MD (TOP 5 - FEB 2015 MED BOARDS; TOPNOTCH MD FROM EAC)

MIDTERM 1 EXAM - AUG 2015

136

Patient underwent splenectomy after he suffered from a vehicular crash severe injury to the spleen. Which of the following ligaments is the most vascular? A. Gastrosplenic ligament B. Splenorenal ligament C. Splenocolic D. Splenophrenic E. Spleno-omental

Splenorenal ligament connects the spleen to the left kidney. Between the layers of ths ligament, the splenic artery divides into 5 or more branches that enter the hilum. Gastrosplenic ligament is closely related to to short gastricvessels.

LYNN DARYL FELICIANO VILLAMATE R, MD (TOP 5 - FEB 2015 MED BOARDS; TOPNOTCH MD FROM EAC)

MIDTERM 1 EXAM - AUG 2015

TOPNOTCH MEDICAL BOARD PREP ANATOMY SUPEREXAM Page 18 of 94 For inquiries visit www.topnotchboardprep.com.ph or email us at [email protected]

MIDTERM 1 EXAM - AUG 2015

MIDTERM 1 EXAM - AUG 2015

TOPNOTCH MEDICAL BOARD PREP ANATOMY SUPEREXAM For inquiries visit www.topnotchboardprep.com.ph or email us at [email protected] Item # 137

QUESTION

EXPLANATION

Which of the following blood vessel joins the splenic vein to form the portal vein? A. Inferior mesenteric vein B. Right gastric vein C. Inferior mesenteric vein D. Superior mesenteric vein E. Hepatic vein

The portal vein is about 5 cm long and is formed behind the neck of the pancreas by the union of superior mesenteric and splenic veins. It ascends to the right, behind the first part of duodenum. It then runs upward in front of the opening into the lesser sac to the porta hepatis, where it divides into right and left terminal branches.

138

When oculomotor nerve is affected, patient may present with A. Inability to blink B. Extorsion of the eye C. Loss of afferent pupillary reflex D. Drooping of the eyelid E. Medial deviation of the affected eye

loss of corneal/blink reflex - CN VII; Extorsion of the eye and weakness of downward gaze - CN IV; loss of afferent pupillary reflex, blindness - CN II; Medial deviation - CN VI; Ptosis, dilated and fixed pupil, eyes turned down and out - CN III

139

Which of the following segment of the colon is most susceptible to ischemia? A. Cecum B. Ascending colon C. Descending colon D. Splenic flexure E. Sigmoid



140

Which of the following statements is true regarding renal cortex? A. It is thicker and has a lighter brown in color thnan the medulla. B. It contains the upper expanded end of the ureter. C. The apices of the renal pyramids extends into the renal cortex. D. The renal fascia closely adheres to the renal cortex and is continuously laterally with fascia transversalis. E. It extends into the medulla between adjacent pyramids.

The kidney has a dark brown outer cortex and a light brown inner medulla. The cortex closely adheres to renal capsule, not the renal fascia. The renal cortex extends into the medulla between adjacent pyramids as the renal columns, while the medulla is composed of renal pyramids, each having its base oriented toward the cortex and itx apex, the renal papilla, projecting medially.

141

A patient with leukemia needs to undergo bone marrow biopsy. The following bones are ideal for bone marrow biopsy except: A. Sternum B. Femur C. Posterior iliac crest D. Tibia E. None of the above

All the other options are the usual sites for bone marrow biopsy, the Posterior iliac crest and sternum are for adults, in children, the tibia especially the anterior portion may be used and the vertebra

142

Articular cartilage is mostly made up of this proteoglycan A. Chondroitin sulfate B. Keratan sulfate C. Heparan sulfate D. Dermatan sulfate E. Heparin

Chondroitin sulfate is the major component of articular cartilage, in combination with some keratan sulfate, it forms aggrecan which is the major component of articular cartilage.

143

Tennis elbow involves which among of the following muscle? A. Extensor carpi radialis B. Biceps brachii C. Brachioradialis D. Flexor carpi ulnaris E. Flexor carpi radialis

The extensor muscles of the wrist is involved in tennis elbow

144

The following are muscles for dorsiflexion of the ankle except: A. Extensor hallucis longus B. Extensor digitorum longus C. Peroneus tertius D. Tibialis anterior E. None of the above

All of the muscles above are used for dorsiflexion of the ankle

AUTHOR LYNN DARYL FELICIANO VILLAMATE R, MD (TOP 5 - FEB 2015 MED BOARDS; TOPNOTCH MD FROM EAC) LYNN DARYL FELICIANO VILLAMATE R, MD (TOP 5 - FEB 2015 MED BOARDS; TOPNOTCH MD FROM EAC) LYNN DARYL FELICIANO VILLAMATE R, MD (TOP 5 - FEB 2015 MED BOARDS; TOPNOTCH MD FROM EAC) LYNN DARYL FELICIANO VILLAMATE R, MD (TOP 5 - FEB 2015 MED BOARDS; TOPNOTCH MD FROM EAC)

EDWARD HARRY VALLAJERA, MD (TOP 8 - FEB 2015 MED BOARDS; TOPNOTCH MD FROM PERPETUAL BINAN) EDWARD HARRY VALLAJERA, MD (TOP 8 - FEB 2015 MED BOARDS; TOPNOTCH MD FROM PERPETUAL BINAN) EDWARD HARRY VALLAJERA, MD (TOP 8 - FEB 2015 MED BOARDS; TOPNOTCH MD FROM PERPETUAL BINAN) EDWARD HARRY VALLAJERA, MD (TOP 8 - FEB 2015 MED BOARDS; TOPNOTCH MD FROM PERPETUAL

TOPNOTCH MEDICAL BOARD PREP ANATOMY SUPEREXAM Page 19 of 94 For inquiries visit www.topnotchboardprep.com.ph or email us at [email protected]

TOPNOTCH EXAM MIDTERM 1 EXAM - AUG 2015

MIDTERM 1 EXAM - AUG 2015

MIDTERM 1 EXAM - AUG 2015

MIDTERM 1 EXAM - AUG 2015

MIDTERM 2 EXAM - AUG 2015

MIDTERM 2 EXAM - AUG 2015

MIDTERM 2 EXAM - AUG 2015

MIDTERM 2 EXAM - AUG 2015

TOPNOTCH MEDICAL BOARD PREP ANATOMY SUPEREXAM For inquiries visit www.topnotchboardprep.com.ph or email us at [email protected] Item #

QUESTION

EXPLANATION

AUTHOR

TOPNOTCH EXAM

BINAN)

145

During subclavian vein catheterization, how would you insert the needle as you puncture the inferior border of the clavicle in the infraclavicular approach? A. Upward and directed posterior to the middle of the SCM B. Upward and directed to the suprasternal notch C. Downward D. Straight E. None of the above

The 18-gauge introducer needle is inserted 1 cm inferior to the junction of the middle and proximal third of the clavicle while aiming slightly cephalad toward your index finger in the suprasternal notch.

146

Which of the following veins in the arm is used for large vein catheterization due to its increasing diameter as it goes proximally? A. Cephalic B. Basilic C. Median cubital vein D. Innominate vein E. None of the above

The basilic vein is most commonly used as a venous access for venous catheterization because it becomes larger as goes proximally

147

A man sustained a stab wound located at the 4th ICS directed inferiorly, you were able to appreciate Beck's triad. What is the quickest way to perform pericardiocentesis? A. 4th ICS parasternal line left B. 4th ICS parasternal line right C. 5th ICS midclavicular line D. Subxiphoid E. 2nd ICS parasternal line left

The needle is inserted at the subxiphoid area and is directed superiorly and to the left aiming for the patient's left shoulder.

148

At which tracheal cartilage would you perform tracheostomy? A. 1st and 2nd B. 2nd and 3rd C. 3rd and 4th D. 4th and 5th E. cricothyroid membrane

The 2nd and 3rd tracheal rings are the best sites to perform tracheostomy but care must be noted to avoid hitting the isthmus of the thyroid gland which is highly vascularized. If ever the thyroid isthmus is encountered, it should be retracted inferiorly.

149

The dorsalis pedis pulse is best appreciated where? A. Between 1st and 2nd metatarsal space B. Between 2nd and 3rd metatarsal space C. 3rd and 4th metatarsal space D. Between 4th and 5th metatarsal space E. Behind medial malleolus

It is appreciated best between the 1st and 2nd metatarsal space of the foot.

150

The dorsalis pedis pulse is related to which structure? A. Lateral to the extensor hallucis longus B. Medial to the extensor hallucis longus C. Lateral to the flexor hallucis longus D. Lateral to the medial malleolus E. None of the above

It is closely related to the lateral border of the extensor hallucis longus of the 1st toe.

151

The spleen is closest to which of the following? A. Kidney B. Stomach C. Splenic flexure D. Tail of the pancreas E. Inferior border of the diaphragm

The spleen is closest to the inferior border of the diaphragm

EDWARD HARRY VALLAJERA, MD (TOP 8 - FEB 2015 MED BOARDS; TOPNOTCH MD FROM PERPETUAL BINAN) EDWARD HARRY VALLAJERA, MD (TOP 8 - FEB 2015 MED BOARDS; TOPNOTCH MD FROM PERPETUAL BINAN) EDWARD HARRY VALLAJERA, MD (TOP 8 - FEB 2015 MED BOARDS; TOPNOTCH MD FROM PERPETUAL BINAN) EDWARD HARRY VALLAJERA, MD (TOP 8 - FEB 2015 MED BOARDS; TOPNOTCH MD FROM PERPETUAL BINAN) EDWARD HARRY VALLAJERA, MD (TOP 8 - FEB 2015 MED BOARDS; TOPNOTCH MD FROM PERPETUAL BINAN) EDWARD HARRY VALLAJERA, MD (TOP 8 - FEB 2015 MED BOARDS; TOPNOTCH MD FROM PERPETUAL BINAN) EDWARD HARRY VALLAJERA, MD (TOP 8 - FEB 2015 MED BOARDS; TOPNOTCH MD FROM PERPETUAL BINAN)

TOPNOTCH MEDICAL BOARD PREP ANATOMY SUPEREXAM Page 20 of 94 For inquiries visit www.topnotchboardprep.com.ph or email us at [email protected]

MIDTERM 2 EXAM - AUG 2015

MIDTERM 2 EXAM - AUG 2015

MIDTERM 2 EXAM - AUG 2015

MIDTERM 2 EXAM - AUG 2015

MIDTERM 2 EXAM - AUG 2015

MIDTERM 2 EXAM - AUG 2015

MIDTERM 2 EXAM - AUG 2015

TOPNOTCH MEDICAL BOARD PREP ANATOMY SUPEREXAM For inquiries visit www.topnotchboardprep.com.ph or email us at [email protected] Item # 152

QUESTION

EXPLANATION

AUTHOR EDWARD HARRY VALLAJERA, MD (TOP 8 - FEB 2015 MED BOARDS; TOPNOTCH MD FROM PERPETUAL BINAN) EDWARD HARRY VALLAJERA, MD (TOP 8 - FEB 2015 MED BOARDS; TOPNOTCH MD FROM PERPETUAL BINAN) EDWARD HARRY VALLAJERA, MD (TOP 8 - FEB 2015 MED BOARDS; TOPNOTCH MD FROM PERPETUAL BINAN)

TOPNOTCH EXAM MIDTERM 2 EXAM - AUG 2015

Which of the following cells produces the fibers that form the structural framework of most lymphatic organs? A. Mononuclear cells B. Reticular cells C. Macrophages D. Bilobed cells E. None of the above

These cells produce the fibers that form the structure of lymphatic organs

153

The spleen is easily ruptured in blunt abdominal trauma because of which of the following factors: A. Highly vascular organ B. Soft parenchyma C. Sudden deceleration and acceleration D. Direct blow to the area of the spleen E. All of the above

All of the above are possible causes of the spleen ruputuring in blunt abdominal trauma.

154

A patient came in the ER due to blunt abdominal trauma secondary to a vehicular accident, the patient is awake and conversant and just complains of a minimal vague abdominal pain, vital signs are still stable yet you were able to elicit a positive Kehr's sign, what would you do next? A. Perform emergency exploratory laparotomy B. Observe the patient C. GIve analgesics D. Send the patient home E. None of the above

The patient has probably a ruptured spleen, Kehr's sign is the occurrence of acute pain in the tip of the shoulder due to the presence of blood or other irritants in the peritoneal cavity when a person is lying down and the legs are elevated. Kehr's sign in the left shoulder is considered a classical symptom of a ruptured spleen.

155

A 1 year old boy is brought to you due to vomiting everytime after feeding and passage of jelly like stools, you were able to appreciate a small hard mass on the right lower quadrant, what would you do next? A. Perform exploratory laparotomy B. Perform a barium enema C. Observe the patient D. Give an anxiolytic E. None of the above

The treatment of choice for intussusception in this age group is barium enema or air enema accompanied by GI decompression by inserting a NGT.

EDWARD HARRY VALLAJERA, MD (TOP 8 - FEB 2015 MED BOARDS; TOPNOTCH MD FROM PERPETUAL BINAN)

MIDTERM 2 EXAM - AUG 2015

156

A 5 year old accidentally swallowed a 5 peso coin, on performing radiograph, the coin was located at the level of T4, the coin on AP CXR looks like a slit while on lateral view it looks round, you know that the coin is most likely in the: A. Esophagus B. Bifurcation of the trachea C. Right main bronchus D. Left main bronchus E. Inferior right bronchopulmonary segment

The orientation of the coin on both anterior and lateral views suggest that the coin is at the level of the bifurcation of the trachea

EDWARD HARRY VALLAJERA, MD (TOP 8 - FEB 2015 MED BOARDS; TOPNOTCH MD FROM PERPETUAL BINAN)

MIDTERM 2 EXAM - AUG 2015

157

All of the following organs actively fight pathogens except: A. Cervical lymph nodes B. Thymus C. Spleen D. Axillary lymph nodes E. Tonsils

The thymus is the site of maturation and selection of self-tolerant T lymphocytes

MIDTERM 2 EXAM - AUG 2015

158

Which of the following does not empty into the thoracic duct? A. Right subclavian trunk B. Left jugular trunk C. Cisterna chyli D. Left subclavian trunk E. None of the above

The right subclavian trunk drains into the right lymphatic duct to enter the right subclavian vein

EDWARD HARRY VALLAJERA, MD (TOP 8 - FEB 2015 MED BOARDS; TOPNOTCH MD FROM PERPETUAL BINAN) EDWARD HARRY VALLAJERA, MD (TOP 8 - FEB 2015 MED BOARDS; TOPNOTCH MD FROM PERPETUAL BINAN)

TOPNOTCH MEDICAL BOARD PREP ANATOMY SUPEREXAM Page 21 of 94 For inquiries visit www.topnotchboardprep.com.ph or email us at [email protected]

MIDTERM 2 EXAM - AUG 2015

MIDTERM 2 EXAM - AUG 2015

MIDTERM 2 EXAM - AUG 2015

TOPNOTCH MEDICAL BOARD PREP ANATOMY SUPEREXAM For inquiries visit www.topnotchboardprep.com.ph or email us at [email protected] Item # 159

QUESTION

EXPLANATION

As lymph flows through a lymph node, which of the following would it go through last A. Subcapsular sinus B. Afferent lymphatic vessel C. Medullary sinus D. Cortical sinus E. None of the above

It flows from the afferent vessel then to the subcapsular sinus then to the cortical sinus then to the medullary sinus and finally into the efferent vessel

160

Most of the body's mucosa-associated lymphatic tissue can be found in which of the following A. Respiratory and Reproductive tract B. Gastrointestinal and urinary tract C. Urinary and reproductive tract D. Respiratory and gastrointestinal tract E. None of the above

These 2 systems are constantly exposed to pathogens and are therefore with the most number of mucosa associated lymphatic tissue.

161

The dorsalis pedis pulse can be palpated immediately on which part of the foot? A. Medially to the external hallucis longus tendon B. Lateral to the external digitorum longus tendon C. Medial to the tibialis anterior tendon D. Lateral to the external hallucis longus tendon E. lateral to the tibialis anterior tendon

PBEQ

162

The dorsalis pedis artery courses its way distally and can be palpated between the following landmarks. A. 1st and 2nd metatarsals B. 2nd and 3rd metatarsals C. 3rd an 4th metatarsals D. 1st and 2nd proximal phalanges E. 2nd and 3rd proximal phalanges

PBEQ

163

Which of the following part is mostly affected in tennis elbow? A. Medial part of the elbow B. lateral part of the elbow C. superior part of the elbow D. Inferior part of the elbow E. Posterior part of the elbow

tennis elbow is lateral epicondylitis

164

25. SB, a 35 year old male, was diagnosed with hypertension just last week. Her doctor decided to start him with hydrochlorothiazide. Where is the site of action of the drug? A. PCT B. DCT C. CT D. LOH E. Glomerulus

hydrochlorothiazide inhibits sodium reabsorption in distal renal tubules resulting in increased excretion of water and of Na, K, and hydrogen ions

165

The breast receives its blood supply from which of the following blood vessels? A. Perforating branches of internal mammary artery B. Lateral branches of the posterior intercostal arteries C. Branches from axillary artery D. A and B E. All of the above



166

By weight, which of the following is considered the largest organ of the body? A. Skin B. Liver C. Lung D. Heart E. Brain

ft if the question is largest organ, the answer should be liver since the skin is not considered an organ but an organ system

167

Which among the following muscle is considered as the longest muscle of the human body? A. Gluteus maximus B. Latissimus dorsi C. sartorius D. Quadriceps femoris E. Psoas major

gluteus maximus is generally the biggest. Latissimus dorsi is the widest

AUTHOR EDWARD HARRY VALLAJERA, MD (TOP 8 - FEB 2015 MED BOARDS; TOPNOTCH MD FROM PERPETUAL BINAN) EDWARD HARRY VALLAJERA, MD (TOP 8 - FEB 2015 MED BOARDS; TOPNOTCH MD FROM PERPETUAL BINAN) HAROLD JAY S. BAYTEC, MD (TOP 10 - FEB 2015 MED BOARDS; TOPNOTCH MD FROM FEU) HAROLD JAY S. BAYTEC, MD (TOP 10 - FEB 2015 MED BOARDS; TOPNOTCH MD FROM FEU) HAROLD JAY S. BAYTEC, MD (TOP 10 - FEB 2015 MED BOARDS; TOPNOTCH MD FROM FEU) HAROLD JAY S. BAYTEC, MD (TOP 10 - FEB 2015 MED BOARDS; TOPNOTCH MD FROM FEU) HAROLD JAY S. BAYTEC, MD (TOP 10 - FEB 2015 MED BOARDS; TOPNOTCH MD FROM FEU) HAROLD JAY S. BAYTEC, MD (TOP 10 - FEB 2015 MED BOARDS; TOPNOTCH MD FROM FEU) HAROLD JAY S. BAYTEC, MD (TOP 10 - FEB 2015 MED BOARDS; TOPNOTCH MD FROM FEU)

TOPNOTCH MEDICAL BOARD PREP ANATOMY SUPEREXAM Page 22 of 94 For inquiries visit www.topnotchboardprep.com.ph or email us at [email protected]

TOPNOTCH EXAM MIDTERM 2 EXAM - AUG 2015

MIDTERM 2 EXAM - AUG 2015

MIDTERM 3 EXAM - AUG 2015

MIDTERM 3 EXAM - AUG 2015

MIDTERM 3 EXAM - AUG 2015

MIDTERM 3 EXAM - AUG 2015

MIDTERM 3 EXAM - AUG 2015

MIDTERM 3 EXAM - AUG 2015

MIDTERM 3 EXAM - AUG 2015

TOPNOTCH MEDICAL BOARD PREP ANATOMY SUPEREXAM For inquiries visit www.topnotchboardprep.com.ph or email us at [email protected] Item # 168

QUESTION

EXPLANATION

AUTHOR

TOPNOTCH EXAM MIDTERM 3 EXAM - AUG 2015

All of the following statements regarding the vessels of the vagina EXCEPT: A. Lymphatics from the middle third drain into the external iliac nodes B. The proximal portion is supplied by the cervical branch of the uterine artery C. the middle rectal artery contributes to supply the posterior vaginal wall D. distal walls receive contributions from the internal pudendal artery E. extensive venous plexus immediately sorrounds the vagina and follows the course of the arteries

lymphatics from the middle third drain into the internal iliac nodes. the table combined the lymphatic drainage of the upper and middle 3rd. OB Williams 23rd edition page 18. "Those from the middle third drain into the internal iliac nodes, and those from the upper third drain into the external, internal, and common iliac nodes."

HAROLD JAY S. BAYTEC, MD (TOP 10 - FEB 2015 MED BOARDS; TOPNOTCH MD FROM FEU)

169

This structure originated from the densest portion of the broad ligament and is considered as provider of the major support for the uterus and cervix A. Cardinal ligament B. Mackenrodt ligament C. Transverse cervical ligament D. None of the above E. all of the above

A B and C are all the same.

HAROLD JAY S. BAYTEC, MD (TOP 10 - FEB 2015 MED BOARDS; TOPNOTCH MD FROM FEU)

MIDTERM 3 EXAM - AUG 2015

170

What is the most common type of leiomyoma of the uterus? A. subserous B. Intramural C. submucous D. cervical E. None of the above

SIMILAR TO PREVIOUS BOARD EXAM CONCEPT/PRINCIPLE

MIDTERM 3 EXAM - AUG 2015

171

In a kidney surgery, when the surgeon penetrated the peritoneum, the next layer that he will encounter is A. Pararenal fat B. Renal fascia C. Perinephric fat or perirenal fat D. Renal capsule E. Renal cortex

the choices are arranged from the outer to inner layers. almost all the pictures that i saw online and on books have a fat in between the peritoneum and the gerotas fascia, and almost all sources describe this layer as the paranephric fat.. SIMILAR TO PREVIOUS BOARD EXAM CONCEPT/PRINCIPLE

172

Gerota's fascia is the structure that immediately encloses both the kidneys and suprarenal glands. This fascia is also called as A. Pararenal fat B. Renal fascia C. Perinephric fat or perirenal fat D. Renal capsule E. Renal cortex

SIMILAR TO PREVIOUS BOARD EXAM CONCEPT/PRINCIPLE

173

Which of the following structures is NOT derived from endoderm? A. bronchus B. trachea C. spleen D. stomach E. Duodenum

spleen is from mesoderm

174

Peyer's patches are seen in what part of the GIT? A. Large intestines B. Ileum C. jejunum D. duodenum E. Stomach

SIMILAR TO PREVIOUS BOARD EXAM CONCEPT/PRINCIPLE. Peyer's patches are only found in ileum

175

Brunner's glands are found in what part/s of the GIT A. Stomach B. jejunum C. ileum D. duodenum E. B and D

Brunner's gland are found in duodenum and peyer's patches are found in ileum. No peyer's patches nor brunner's glands are found in jejunum

HAROLD JAY S. BAYTEC, MD (TOP 10 - FEB 2015 MED BOARDS; TOPNOTCH MD FROM FEU) HAROLD JAY S. BAYTEC, MD (TOP 10 - FEB 2015 MED BOARDS; TOPNOTCH MD FROM FEU) HAROLD JAY S. BAYTEC, MD (TOP 10 - FEB 2015 MED BOARDS; TOPNOTCH MD FROM FEU) HAROLD JAY S. BAYTEC, MD (TOP 10 - FEB 2015 MED BOARDS; TOPNOTCH MD FROM FEU) HAROLD JAY S. BAYTEC, MD (TOP 10 - FEB 2015 MED BOARDS; TOPNOTCH MD FROM FEU) HAROLD JAY S. BAYTEC, MD (TOP 10 - FEB 2015 MED BOARDS; TOPNOTCH MD FROM FEU)

TOPNOTCH MEDICAL BOARD PREP ANATOMY SUPEREXAM Page 23 of 94 For inquiries visit www.topnotchboardprep.com.ph or email us at [email protected]

MIDTERM 3 EXAM - AUG 2015

MIDTERM 3 EXAM - AUG 2015

MIDTERM 3 EXAM - AUG 2015

MIDTERM 3 EXAM - AUG 2015

MIDTERM 3 EXAM - AUG 2015

TOPNOTCH MEDICAL BOARD PREP ANATOMY SUPEREXAM For inquiries visit www.topnotchboardprep.com.ph or email us at [email protected] Item # 176

QUESTION

EXPLANATION

Which of the following muscles is LEAST likely to cause elevation of the hyoid bone? A. hypoglossus B. stylohyoid C. Geniohyoid D. Mylohyoid E. Thyrohyoid

Thyrohyoid is attached to the inferior of the hyoid bone so it depresses the latter. choice A is hypoglossus and not hyoglossus. i intentionally put the choice A which is hyPOglossus (CN XI) to make a little trick to the question just like a typical board exam question. some might have read the choice A as hyoglossus which is a muscle which does not elevate the hyoid bone.

177

The sternal angle of Louis is an important landmark of the chest because it lies on the following structures EXCEPT: A. Second costal cartilage B. Bifurcation of the trachea C. Intervertebral disc between T2 and T3 D. Junction of the superior and inferior mediastinum E. junction of the ascending aorta and the aortic arch

IVD of T4 and T5

178

A patient came in at the ER due to gunshot wound on the abdomen which prompted exploratory laparotomy. Upon opening, you saw that a portion of the abdominal aorta was hit. Which of the following part of the large intestine is most vulnerable for ischemia? A. cecum B. ascending colon C. hepatic flexure D. splenic flexure E. sigmoid

SIMILAR TO PREVIOUS BOARD EXAM CONCEPT/PRINCIPLE. It is the part of the large intestines that is farthest from the SMA and IMA

179

The total refractive power of the eye is mostly contributed by: A. cornea B. pupil C. lens D. Vitreous humor E. Acqueous humor

2/3 of the total refractive power is contributed by cornea

180

Cricoid cartilage can be seen at what vertebral level? A. C4 B. C5 C. C6 D. C7 E. T1



181

You can palpate the pulsations of dorsalis pedis artery in what location in the foot? A. Between 1st and 2nd metatarsals B. Between 2nd and 3rd metatarsals C. Between 3rd and 4th metatarsals D. In front of the ankle joint E. Medial to 1st metatarsal

SIMILAR TO PREVIOUS BOARD EXAM CONCEPT/PRINCIPLE.

182

What is the lymph drainage of the lower 1/3 of esophagus? A. Bronchomediastinal nodes B. Tracheobronchial nodes C. Paraaortic nodes D. Left gastric nodes E. Posterior mediastinal nodes

SIMILAR TO PREVIOUS BOARD EXAM CONCEPT/PRINCIPLE.. Lymph vessels from the upper third of the esophagus drain into the deep cervical nodes, from the middle third into the superior and posterior mediastinal nodes, and from the lower third into nodes along the left gastric blood vessels and the celiac nodes

183

During TAHBSO of a 45 year old G3P3, what part of the ureter is most commonly injured? A. Proximal 1/3 B. Middle 1/3 C. Distal 1/3 D. B and C E. None of the above

SIMILAR TO PREVIOUS BOARD EXAM CONCEPT/PRINCIPLE.

AUTHOR HAROLD JAY S. BAYTEC, MD (TOP 10 - FEB 2015 MED BOARDS; TOPNOTCH MD FROM FEU) HAROLD JAY S. BAYTEC, MD (TOP 10 - FEB 2015 MED BOARDS; TOPNOTCH MD FROM FEU) HAROLD JAY S. BAYTEC, MD (TOP 10 - FEB 2015 MED BOARDS; TOPNOTCH MD FROM FEU)

HAROLD JAY S. BAYTEC, MD (TOP 10 - FEB 2015 MED BOARDS; TOPNOTCH MD FROM FEU) HAROLD JAY S. BAYTEC, MD (TOP 10 - FEB 2015 MED BOARDS; TOPNOTCH MD FROM FEU) JEAN PAOLO M. DELFINO, MD (TOP 10 - FEB 2015 MED BOARDS; TOPNOTCH MD FROM FATIMA) JEAN PAOLO M. DELFINO, MD (TOP 10 - FEB 2015 MED BOARDS; TOPNOTCH MD FROM FATIMA) JEAN PAOLO M. DELFINO, MD (TOP 10 - FEB 2015 MED BOARDS; TOPNOTCH MD FROM FATIMA)

TOPNOTCH MEDICAL BOARD PREP ANATOMY SUPEREXAM Page 24 of 94 For inquiries visit www.topnotchboardprep.com.ph or email us at [email protected]

TOPNOTCH EXAM MIDTERM 3 EXAM - AUG 2015

MIDTERM 3 EXAM - AUG 2015

MIDTERM 3 EXAM - AUG 2015

MIDTERM 3 EXAM - AUG 2015

MIDTERM 3 EXAM - AUG 2015

FINAL EXAM - AUG 2015

FINAL EXAM - AUG 2015

FINAL EXAM - AUG 2015

TOPNOTCH MEDICAL BOARD PREP ANATOMY SUPEREXAM For inquiries visit www.topnotchboardprep.com.ph or email us at [email protected] Item # 184

QUESTION

EXPLANATION

AUTHOR

In doing subclavian vein catheterization, the needle should be inserted through the skin just below the lower border of the clavicle at the junction of the medial third and outer two thirds. The needle is then pointed in what direction? A. downward and medially toward the mediastinum B. downward and medially towards the opposite nipple C. downward and posteriorly towards the suprasternal notch D. upward and posteriorly toward the suprasternal notch E. upward and medially towards the opposite nipple

SIMILAR TO PREVIOUS BOARD EXAM CONCEPT/PRINCIPLE.. The question describes an Infraclavicular approach: needle is inserted at lower border of clavicle, then it is pointed upward and posteriorly toward the middle of the suprasternal notch. Supraclavicular approach: The needle is inserted through the skin at the site where the posterior border of the clavicular origin of sternocleidomastoid is attached to the upper border of the clavicle. then it is pointed downward and medially toward the mediastinum.

JEAN PAOLO M. DELFINO, MD (TOP 10 - FEB 2015 MED BOARDS; TOPNOTCH MD FROM FATIMA)

185

The first branch of the internal iliac artery supplies what organ? A. Middle portion of ureter B. Bladder C. Uterus D. Rectum E. Prostate gland



186

Which of the following intrinsic muscles of the larynx can abduct the vocal fold? A. Cricothyroid B. Transverse arytenoid C. Posterior cricoarytenoid D. Lateral cricoarytenoid E. Oblique arytenoid

Posterior cricoarytenoid abducts the vocal cords by rotating arytenoid cartilage

187

During splenectomy, injury to what structure can result to damage of pancreas? A. Gastrosplenic ligament B. Splenorenal ligament C. Hepatosplenic ligament D. Splenocolic ligament E. Pancreaticosplenic ligament

SIMILAR TO PREVIOUS BOARD EXAM CONCEPT/PRINCIPLE.. The presence of the tail of the pancreas in the splenicorenal ligament sometimes results in its damage during splenectomy

188

What is the most common cause of aortic aneurysm? A. Atherosclerosis B. Medial degeneration C. Congenital D. Infection E. None of the above

SIMILAR TO PREVIOUS BOARD EXAM CONCEPT/PRINCIPLE.. Atherosclerosis is the most common cause of aortic aneurysm.

189

What is the location of epididymis in relation to the testis? A. anteromedial B. medial C. posterolateral D. posterior E. superolateral

SIMILAR TO PREVIOUS BOARD EXAM CONCEPT/PRINCIPLE.

190

What is the lining epithelium of seminal vesicles? A. transitional B. pseudostratified columnar C. simple columnar D. simple cuboidal E. stratified columnar

seminal vesicles, prostate gland, epididymis and vas deferens have pseudostratified columnar nonciliated lining epithelium. The latter 2 have stereocilia.

191

What structure divides the subclavian artery into 3 segments? A. 1st rib B. subclavius C. pectoralis minor D. teres major E. scalene anterior

Scalene anterior divides the subclavian artery into 3 segments. 1st segment is medial to the muscle, 2nd segment is posterior, and the 3rd segment is lateral to the muscle.

192

The posterior belly of digastric is innervated by what nerve? A. CN X B. CN IX C. CN VIII D. CN VII E. CN V

Facial nerve innervates structures derived from the 2nd pharyngeal arch: muscles of facial expression, stapedius, stylohyoid and posterior bellt of digastric. The anterior belly of digastric is innervated by trigeminal nerve.

JEAN PAOLO M. DELFINO, MD (TOP 10 - FEB 2015 MED BOARDS; TOPNOTCH MD FROM FATIMA) JEAN PAOLO M. DELFINO, MD (TOP 10 - FEB 2015 MED BOARDS; TOPNOTCH MD FROM FATIMA) JEAN PAOLO M. DELFINO, MD (TOP 10 - FEB 2015 MED BOARDS; TOPNOTCH MD FROM FATIMA) JEAN PAOLO M. DELFINO, MD (TOP 10 - FEB 2015 MED BOARDS; TOPNOTCH MD FROM FATIMA) JEAN PAOLO M. DELFINO, MD (TOP 10 - FEB 2015 MED BOARDS; TOPNOTCH MD FROM FATIMA) JEAN PAOLO M. DELFINO, MD (TOP 10 - FEB 2015 MED BOARDS; TOPNOTCH MD FROM FATIMA) JEAN PAOLO M. DELFINO, MD (TOP 10 - FEB 2015 MED BOARDS; TOPNOTCH MD FROM FATIMA) JEAN PAOLO M. DELFINO, MD (TOP 10 - FEB 2015 MED BOARDS; TOPNOTCH MD FROM FATIMA)

TOPNOTCH MEDICAL BOARD PREP ANATOMY SUPEREXAM Page 25 of 94 For inquiries visit www.topnotchboardprep.com.ph or email us at [email protected]

TOPNOTCH EXAM FINAL EXAM - AUG 2015

FINAL EXAM - AUG 2015

FINAL EXAM - AUG 2015

FINAL EXAM - AUG 2015

FINAL EXAM - AUG 2015

FINAL EXAM - AUG 2015

FINAL EXAM - AUG 2015

FINAL EXAM - AUG 2015

FINAL EXAM - AUG 2015

TOPNOTCH MEDICAL BOARD PREP ANATOMY SUPEREXAM For inquiries visit www.topnotchboardprep.com.ph or email us at [email protected] Item # 193

QUESTION

EXPLANATION

All of the following drains into the right atrium except A. Superior vena cava B. Coronary sinus C. Small cardiac vein D. Anterior cardiac vein E. Thebesian vein

SIMILAR TO PREVIOUS BOARD EXAM CONCEPT/PRINCIPLE.. SVC, IVC, coronary sinus, anterior cardiac veins and least cardiac vein (aka Thebesian vein) drain into the right atrium. The following drains into the coronary sinus: great cardiac vein, middle cardiac vein, small cardiac vein, posterior vein of left ventricle.

194

During a difficult dental extraction, patient accidentally aspirated a tooth. He was immediately brought to you at the ER. Upon PE, you noted that breath sounds are decreased on an area of the chest. There was noted intermittent cough. Where is the foreign body probably located? A. trachea B. R primary bronchus C. L secondary bronchus D. Larynx E. carina

Because the right bronchus is the wider and more direct continuation of the trachea, foreign bodies tend to enter the right instead of the left bronchus.

195

What is the reason for testis descent during infancy? A. Maternal estrogen B. Testosterone C. Temperature D. pH E. Gravity

In the male, the testis descends through the pelvis and inguinal canal during the seventh and eighth months of fetal life. The normal stimulus for the descent of the testis is testosterone, which is secreted by the fetal testes.SIMILAR TO PREVIOUS BOARD EXAM CONCEPT/PRINCIPLE.

196

Lesion in CN III will result to which eye position? A. Abducted and depressed B. Adducted and extorted C. Abducted and extorted D. Adducted and depressed E. A and C

CN III supplies all extraocular muscles except lateral rectus and superior oblique. The eye position after injury to CN III is abducted and depressed. Lesion in CN IV would result to extorted eyeball while lesion in CN VI would result to adducted position.

197

Thiazide diuretic acts on this portion of the nephron? A. PCT B. DCT C. Ascending limb of LOH D. Collecting duct E. Collecting tubule

giveaway question last boards

198

Which structure is not traversed during pericardiocentesis? A. Rectus sheath B. Rectus abdominis C. Serous pericardium D. Endothoracic fascia E. Fibrous layer of pericardium

Layers traversed during pericardiocentesis: subxiphoid approach- skin, fascia, rectus sheath, rectus abdominis, fibrous layer of pericardium, parietal layer of visceral pericardium

199

Goblet cells are present until what structure in the respiratory system? A. Trachea B. Bronchus C. Bronchiole D. Terminal bronchiole E. Respiratory bronchiole

Respiratory epithelium (pseudostratified columnar with goblet cells) is present up to the level of bronchioles.

200

The following are anterior relations of the right kidney except? A. adrenal B. liver C. transversus abdominis D. R colic flexure E. 2nd part of duodenum

Transversus abdominis is at the posterior of the kidney, not anterior

201

This shunt muscle contains the median nerve and the brachial artery. A. Biceps brachii B. Brachialis C. Coracobrachialis D. Deltoid E. Triceps brachii

With the deltoid and long head of the triceps, the coracobrachialis serves as a shunt muscle, resisting downward dislocation of th e head of the humerus, as when carrying a heavy suitcase.

AUTHOR JEAN PAOLO M. DELFINO, MD (TOP 10 - FEB 2015 MED BOARDS; TOPNOTCH MD FROM FATIMA) JEAN PAOLO M. DELFINO, MD (TOP 10 - FEB 2015 MED BOARDS; TOPNOTCH MD FROM FATIMA)

JEAN PAOLO M. DELFINO, MD (TOP 10 - FEB 2015 MED BOARDS; TOPNOTCH MD FROM FATIMA) JEAN PAOLO M. DELFINO, MD (TOP 10 - FEB 2015 MED BOARDS; TOPNOTCH MD FROM FATIMA) JEAN PAOLO M. DELFINO, MD (TOP 10 - FEB 2015 MED BOARDS; TOPNOTCH MD FROM FATIMA) JEAN PAOLO M. DELFINO, MD (TOP 10 - FEB 2015 MED BOARDS; TOPNOTCH MD FROM FATIMA) JEAN PAOLO M. DELFINO, MD (TOP 10 - FEB 2015 MED BOARDS; TOPNOTCH MD FROM FATIMA) JEAN PAOLO M. DELFINO, MD (TOP 10 - FEB 2015 MED BOARDS; TOPNOTCH MD FROM FATIMA) GRACE ARVIOLA, MD (TOP 3 - AUG 2014 MED BOARDS; TOPNOTCH MD)

TOPNOTCH MEDICAL BOARD PREP ANATOMY SUPEREXAM Page 26 of 94 For inquiries visit www.topnotchboardprep.com.ph or email us at [email protected]

TOPNOTCH EXAM FINAL EXAM - AUG 2015

FINAL EXAM - AUG 2015

FINAL EXAM - AUG 2015

FINAL EXAM - AUG 2015

FINAL EXAM - AUG 2015

FINAL EXAM - AUG 2015

FINAL EXAM - AUG 2015

FINAL EXAM - AUG 2015

DIAGNOSTIC EXAM - FEB 2015

TOPNOTCH MEDICAL BOARD PREP ANATOMY SUPEREXAM For inquiries visit www.topnotchboardprep.com.ph or email us at [email protected] Item # 202

QUESTION

EXPLANATION

The joint that is responsible for thumb opposition is: A. carpometacarpal joint B. interphalangeal joint C. radiocarpal joint D. metacarpophalangeal joint E. intercarpal joint

The CMC joint of the thumb is a saddle joint. Other CMC joints are of the plane type of synovial joint.

203

During CABG, the surgeon inserted his finger into the transverse pericardial sinus. What structures are demarcated? A. SVC, pulmonary trunk, and aorta B. IVC, pulmonary veins C. Right and left atria D. Right and left ventricles E. SVC and IVC

In the embryology of the heart, as the heart tube folds, its venous end moves posterosuperiorly so that the venous end of the tube lies adjacent to the arterial end, separated only by the transverse pericardial sinus.

204

The oblique vein of Marshall is a remnant of: A. Right IVC B. Right SVC C. Left SVC D. Left IVC E. Left brachiocephalic vein

The oblique vein of the left atrium (of Marshall) is a small vessel, relatively unimportant postnatally, that descends over the posterior wall of the left atrium and merges with the great cardiac vein to form the coronary sinus. This occasionally persists in adults, replacing or augmenting the right SVC. **Expect at least one question in embryology.**

205

This space is involved in inguinal hernia repair. A. Space of Burns B. Space of Bogros C. Space of Retzius D. Space of Douglas E. Space of Nuck

206

What vertebra demarcates the junction of the sigmoid colon and the rectum? A. L5 B. S1 C. S2 D. S3 E. S4

The space of Bogros is the anterolateral part of the potential space between the transversalis fascia and the parietal peritoneum. This provides a plane that can be opened without entering the membranous peritoneal sac. Thus, the risk of contamination is minimized. This sac is used for placing prostheses when repairing inguinal hernias. The sigmoid colon extends from the iliac fossa to the S3 segment, where it joins the rectum.

207

During childbirth, which levator ani muscle is most often injured? A. Puborectalis B. Pubococcygeus C. Iliococcygeus D. Coccygeus E. Sphincter urethra

208

AUTHOR GRACE ARVIOLA, MD (TOP 3 - AUG 2014 MED BOARDS; TOPNOTCH MD) GRACE ARVIOLA, MD (TOP 3 - AUG 2014 MED BOARDS; TOPNOTCH MD)

TOPNOTCH EXAM DIAGNOSTIC EXAM - FEB 2015

DIAGNOSTIC EXAM - FEB 2015

GRACE ARVIOLA, MD (TOP 3 - AUG 2014 MED BOARDS; TOPNOTCH MD) GRACE ARVIOLA, MD (TOP 3 - AUG 2014 MED BOARDS; TOPNOTCH MD) GRACE ARVIOLA, MD (TOP 3 - AUG 2014 MED BOARDS; TOPNOTCH MD)

DIAGNOSTIC EXAM - FEB 2015

The pubococcygeus, the main and most medial part of the levator ani, is torn most often during childbirth.

GRACE ARVIOLA, MD (TOP 3 - AUG 2014 MED BOARDS; TOPNOTCH MD)

DIAGNOSTIC EXAM - FEB 2015

Which structure forms the divide between vagal and pelvic splanchnic parasympathetic nerves? A. Hepatic flexure B. Splenic flexure C. Junction between sigmoid colon and rectum D. Junction between rectum and anus E. Dentate line

Orad to the left colic flexure, the parasympathetic innervation of the large intestine is vagallymediated. Aborad to this, it is innervated by pelvic splanchnic nerves.

GRACE ARVIOLA, MD (TOP 3 - AUG 2014 MED BOARDS; TOPNOTCH MD)

DIAGNOSTIC EXAM - FEB 2015

209

The Ligament of Treitz corresponds to what vertebra? A. L1 B. L2 C. L3 D. L4 E. L5

The ligament of Treitz marks the duodenojejunal junction at the level of L2 vertebra, 2-3 cm to the left of the midline.

DIAGNOSTIC EXAM - FEB 2015

210

What structures form the left sagittal fissure of the liver? A. Ligamentum teres and ligamentum venosun B. Right and left hepatic lobes C. Caudate lobe and quadrate lobe D. Galbblader and IVC E. Falciform ligament and coronary ligament

The gallblader and IVC form the right sagittal fissure.

211

Trigeminal neuralgia most commonly affects what division of cranial nerve V? A. V1 B. V2 C. V3 D. V4 E. None; it has no preferences

CN V1 is least frequently involved.

GRACE ARVIOLA, MD (TOP 3 - AUG 2014 MED BOARDS; TOPNOTCH MD) GRACE ARVIOLA, MD (TOP 3 - AUG 2014 MED BOARDS; TOPNOTCH MD) GRACE ARVIOLA, MD (TOP 3 - AUG 2014 MED BOARDS; TOPNOTCH MD)

TOPNOTCH MEDICAL BOARD PREP ANATOMY SUPEREXAM Page 27 of 94 For inquiries visit www.topnotchboardprep.com.ph or email us at [email protected]

DIAGNOSTIC EXAM - FEB 2015

DIAGNOSTIC EXAM - FEB 2015

DIAGNOSTIC EXAM - FEB 2015

DIAGNOSTIC EXAM - FEB 2015

TOPNOTCH MEDICAL BOARD PREP ANATOMY SUPEREXAM For inquiries visit www.topnotchboardprep.com.ph or email us at [email protected] Item # 212

QUESTION

EXPLANATION

Nodose ganglion is associated with what cranial nerve? A. V B. VII C. IX D. X E. XI

The nodose ganglion is the inferior ganglion of the vagus nerve that is concerned with the visceral sensory components of the nerve. The superior ganglion, on the other hand, is concerned with the general sensory component.

213

Serous demilunes are seen in: A. Parotid gland B. Sublingual gland C. Submandibular gland D. Lacrimal gland E. Sebaceous gland

Serous demilunes are found in glands that produce both watery and mucoid secretions, i.e. submandibular gland. They consist of elongated mucus acini that have a crescentic cap of serous cells over their ends.

214

Synapsis during meiosis occurs in: A. Prophase B. Metaphase C. Anaphase D. Telophase E. Interphase

Synapsis is the event wherein pairs of conjoined chromatids derived from homologous chromosomes come together on the equatorial plate to form groups of four chromatids called tetrads

215

Schmidt-Lantermann clefts are formed by: A. Cardiac myocytes B. Endothelial cells C. Goblet cells D. Schwann cells E. B-cells

These clefts are formed when Schwann cells envelope neurons with myelin. These represent thin threads of Schwann cell cytoplasm that pursue a spiral course from the cell body nearly to the axon.

216

The acrosome reaction of the sperm involves what receptor of the zona pellucida? A. ZP1 B. ZP2 C. ZP3 D. ZP4 E. ZP5

The zona pellucida is composed of three glycoproteins: ZP1, ZP2, and ZP3.

217

Which epithelium has only a single layer of cells? A. Skin B. Trachea C. Urinary bladder D. Cornea E. Sweat glands

The trachea is lined with pseudostratified columnar epithelium. Pseudostratified implies a single layer of cells that only appear as multiple layers when viewed under the light microscope.

218

Herring bodies contain: A. Oxytocin B. Prolactin C. Growth Hormone D. Melatonin E. Aldosterone

Herring bodies are found in the posterior pituitary. They contain ADH and oxytocin.

219

Which cells undergo endomitosis? A. Neurons B. Ovum C. RBCs D. Platelets E. Keratinocytes

In endomitosis, the DNA undergoes multiple replications without division of the cytoplasm. This results in a giant polypoid cell.

220

High endothelial venules are found in: A. Thymus B. Lymph nodes C. Bone marrow D. Spleen E. All of the above

HEVs are the portal of entry of new blood-borne lymphocytes.

AUTHOR GRACE ARVIOLA, MD (TOP 3 - AUG 2014 MED BOARDS; TOPNOTCH MD) GRACE ARVIOLA, MD (TOP 3 - AUG 2014 MED BOARDS; TOPNOTCH MD) GRACE ARVIOLA, MD (TOP 3 - AUG 2014 MED BOARDS; TOPNOTCH MD) GRACE ARVIOLA, MD (TOP 3 - AUG 2014 MED BOARDS; TOPNOTCH MD) GRACE ARVIOLA, MD (TOP 3 - AUG 2014 MED BOARDS; TOPNOTCH MD) GRACE ARVIOLA, MD (TOP 3 - AUG 2014 MED BOARDS; TOPNOTCH MD) GRACE ARVIOLA, MD (TOP 3 - AUG 2014 MED BOARDS; TOPNOTCH MD) GRACE ARVIOLA, MD (TOP 3 - AUG 2014 MED BOARDS; TOPNOTCH MD) GRACE ARVIOLA, MD (TOP 3 - AUG 2014 MED BOARDS; TOPNOTCH MD)

TOPNOTCH MEDICAL BOARD PREP ANATOMY SUPEREXAM Page 28 of 94 For inquiries visit www.topnotchboardprep.com.ph or email us at [email protected]

TOPNOTCH EXAM DIAGNOSTIC EXAM - FEB 2015

DIAGNOSTIC EXAM - FEB 2015

DIAGNOSTIC EXAM - FEB 2015

DIAGNOSTIC EXAM - FEB 2015

DIAGNOSTIC EXAM - FEB 2015

DIAGNOSTIC EXAM - FEB 2015

DIAGNOSTIC EXAM - FEB 2015

DIAGNOSTIC EXAM - FEB 2015

DIAGNOSTIC EXAM - FEB 2015

TOPNOTCH MEDICAL BOARD PREP ANATOMY SUPEREXAM For inquiries visit www.topnotchboardprep.com.ph or email us at [email protected] Item # 221

QUESTION Which of the following manifestation is consistent if the patient presents with a ® sphenoid wing meningioma? A. Inability to protrude the tongue B. Inability to close the ® eye and wrinkle the ipsilateral forehead C. Weakness in closing the jaw D. increase risk for aspiration E. nystagmus

EXPLANATION mass lesion on the ® sphenoid wing will directly compress structures at the middle cranial fossa. Skull base openings located at the sphenoid are as follows: optic canal, Superior orbital fissure, rotundum, spinsoum, ovale. Hypoglossal nerve responsibe for tongue movement is not affected since it exits through the hypoglossal canal found at the occipital bone.facial nerve as well as the vestibular nerve passes through the internal acoustic meatus located at the petrous part of the temporal bone. therefore, facial movement and balance perception is not affected. pt will not have increase risk for aspiration since the vagus and glossopharyngeal nerve passing through the jugular foramen is not affected. Abducent nerve is the only cranial nerve that will pass through the fenestrations of the cavernous sinuses. CN III,IV,V1,and V2 runs forward along the lateral wall of the sinus and not through the sinus

222

which of the following structure will strictly pass through the cavernous sinus along with the internal carotid artery? A. Abducent nerve B. Opthalmic division of trigeminal nerve C. Trochlear nerve D. Oculomotor nerve E. Maxillary nerve

223

A 32 y/o male suffered a skull base fracture secondary MVA causing transection of the (L) mandibular nerve. During recovery, which of the following will the patient unable to perform? A. Puff the left cheek B. Taste sensation of the left anterior 2/3rd of the tongue C. tense the ipsilateral palate D. raises the soft palate during swallowing E. none of the above

mandibular nerve provides motor to the muscles of mastication, general sensation to the ant 2/3 of the tongue and the lower jaw, and innervation to tensor veli palatini. Among the choices, patient will not be able to tense the ipsilateral soft palate. buccinator and taste sensation of the anterior tongue is supplied by the facial nerve. levator veli palatini which raises the soft palate is supplied by the pharyngeal plexus.

224

what is the characteristic feature of a 3rd lumbar vertebrae? A. Short slender transverse process B. Superior articular process face laterally C. Thick cylindrical pedicles D. Large round vertebral foramen E. Long flat quadrangular spinous process

typical lumbar vertebrae has the ff characteristics: large kidney shaped body, strong cylindical pedicles, thick lamina, triangular vertebral foramen, long slender transverse process, short, flat, quadrangular spinous process. Superior articular process faces medially, and inferior articular process faces laterally.

225

A 56 y/o male underwent a colonoscopy secondary to recent onset weightloss, change in the stool caliber, and hematochezia. During the procedure, a hemorrhagic polypoid mass noted on the anterior wall of the colon 14 cm from the anal verge. where is the exact location of the mass? A. anal canal B. rectum C. rectosigmoid D. sigmoid E. cannot be determine

at the distance of 1.5inches of 4cm the scope is already at the rectal ampulla. At the distance of 6.5in or 16cm, rectosigmoid junction is reach. Therefore, the mass is located at the rectum.

226

Which of the following is true about the liver A. Tha main suspensory structure of the liver is the hepatic veins draining to the inferior vena cava B. Periportal regions of the liver acinus is highly vulnerable to ischemic insult C. cantlie line separates the IV segment from V and VII segment D. the falciform ligament divides the liver into left and right functionally in terms of blood supply E. the caudate lobe belongs to the left inferior segment

227

aneurysmal dilatation and thrombosis of the proximal segment of the superior mesenteric artery will produce which of the following symptoms? A. gross hematochezia B. Jaundice secondary to necrosis and inflammation of the head of the pancreas C. Upper GI obstruction presenting as nonbilous emesis D. compression of the head of the pancreas E. severe continuous epigastric pain.

AUTHOR LEAN ANGELO SILVERIO, MD (TOP 4 - AUG 2014 MED BOARDS; TOPNOTCH MD), MD

TOPNOTCH EXAM MIDTERM EXAM 1 - FEB 2015

LEAN ANGELO SILVERIO, MD (TOP 4 - AUG 2014 MED BOARDS; TOPNOTCH MD), MD LEAN ANGELO SILVERIO, MD (TOP 4 - AUG 2014 MED BOARDS; TOPNOTCH MD), MD

MIDTERM EXAM 1 - FEB 2015

LEAN ANGELO SILVERIO, MD (TOP 4 - AUG 2014 MED BOARDS; TOPNOTCH MD), MD LEAN ANGELO SILVERIO, MD (TOP 4 - AUG 2014 MED BOARDS; TOPNOTCH MD), MD

MIDTERM EXAM 1 - FEB 2015

the centrilobular region of the liver acinus is the most vulnerable to ischemic insult while the periportal region is the most vulnerable to toxin induced injury. The line of Cantlie functionally divides the liver into right and left in terms of blood supply. falciform ligament only demarcates the liver anatomically. the caudate lobe is the 1st liver segment ( posterior segment).

LEAN ANGELO SILVERIO, MD (TOP 4 - AUG 2014 MED BOARDS; TOPNOTCH MD), MD

MIDTERM EXAM 1 - FEB 2015

superior mesenteric artery lies behind the neck of the pancreas. It traverses downward crossing the 3rd part of the duodenum anteriorly. It provides blood supply from the 2nd part of the duodenum up to proximal 2/3 of the transverse colon. Referred pain is periumbilical because it supplies the midgut derivative. patient will present with UGI obstruction however it would be a bilous type of emesis since the biliary drainage is proximal to the external obstruction. the head of the pancreas will least likely to necrosis since there extensive anastomosis b/w SMA and gastroduodenal artery.

LEAN ANGELO SILVERIO, MD (TOP 4 - AUG 2014 MED BOARDS; TOPNOTCH MD), MD

MIDTERM EXAM 1 - FEB 2015

TOPNOTCH MEDICAL BOARD PREP ANATOMY SUPEREXAM Page 29 of 94 For inquiries visit www.topnotchboardprep.com.ph or email us at [email protected]

MIDTERM EXAM 1 - FEB 2015

MIDTERM EXAM 1 - FEB 2015

TOPNOTCH MEDICAL BOARD PREP ANATOMY SUPEREXAM For inquiries visit www.topnotchboardprep.com.ph or email us at [email protected] Item # 228

QUESTION

EXPLANATION

AUTHOR LEAN ANGELO SILVERIO, MD (TOP 4 - AUG 2014 MED BOARDS; TOPNOTCH MD), MD LEAN ANGELO SILVERIO, MD (TOP 4 - AUG 2014 MED BOARDS; TOPNOTCH MD), MD LEAN ANGELO SILVERIO, MD (TOP 4 - AUG 2014 MED BOARDS; TOPNOTCH MD), MD LEAN ANGELO SILVERIO, MD (TOP 4 - AUG 2014 MED BOARDS; TOPNOTCH MD), MD

TOPNOTCH EXAM MIDTERM EXAM 1 - FEB 2015

which of the following is a derivative of the 5th pharyngeal pouch or the ultimobranchial body? A. thymus B. Palatine tonsils C. C cells D. Superior parathyroid gland E. None of the above

thymus gland is derived from 3rd pharyngeal pouch, same is true with the inferior parathyroid gland. The superior parathyroid is derived from 4th pharyngeal pouch. The C cells or parafollicular cells is the only derivative of the 5th pharyngeal pouch. Palatine tonsils is related to 2nd pharyngeal pouch.

229

which of the following is a functional remnant of the umbilical arteries? A. Median umbilical ligament B. Distal part of the inferior epigastric artery C. Superior vesical artery D. Distal part of the common iliac artery E. None of the above

at birth, the proximal portion of the umbilical arteries remain as the superior vesical artery. While the distal part will become vestigial remnant known as the medial umbilical ligament.

230

what layer of the epidermis contains eleidin? A. Stratum spinosum B. Stratum germinativum C. Stratum lucidum D. Stratum granulosum E. All of the above

stratum lucidum contains refractile droplets of eleidin in the cytoplasm which is responsible for its translucent appearance during microscopy.

231

which of the following is not true regarding regarding the histological difference between an artery and a vein ? A. The thickest coat of the artery is the tunica media while that of the vein is the tunica adventitia B. Vasa vasorum is more abundant in the arteries than in the veins C. the tunica intima of the arteries scalloped or corrugated during cross section. D. the internal elastic lamina is more prominent in the arteries than in the veins E. none of the above

the arterial vasa vasorum is lesser and extends only up to tunica media, while in the veins they are more prominent and even extends up to tunica intima. Expect hard questions in the board exam.

232

A 37 y/o male suffered a motor vehicular accident resulting to transection of the deep branch of the radial nerve secondary to comminuted fracture of the right radius. Which of the following movements will the patient still be able to perform? A. extends and abducts the hand at the wrist joint B. extend the distal phalanx of the thumb C. extends and adducts hand at the wrist joint D. extends fingers and hands E. none of the above

radial nerve innervates the ECRL prior to branching out to superfical and deep branch. The rest of the choices are supplied by the deep branch of the radial nerve.

LEAN ANGELO SILVERIO, MD (TOP 4 - AUG 2014 MED BOARDS; TOPNOTCH MD), MD

MIDTERM EXAM 1 - FEB 2015

233

a 37 y/o male had suppurative inflammation of the wrist joint secondary to previous open trauma. Wrist radiograph shows affectation of the carpal tunnel. Which of the following structures will not be affected? A. Flexor digitorum superficialis tendon. B. Flexor pollicis longus tendon. C. Flexor carpi radialis tendon D. Flexor digitorum profundus E. none of the above.

the components of the carpal tunnel are as follows: FPL, FDP, FPS, median nerve. The FCR tendon traverses the wrist joint through a separate compartment lateral to the carpal tunnel.

MIDTERM EXAM 1 - FEB 2015

234

A 47 y/o female is suffering from an suppurative ischial bursitis affecting the lesser sciatic foramen. Which of the following structures will be affected? A. Inferior gluteal nerve B. Sciatic nerve C. Posterior cutaneous nerve of the thigh D. tendon of the obturator internus E. none of the above

lesser sciatic foramen provides entrance into the perineum from the gluteal region. The follwing structures passes through this foramen: obturator internus tendon, pudendal nerve, internal pudendal artery and vein, obturator internus nerve.

LEAN ANGELO SILVERIO, MD (TOP 4 - AUG 2014 MED BOARDS; TOPNOTCH MD), MD LEAN ANGELO SILVERIO, MD (TOP 4 - AUG 2014 MED BOARDS; TOPNOTCH MD), MD

235

A 68 y/o female suffering from a chronic osteoarthritis of the ® knee underwent total knee arthroplasty. Which of the following muscle needs to be strengthen since it is the last to recover among the quadriceps group? A. Vastus lateralis B. popliteus C. Vastus medialis D. rectus femoris E. Vastus intermedius

Quadriceps muscle is composed of Vastus lateralis, intermedius, medialis and rectus femoris. During chronic knee disease, alteration of the knee joint mobility and Limitation of motion causes weakness of the quadriceps. Among the quadriceps, it I the vastus medialis that is first to atrophy and last to recovery.

LEAN ANGELO SILVERIO, MD (TOP 4 - AUG 2014 MED BOARDS; TOPNOTCH MD), MD

MIDTERM EXAM 1 - FEB 2015

TOPNOTCH MEDICAL BOARD PREP ANATOMY SUPEREXAM Page 30 of 94 For inquiries visit www.topnotchboardprep.com.ph or email us at [email protected]

MIDTERM EXAM 1 - FEB 2015

MIDTERM EXAM 1 - FEB 2015

MIDTERM EXAM 1 - FEB 2015

MIDTERM EXAM 1 - FEB 2015

TOPNOTCH MEDICAL BOARD PREP ANATOMY SUPEREXAM For inquiries visit www.topnotchboardprep.com.ph or email us at [email protected] Item # 236

QUESTION

EXPLANATION

AUTHOR

TOPNOTCH EXAM MIDTERM EXAM 1 - FEB 2015

A 21 y/o basketball player complains of pain and instability of the (L) knee. Upon your PE. It shows that it is positive for Apley and McMurray test. Which structure is primarily involved? A. ACL B. PCL C. Medial meniscus D. Plica syndrome E. Patella

McMurray is performed with the patient is in supine position and the test leg is in full flexion. The examiner laterally rotates the knee and then extend it. If there would be click or pain upon extension. It is highly suggestive of meniscal tear. Apley test is done in prone with the test leg in 90 knee flexion. examiner applies compresion to the knee joint while doing rotation. if there is pain it is suggestive of meniscal tear.

LEAN ANGELO SILVERIO, MD (TOP 4 - AUG 2014 MED BOARDS; TOPNOTCH MD), MD

237

A 21 y/o male suffered a monteggia fracture after being hit during a basketball match. Upon your PE, you noted that he cannot perform OK sign and there is extension of th index DIP and thumb IP joint. Which of the following is primarily affected. ? A. radial nerve B. Median nerve C. ulnar nerve D. anterior interosseous nerve E. none of the above

Monteggia fracture is a fracture of the ulna with concomitant dislocation of the proximal radioulnar joint. It is usually prone to anterior interosseous nerve injury. This nerve supplies FPL, lateral half FDP, pronator quadratus, thenar eminence and lateral two lumbricals.patient will not be able to flex the DIP of the index and IP of the thumb. there will be no sensory deficit.

LEAN ANGELO SILVERIO, MD (TOP 4 - AUG 2014 MED BOARDS; TOPNOTCH MD), MD

MIDTERM EXAM 1 - FEB 2015

238

Compression of the nerve that passess through the canal of Frohse will cause which of the following deficit? A. Weakness of wrist flexors B. Numbness and paresthesia along dorsal aspect of the lateral hand. C. Wrist drop D. Weakness of elbow extension E. none of the above

the posterior interosseous branch of the radial nerve passes through the canal of frohse formed by the two heads of the supinator muscle. It is a purely motor nerve supplying the extensor muscles of the forearm.

LEAN ANGELO SILVERIO, MD (TOP 4 - AUG 2014 MED BOARDS; TOPNOTCH MD), MD

MIDTERM EXAM 1 - FEB 2015

239

What structure is an upward extension of the posterior longitudinal ligament of the spine? A. Tectorial membrane B. Apical ligament C. Alar ligament D. Cruciate ligament E. None of the above

SIMILAR TO PREVIOUS BOARD EXAM CONCEPT/PRINCIPLE.

MIDTERM EXAM 1 - FEB 2015

240

retrolisthesis or backward extension of T8 vertebra will affect what level of spinal cord? A. T10 B. T11 C. T12 D. L1 E. None of the above

use this guide in determining which spinal cord is contiguous with a given vertebra. Cervical vertebra- add 1; upper thoracic- add 2, T7-T9 - add 3, tenth thoracic- L1 and L2, 11th thoracic-L3 and L4, 12th thoracic- L5 segment, 1st lumbar - sacral and coccygeal cord segment. therefore T8 vertebra will affect T11 spinal segment.

241

The thyroid isthmus is known to lie in which area? A. 2nd to 4th tracheal rings B. 3rd to 5th tracheal rings C. midway from the thyroid and cricoid cartilages at C4 D. C2 to C4 E. C5 to C6

SIMILAR TO PREVIOUS BOARD EXAM CONCEPT/PRINCIPLE. Isthmus is situated between the 2nd to 4th tracheal rings

242

A 56 year old male patient suffered a motor vehicular accident, he suffered multiple rib fractures and had ecchymoses along the right hemithorax, chest xray revealed blunting of the right costophrenic sulci. The surgeon plans to insert a chest tube to drain the fluid at which site? A. 2nd ICS right mid clavicular line upper border of the rib B. 7th ICS right posterior axillary line upper border of the rib C. 5th ICS right anterior axillary line upper border of the rib D. 5th ICS left midaxillary line lower border of the rib E. 7th ICS left midaxillary line lower border of the rib

SIMILAR TO PREVIOUS BOARD EXAM CONCEPT/PRINCIPLE, 2nd ICS Mid clavicular for emergency thoracentesis for pneumothorax, use upper border of the rib to avoid neurovascular bundle, be sure you're in the proper hemithorax

LEAN ANGELO SILVERIO, MD (TOP 4 - AUG 2014 MED BOARDS; TOPNOTCH MD), MD LEAN ANGELO SILVERIO, MD (TOP 4 - AUG 2014 MED BOARDS; TOPNOTCH MD), MD KEVIN BRYAN LO, MD (TOP 7 - AUG 2014 MED BOARDS; TOPNOTCH MD) KEVIN BRYAN LO, MD (TOP 7 - AUG 2014 MED BOARDS; TOPNOTCH MD)

243

The ovaries are lined by this specific epithelium: ___________, A. simple cuboidal B. simple squamous C. simple columnar D. stratified squamous E. pseudostratified columnar

SIMILAR TO PREVIOUS BOARD EXAM CONCEPT/PRINCIPLE it is simple cuboidal

KEVIN BRYAN LO, MD (TOP 7 - AUG 2014 MED BOARDS; TOPNOTCH MD)

MIDTERM 2 EXAM - FEB 2015

TOPNOTCH MEDICAL BOARD PREP ANATOMY SUPEREXAM Page 31 of 94 For inquiries visit www.topnotchboardprep.com.ph or email us at [email protected]

MIDTERM EXAM 1 - FEB 2015

MIDTERM 2 EXAM - FEB 2015

MIDTERM 2 EXAM - FEB 2015

TOPNOTCH MEDICAL BOARD PREP ANATOMY SUPEREXAM For inquiries visit www.topnotchboardprep.com.ph or email us at [email protected] Item # 244

QUESTION

EXPLANATION

AUTHOR

TOPNOTCH EXAM MIDTERM 2 EXAM - FEB 2015

The SA and AV node are the primary and secondary pacemakers of the heart respectively, from which of the following vessles does their predominant blood supply come from? A. left circumflex B. left anterior descending C. right coronary artery D. left coronary artery E. both B and C

SIMILAR TO PREVIOUS BOARD EXAM CONCEPT/PRINCIPLE, the right coronary artery supplies both SA and AV nodes

KEVIN BRYAN LO, MD (TOP 7 - AUG 2014 MED BOARDS; TOPNOTCH MD)

245

A patient came in with a pea sized lump non tender on the wrist, he was assessed to have a ganglion cyst, the surgeon knows that he must carefully dissect it out from this underlying tissue? A. extensor muscles B. superficial fasia C. tendon sheath D. synovial membrane E. deep fascia

SIMILAR TO PREVIOUS BOARD EXAM CONCEPT/PRINCIPLE, usually ganglion cysts arises from tendon sheaths

KEVIN BRYAN LO, MD (TOP 7 - AUG 2014 MED BOARDS; TOPNOTCH MD)

MIDTERM 2 EXAM - FEB 2015

246

Which of the following is considered as a direct branch from the aorta? A. right common carotid B. internal jugular C. right subclavian D. left subclavian E. left brachiocephalic

SIMILAR TO PREVIOUS BOARD EXAM CONCEPT/PRINCIPLE, direct branches from the aorta are right brachiocephalic, left common carotid, left subclavian,

KEVIN BRYAN LO, MD (TOP 7 - AUG 2014 MED BOARDS; TOPNOTCH MD)

MIDTERM 2 EXAM - FEB 2015

247

56 year old male chronic smoker had a history of weight loss, body malaise comes for consult due to multiple swellings or neck masses. Which set of lymph nodes should be palpated and biopsied for suspicion of metastasis of nasopharyngeal carcinoma A. upper jugular B. lower jugular C. posterior cervical D. submandibular E. middle jugular

SIMILAR TO PREVIOUS BOARD EXAM CONCEPT/PRINCIPLE, Nasopharyngeal primaries are the most common subtype to involve the posterior cervical lymph node chain

KEVIN BRYAN LO, MD (TOP 7 - AUG 2014 MED BOARDS; TOPNOTCH MD)

MIDTERM 2 EXAM - FEB 2015

248

A 50 year old female patient comes in for a mass near the nipple areolar complex of the right breast, which of the following lymph nodes would most likely be affected A. superficial pectoral B. deep pectoral C. internal mammary D. axillary E. all of the above

SIMILAR TO PREVIOUS BOARD EXAM CONCEPT/PRINCIPLE, the most common sentinel node involved is still the axillary lymph nodes

KEVIN BRYAN LO, MD (TOP 7 - AUG 2014 MED BOARDS; TOPNOTCH MD)

MIDTERM 2 EXAM - FEB 2015

249

A farmer swallowed some santol seeds, which area of the GI tract will these seeds most likely cause obstruction A. antral pyloric area B. ileocecal junction C. colorectal junction D. splenic flexure E. duodenojejunal junction

SIMILAR TO PREVIOUS BOARD EXAM CONCEPT/PRINCIPLE, narrowed area due to the ileocecal sphincter where the seeds may lodge cause obstruction

KEVIN BRYAN LO, MD (TOP 7 - AUG 2014 MED BOARDS; TOPNOTCH MD)

MIDTERM 2 EXAM - FEB 2015

250

Which of the following bones in the body undergoes endochondral ossification? A. mandible B. clavicle C. maxilla D. frontal bone E. parietal bone

SIMILAR TO PREVIOUS BOARD EXAM CONCEPT/PRINCIPLE, clavicle medial end is endochondral lateral end is intramembranous ossification Endochondral ossification - cartilage is initially present.

KEVIN BRYAN LO, MD (TOP 7 - AUG 2014 MED BOARDS; TOPNOTCH MD)

MIDTERM 2 EXAM - FEB 2015

251

Which is responsible for detecting linear (vertical) deceleration? A. utricle B. saccule C. anterior semicircular canals D. posterior semicircular canals E. lateral semicircular canals

SIMILAR TO PREVIOUS BOARD EXAM CONCEPT/PRINCIPLE. Slightly vague since both utricle and saccule are said to detect linear acceleration and deceleration. Utricle detects linear acceleration and deceleration. Saccule more on the vertical acceleration as in riding an elevator, semicircular canals deal with angular acceleration

KEVIN BRYAN LO, MD (TOP 7 - AUG 2014 MED BOARDS; TOPNOTCH MD)

MIDTERM 2 EXAM - FEB 2015

TOPNOTCH MEDICAL BOARD PREP ANATOMY SUPEREXAM Page 32 of 94 For inquiries visit www.topnotchboardprep.com.ph or email us at [email protected]

TOPNOTCH MEDICAL BOARD PREP ANATOMY SUPEREXAM For inquiries visit www.topnotchboardprep.com.ph or email us at [email protected] Item # 252

QUESTION

EXPLANATION

AUTHOR

TOPNOTCH EXAM MIDTERM 2 EXAM - FEB 2015

A patient presenting with adenocarcinoma of the lower 1/3 of the esophagus would most likely have metastasis to which of the following lymph nodes? A. bronchomediastinal B. celiac C. lower jugular D. diaphragmatic E. posterior cervical

SIMILAR TO PREVIOUS BOARD EXAM CONCEPT/PRINCIPLE. Celiac nodes drain the lower 1/3 of the esophagus

KEVIN BRYAN LO, MD (TOP 7 - AUG 2014 MED BOARDS; TOPNOTCH MD)

253

The most common anatomical location of the esophageal tear in boerhaave's syndrome A. Right distal esophagus B. Right middle 1/3 of the esophagus C. Left distal esophagus D. Left middle 1/3 of the esophagus E. A and B

SIMILAR TO PREVIOUS BOARD EXAM CONCEPT/PRINCIPLE, exact answer should be left posterolateral distal 1/3 of the esophagus

KEVIN BRYAN LO, MD (TOP 7 - AUG 2014 MED BOARDS; TOPNOTCH MD)

MIDTERM 2 EXAM - FEB 2015

254

A 26 year old male was rushed to the ER following a motor vehicular accident, patient was noted to have clear to serosanguinous fluid flowing from the nose positive for glucose and with (+) halo sign on gauze pad, which of the following bone has most likely been fractured? A. sphenoidal sinuses B. petrous portion of the temporal bone C. cribriform plate of the ethmoid D. frontal sinus of the frontal bone E. pterygoid plate

SIMILAR TO PREVIOUS BOARD EXAM CONCEPT/PRINCIPLE, fracture of cribriform plate of the ethmoid may lead to anosmia, and csf leak through the nose.

KEVIN BRYAN LO, MD (TOP 7 - AUG 2014 MED BOARDS; TOPNOTCH MD)

MIDTERM 2 EXAM - FEB 2015

255

a 25 year old male athlete was playing basketball when he experienced sudden inversion of his right foot after landing badly from jumping, which of the following ligaments are most likely damaged ? A. anterior tibiotalar ligament B. deltoid ligament C. talocalcaneal ligament D. anterior talofibular ligament E. calcaneal fibular ligament

SIMILAR TO PREVIOUS BOARD EXAM CONCEPT/PRINCIPLE, in inversion of the foot, most commonly damaged is the anterior talofibular ligament in the lateral side of the foot

KEVIN BRYAN LO, MD (TOP 7 - AUG 2014 MED BOARDS; TOPNOTCH MD)

MIDTERM 2 EXAM - FEB 2015

256

A 50 year old male hypertensive patient has been experiencing chest pains, syncopal attacks and exertional dyspnea. Physical exam revealed a systolic murmur, which of the following valvular pathologies are most likely present? A. mitral regurgitation B. mitral stenosis C. pulmonic stenosis D. aortic stenosis E. aortic regurgitation

SIMILAR TO PREVIOUS BOARD EXAM CONCEPT/PRINCIPLE, questions are indeed becoming more clinical, triad of syncope chest pain exertional dyspnea, systolic murmur is classic of aortic stenosis, murmur may radiate to the carotids

KEVIN BRYAN LO, MD (TOP 7 - AUG 2014 MED BOARDS; TOPNOTCH MD)

MIDTERM 2 EXAM - FEB 2015

257

A prostate examination via digital rectal examination was done for a 50 year old male patient, which of the following lobes of the prostate can be directly felt by this examination? A. middle lobe B. anterior lobe C. posterior lobe D. lateral lobes E. C and D

SIMILAR TO PREVIOUS BOARD EXAM CONCEPT/PRINCIPLE, DRE palpates the posterior lobe of the prostate area where most carcinomas arise

KEVIN BRYAN LO, MD (TOP 7 - AUG 2014 MED BOARDS; TOPNOTCH MD)

MIDTERM 2 EXAM - FEB 2015

258

The membranous urethra is lined by which lining epithelium? A. transitional B. pseudostratified columnar C. stratified squamous D. simple columnar E. both B and C

SIMILAR TO PREVIOUS BOARD EXAM CONCEPT/PRINCIPLE, what was asked was prostatic urethra lining transitional epithelium, membranous urethra is a small (1 or 2 cm) portion passing through the external urethral sphincter. This is the narrowest part of the urethra lined by pseudostratified columnar epithelium, spongy or penile urethra last part proximally is pseudostratified columnar and distally is stratified squamous

KEVIN BRYAN LO, MD (TOP 7 - AUG 2014 MED BOARDS; TOPNOTCH MD)

MIDTERM 2 EXAM - FEB 2015

TOPNOTCH MEDICAL BOARD PREP ANATOMY SUPEREXAM Page 33 of 94 For inquiries visit www.topnotchboardprep.com.ph or email us at [email protected]

TOPNOTCH MEDICAL BOARD PREP ANATOMY SUPEREXAM For inquiries visit www.topnotchboardprep.com.ph or email us at [email protected] Item # 259

QUESTION

EXPLANATION

AUTHOR

TOPNOTCH EXAM MIDTERM 2 EXAM - FEB 2015

A 30 year old male patient had a motorcycle accident, physical examination revealed absence of sensation at the lateral and posterior aspects of bilateral lower legs, this corresponds to what dermatome level? A. L2-L3 B. L4-L5 C. L5-S1 D. S1-S2 E. both A and B

SIMILAR TO PREVIOUS BOARD EXAM CONCEPT/PRINCIPLE, know the dermatome and myotome levels

KEVIN BRYAN LO, MD (TOP 7 - AUG 2014 MED BOARDS; TOPNOTCH MD)

260

On reading the CT scan of a patient revealed a mass at the level near the tracheal bifurcation, in determining the level of the lesion, the doctor is sure that at least part of the mass is at level of A. T2-T3 B. T3-T4 C. T4-T5 D. T5-T6 E. T6-T7

SIMILAR TO PREVIOUS BOARD EXAM CONCEPT/PRINCIPLE, choose the best answer, near the tracheal bifurcation is at the level of T4T5

KEVIN BRYAN LO, MD (TOP 7 - AUG 2014 MED BOARDS; TOPNOTCH MD)

MIDTERM 2 EXAM - FEB 2015

261

All of the following structures pass through the aortic hiatus EXCEPT: A. Thoracic duct B. Aorta C. Azygos Vein D. All of the above E. None of the above



RAYMUND MARTIN LI, MD (TOP 1 - AUG 2014 MED BOARDS; TOPNOTCH MD)

MIDTERM 3 EXAM - FEB 2015

262

A left-sided dominant blood supply of the heart is best characterized by: A. The posterior 1/3 of the interventricular septum is supplied by a branch from the left coronary artery B. The anterior interventricular artery is a branch of the right coronary artery C. The posterior interventricular artery comes from the left main coronary artery D. A and C E. A and B

Right sided dominant - comes from the RCA; leftsided dominant - comes from the LMCA

RAYMUND MARTIN LI, MD (TOP 1 - AUG 2014 MED BOARDS; TOPNOTCH MD)

MIDTERM 3 EXAM - FEB 2015

263

A 7 year old child is brought to you for consult by his mother. Upon examination, the child is noted to keep his left forearm pronated and partially flexed close to the body. Which is the most likely cause? A. Supracondylar humeral fracture B. Subluxation of the radial head C. Surgical humeral neck fracture D. Colles fracture E. None of the above

A case of nursemaid's elbow cause by subluxation of radial head

RAYMUND MARTIN LI, MD (TOP 1 - AUG 2014 MED BOARDS; TOPNOTCH MD)

MIDTERM 3 EXAM - FEB 2015

264

A patient comes to your clinic with complaints of hoarseness of voice. History revealed that the patient recently underwent thyroid surgery. Injury most probably occurred during: A. Ligation of the inferior thyroid artery B. Ligation of the superior thyroid artery C. Section of the ligament of Berry D. A or C E. B or C

Recurrent laryngeal nerve may be injured during ligation of the inferior thyroid artery or section of the ligament of Berry

RAYMUND MARTIN LI, MD (TOP 1 - AUG 2014 MED BOARDS; TOPNOTCH MD)

MIDTERM 3 EXAM - FEB 2015

265

Which of the following statements regarding abdominal structural relationships is correct? A. The portal vein is formed posterior to the 3rd portion of the duodenum B. The superior mesenteric artery passes anterior to the horizontal portion of the duodenum C. The major duodenal papilla is located in the first part of the duodenum D. All are correct E. None are correct

Duodenal papilla in the 2nd part of duodenum. Portal vein is formed behind the pancreas

RAYMUND MARTIN LI, MD (TOP 1 - AUG 2014 MED BOARDS; TOPNOTCH MD)

MIDTERM 3 EXAM - FEB 2015

TOPNOTCH MEDICAL BOARD PREP ANATOMY SUPEREXAM Page 34 of 94 For inquiries visit www.topnotchboardprep.com.ph or email us at [email protected]

TOPNOTCH MEDICAL BOARD PREP ANATOMY SUPEREXAM For inquiries visit www.topnotchboardprep.com.ph or email us at [email protected] Item # 266

QUESTION

EXPLANATION

AUTHOR

TOPNOTCH EXAM MIDTERM 3 EXAM - FEB 2015

Which of the following abdominal structures is NOT derived from the embryologic endoderm? A. Stomach B. Liver C. Spleen D. Pancreas E. All are derived from the endoderm

Speen is derived from mesoderm

RAYMUND MARTIN LI, MD (TOP 1 - AUG 2014 MED BOARDS; TOPNOTCH MD)

267

Which of the following structures forms the majority of the diaphragmatic surface of the heart? A. Rigth ventricle B. Left ventricle C. Right Atrium D. Pulmonary vessels E. Left atrium

Diaphragmatic surface formed by left ventricle. Diaphragmatic border by right ventricle

RAYMUND MARTIN LI, MD (TOP 1 - AUG 2014 MED BOARDS; TOPNOTCH MD)

MIDTERM 3 EXAM - FEB 2015

268

A 62 year old woman presents to your clinic complaining of right arm weakness. Further examination reveals inability to abduct the right arm past horizontal position and protrusion of the right scapula in an abnormal position noted when instructed to push against a wall. Which will most likely be a significant finding in the patient’s history? A. Right Rotator cuff injury B. Right midshaft humeral fracture C. Poorly fitted crutches D. Right modified radical mastectomy E. History of humeral neck fracture

Long thoracic nerve injury usually occurs during MRM

RAYMUND MARTIN LI, MD (TOP 1 - AUG 2014 MED BOARDS; TOPNOTCH MD)

MIDTERM 3 EXAM - FEB 2015

269

Which of the following structures is NOT transmitted through the superior orbital fissure? A. Oculomotor nerve B. Mandibular branch of the mandibular nerve C. Abducens nerve D. Ophthalmic branch of the mandibular nerve E. None of the above

CN III,IV,VI,V1 are transmitted through superior orbital fissure

RAYMUND MARTIN LI, MD (TOP 1 - AUG 2014 MED BOARDS; TOPNOTCH MD)

MIDTERM 3 EXAM - FEB 2015

270

A duodenal ulcer located in the lesser curvature of the stomach is most likely to cause which of the following complications: A. Bleeding from the gastroduodenal artery B. Bleeding from a branch of the celiac artery C. Bleeding from the left gastroepiploic artery D. Pancreatitis secondary to perforation E. None of the above

Bleeding from left gastric artery due to ulcer in lesser curvature.

RAYMUND MARTIN LI, MD (TOP 1 - AUG 2014 MED BOARDS; TOPNOTCH MD)

MIDTERM 3 EXAM - FEB 2015

271

A 57 year old man presents at the ER with complaints of lower back pain. Physical examination findings reveals weakened dorsiflexion of the foot and decreased sensation of the lateral surface of the leg and dorsum of the foot. Which of the following nerve root is most likely compressed? A. L2 B. L5 C. L4 D. S1 E. S2

Review dermatomal distribution

RAYMUND MARTIN LI, MD (TOP 1 - AUG 2014 MED BOARDS; TOPNOTCH MD)

MIDTERM 3 EXAM - FEB 2015

272

A 5 year old male patient is noted to have a soft nontender fullness of the left scrotum with associated positive transillumination. The underlying predisposing factor for this condition is similar to that of: A. Indirect inguinal hernia B. Direct Inguinal hernia C. Femoral hernia D. Varicocele E. All of the above

Due to patent processus vaginalis

RAYMUND MARTIN LI, MD (TOP 1 - AUG 2014 MED BOARDS; TOPNOTCH MD)

MIDTERM 3 EXAM - FEB 2015

273

A tracheostomy tube is best inserted in which of the following locations? A. Between 1st and 2nd tracheal rings B. Between 2nd and 3rd tracheal rings C. Between 3rd and 4th tracheal rings D. Between 4th and 5th tracheal rings E. Between the cricoid and the 1st tracheal ring

Between 2nd and 3rd tracheal rings is preferred. Above that there is danger of tracheal stenosis. Below that there is danger of hitting isthmus and vessels

RAYMUND MARTIN LI, MD (TOP 1 - AUG 2014 MED BOARDS; TOPNOTCH MD)

MIDTERM 3 EXAM - FEB 2015

TOPNOTCH MEDICAL BOARD PREP ANATOMY SUPEREXAM Page 35 of 94 For inquiries visit www.topnotchboardprep.com.ph or email us at [email protected]

TOPNOTCH MEDICAL BOARD PREP ANATOMY SUPEREXAM For inquiries visit www.topnotchboardprep.com.ph or email us at [email protected] Item # 274

QUESTION

EXPLANATION

AUTHOR

TOPNOTCH EXAM MIDTERM 3 EXAM - FEB 2015

Which of the following structures is derived from the embryologic common cardinal veins? A. superior vena cava B. pulmonary veins C. right atrium D. portal vein E. None of the above

Portal vein derived from vitelline vein. Smooth part of right atrium derived from sinus venosus.

RAYMUND MARTIN LI, MD (TOP 1 - AUG 2014 MED BOARDS; TOPNOTCH MD)

275

A sports player is noted to have an abnormal posterior location of the femur in relation to the tibia upon attempt at passive displacement. Which structure is injured? A. Anterior cruciate ligament B. Posterior cruciate ligament C. Medial collateral ligament D. Lateral collateral ligament E. Medial lemniscus

Anterior drawer sign - displacement of tibia anterior to femur or femur located posterior in relation to the tibia

RAYMUND MARTIN LI, MD (TOP 1 - AUG 2014 MED BOARDS; TOPNOTCH MD)

MIDTERM 3 EXAM - FEB 2015

276

When instructed to perform a pudendal nerve block, which of the following landmarks is most important? A. Tip of the ischial spine B. Ischeal tuberosity C. Lesser sciatic foramen D. Greater sciatic foramen E. Perineal body



RAYMUND MARTIN LI, MD (TOP 1 - AUG 2014 MED BOARDS; TOPNOTCH MD)

MIDTERM 3 EXAM - FEB 2015

277

Your resident asked you to insert a chest tube thoracostomy on a patient presenting with pleural effusion. Being a stellar intern, you insert the chest tube on the: A. 5th ICS MCL upper border of the rib B. 5th ICS MCL lower border of the rib C. 8th ICS AAL upper border of the rib D. 5th ICS AAL upper border of the rib E. 5th ICS PAL lower border of the rib



RAYMUND MARTIN LI, MD (TOP 1 - AUG 2014 MED BOARDS; TOPNOTCH MD)

MIDTERM 3 EXAM - FEB 2015

278

Failure of closure of the urethral folds will cause: A. Epispadias B. Phimosis C. Extrophy of the bladder D. Hypospadias E. None of the above

Hypospadias - failure of urethral fold closure. Epispadias - extrophy

MIDTERM 3 EXAM - FEB 2015

279

A 43 year old man was brought to the hospital due to a vehicular accident. Patient was noted to have loss of consciousness at the site of the accident with a lucid interval immediately after. Currently in the ER, the patient is being evaluated for deterioration of sensorium. Careful history reveals that the patient hit the part of his head near the anatomic region called the pterion. CT scan revealed a lentiform-shaped bleed. Which structure is most likely injured?? A. MCA B. Bridging veins C. Branch of the maxillary artery D. Superior sagittal sinus E. Branch of the internal carotid artery

Middle meningeal artery which is a branch of the maxillary artery usual source of epidural hematoma

RAYMUND MARTIN LI, MD (TOP 1 - AUG 2014 MED BOARDS; TOPNOTCH MD) RAYMUND MARTIN LI, MD (TOP 1 - AUG 2014 MED BOARDS; TOPNOTCH MD)

280

A 23 y.o. male patient complaining of left testicular mass underwent biopsy and was diagnosed to have testicular cancer. Which group of lymph nodes will primarily be affected once the cancer spreads? A. deep inguinal lymph nodes B. paraaortic lymph nodes C. superficial inguinal lymph nodes D. internal iliac lymph nodes E. external iliac lymph nodes

Testicular drained by paraaortic; prostate drained by internal iliac nodes; scrotum drained by inguinal nodes

RAYMUND MARTIN LI, MD (TOP 1 - AUG 2014 MED BOARDS; TOPNOTCH MD)

MIDTERM 3 EXAM - FEB 2015

281

Pain at the area of the anatomic "snuff box" following a fall on an outstretched hand may be caused by a fracture of which of the following? A. Scaphoid B. Capitate C. Hamate D. Pisiform E. None of the above

The floor of the anatomic snuff box is formed by the scaphoid and trapezium. Lateral bundary: tendons of EPB and AbdPL. Medial Boundary: tendon of EPL

ERIC ROYD TALAVERA, MD (TOP 1 - AUG 2014 MED BOARDS; TOPNOTCH MD)

FINAL EXAM - FEB 2015

TOPNOTCH MEDICAL BOARD PREP ANATOMY SUPEREXAM Page 36 of 94 For inquiries visit www.topnotchboardprep.com.ph or email us at [email protected]

MIDTERM 3 EXAM - FEB 2015

TOPNOTCH MEDICAL BOARD PREP ANATOMY SUPEREXAM For inquiries visit www.topnotchboardprep.com.ph or email us at [email protected] Item # 282

QUESTION

EXPLANATION

AUTHOR

TOPNOTCH EXAM FINAL EXAM - FEB 2015

Where is the fundus of the gallbladder located? A. At the level of the 10th costal cartilage along the linea semilunaris B. At the transpyloric plane along the linea alba C. At the level of the umbilicus D. At the level of the 9th costal cartilage along the linea semilunaris E. None of the above

The fundus of the gallbadder is located at the level of the 9th costal cartilage along the linea semilunaris

ERIC ROYD TALAVERA, MD (TOP 1 - AUG 2014 MED BOARDS; TOPNOTCH MD)

283

An individual with tertiary syphilis would have damage involving what structure in the spinal cord? A. Lateral Corticospinal Tract B. Dorsal Column C. Ventral Spinocerebellar Tract D. Anterior Spinothalamic Tract E. Lateral Spiothalamic Tract

Individuals with tertiary syphilis can present with tabes dorsalis which involves damage to the dorsal columns. Indiviuals present with loss of vibration, position sense and tactile discrimination.

ERIC ROYD TALAVERA, MD (TOP 1 - AUG 2014 MED BOARDS; TOPNOTCH MD)

FINAL EXAM - FEB 2015

284

Which of the following structures does not communicate with the middle meatus? A. Maxillary Sinus B. Posterior Ethmoidal Sinus C. Frontal Sinus D. Anterior Ethmoidal Sinus E. None of the above

The posterior ethmoidal sinus drain into the superior meatus. The rest of the choices drain into the middle meatus. The nasolacrimal duct drains into the inferior meatus

ERIC ROYD TALAVERA, MD (TOP 1 - AUG 2014 MED BOARDS; TOPNOTCH MD)

FINAL EXAM - FEB 2015

285

Umbrella or Dome cells are part of the lining epithelium of which organ ? A. Stomach B. Small intestines C. Uterus D. Liver E. Bladder

Umbrella cells are multinucleated superficial cell of the bladder’s transitional epithelium, which has vacuolated cytoplasm.

ERIC ROYD TALAVERA, MD (TOP 1 - AUG 2014 MED BOARDS; TOPNOTCH MD)

FINAL EXAM - FEB 2015

286

Which of the following statements is true regarding the abdominal aorta? A. Enters the abdominal cavity at the level of TV 10 B. It is located to the right of the inferior vena cava C. It terminates at the level of LV 4 D. Has four (4) unparied visceral branches E. Terminal branches are the common iliac and inferior phrenic artery

the abdominal aorta enters the abdominal cavity by piercing the aortic hiatus at the level of TV 12. It is located to the left of the IVC. It has three unpaired visceral branches (celiac, superior mesenteric, inferior mesenteric). It terminates at the level of LV4 into the common iliac and median sacral artery

ERIC ROYD TALAVERA, MD (TOP 1 - AUG 2014 MED BOARDS; TOPNOTCH MD)

FINAL EXAM - FEB 2015

287

A 78 year old male presents with hematuria, urgency, frequency and low back pain. Thorough evaulation eventually lead to a diagnosis of prostate cancer. Which of the following lobes of the prostate is most commonly involved? A. Anterior B. Middle C. Posterior D. Right Lateral E. Left Lateral

The middle lobe is commonly involved in BPH resulting in obstruction of the prostatic urethra. While the posterior lobe (part of the peripheral zone) is commonly involved in carcinomatous transformation

ERIC ROYD TALAVERA, MD (TOP 1 - AUG 2014 MED BOARDS; TOPNOTCH MD)

FINAL EXAM - FEB 2015

288

What muscle serves as guide in dividing the axillary artery into three different parts? A. Pectoralis Major B. Pectoralis Minor C. Serratus Anterior D. Teres Major E. Trapezius

The tendon of the pectoralis minor divides the axillary artery into 3 major parts

ERIC ROYD TALAVERA, MD (TOP 1 - AUG 2014 MED BOARDS; TOPNOTCH MD)

FINAL EXAM - FEB 2015

289

Which of the following statements regarding the pancreas is correct? A. It is an intraperitoneal organ B. It derives its blood supply from the inferior mesenteric artery C. It primarily located in the hypogastic and left hypochondriac region D. The portal vein is formed behind the tail of the pancreas E. The major pancreatic duct joins the common bile duct which drains into the second part of the duodenum

The pancreas is a retroperitoneal organ. Blood supply is derived from the celiax and superior mesenteric artery. It is located primarily in the epigastric region. The portal vein is formed behind the neck of the pancreas

ERIC ROYD TALAVERA, MD (TOP 1 - AUG 2014 MED BOARDS; TOPNOTCH MD)

FINAL EXAM - FEB 2015

TOPNOTCH MEDICAL BOARD PREP ANATOMY SUPEREXAM Page 37 of 94 For inquiries visit www.topnotchboardprep.com.ph or email us at [email protected]

TOPNOTCH MEDICAL BOARD PREP ANATOMY SUPEREXAM For inquiries visit www.topnotchboardprep.com.ph or email us at [email protected] Item # 290

QUESTION

EXPLANATION

AUTHOR

TOPNOTCH EXAM FINAL EXAM - FEB 2015

Which of the following structures is most likely to be compressed by an enlargement of the left atrium? A. Esophagus B. Thymus C. Thyroid gland D. Larynx E. Superior vena cava

The esophagus is related posterior to the left atrium, enlargement of the left atrium can cause compression of the esophagus producing symptoms such as dysphagia

ERIC ROYD TALAVERA, MD (TOP 1 - AUG 2014 MED BOARDS; TOPNOTCH MD)

291

What is the only cranial nerve that exits from the dorsal surface of the brainstem? A. Abducens (CN VI) B. Trigeminal (CN V) C. Hypoglossal (CN XII) D. Facial (CN VII) E. Trochlear (CN IV)

The trochlear nerve is the only cranial nerve of the brainstem that exits from the dorsal surface (particularly from the dorsal surface of the midbrain)

ERIC ROYD TALAVERA, MD (TOP 1 - AUG 2014 MED BOARDS; TOPNOTCH MD)

FINAL EXAM - FEB 2015

292

Brunner's glands are located in the submucosa of what segment of the gastrointestinal tract? A. Appendix B. Ileum C. Duodenum D. Stomach E. Jejunum

Brunner's glands (or duodenal glands) are compound tubular submucosal glands found in that portion of the duodenum which is above the hepatopancreatic sphincter (Sphincter of Oddi). The main function of these glands is to produce a mucus-rich alkaline secretion (containing bicarbonate)

ERIC ROYD TALAVERA, MD (TOP 1 - AUG 2014 MED BOARDS; TOPNOTCH MD)

FINAL EXAM - FEB 2015

293

The fibers of the corticospinal tract passes through what structure at the level of the midbrain? A. Cerebral aqueduct B. Superior Colliculus C. Red Nucleus D. Medial Longitudinal Fasciculus E. Cerebral peduncles

Path of the corticopsinal tract: Motor Cortex (Cerebrum) --> corona radiata --> posterior limb of internal capsule --> cerebral peduncle (midbrain)--> basis pontis (pons) --> pyramids (medulla) --> Decussate at level of lower medulla -> lateral corticopinal tract (spinal cord)

ERIC ROYD TALAVERA, MD (TOP 1 - AUG 2014 MED BOARDS; TOPNOTCH MD)

FINAL EXAM - FEB 2015

294

A 65 year old chronic smoker presented with right arm pain and drooping of the right upper eyelid. Diagnostic work up revealed a mass over the apex of the right lung. Which of the following muscles is affected to account for the finding of ptosis over the right eye? A. Lateral rectus B. Superior Oblique C. Levator palpebrae superioris D. Superior tarsal E. Medial rectus

This is a case of a Pancoast tumor. The said symptoms arise from compression of the adjacent structure, particularly the sympathetic chain ganglia (Horner's Syndrome). The superior tarsal muscle or the MULLER's muscle is innervated by sympathetic nerves. Compression of the said structures by the mass would produce weakness of the said muscle thus accounting for the ptosis. LPS is innervated by CN III and is not involved in Horner's syndrome

ERIC ROYD TALAVERA, MD (TOP 1 - AUG 2014 MED BOARDS; TOPNOTCH MD)

FINAL EXAM - FEB 2015

295

Which of the following structures is not located in the posterior mediastinum? A. Vagus nerve B. Esophagus C. Pericardium D. Thoracic duct E. Thoracic aorta

The heart and the pericardium are located in the middle mediastinum

ERIC ROYD TALAVERA, MD (TOP 1 - AUG 2014 MED BOARDS; TOPNOTCH MD)

FINAL EXAM - FEB 2015

296

A 76 year old male was brought in the ER due to chest pain. A 12L ECG was done which showed ST-segment elevation involving leads II, III and avF. Involvement of the said leads signify infarction of which area of the heart? A. Inferior wall B. Septal wall C. RV wall D. Lateral Wall E. Anterior Wall

SIMILAR TO PREVIOUS BOARD EXAM CONCEPT/PRINCIPLE: V1, V2: septal wall, I, avL: high lateral wall, V3-V4: anterior wall, V5-V6: lateral wall, V3r-V4r: RV wall

ERIC ROYD TALAVERA, MD (TOP 1 - AUG 2014 MED BOARDS; TOPNOTCH MD)

FINAL EXAM - FEB 2015

297

Which of the following statements concerning the larynx is INCORRECT? A. Sensory innervation is provided by the recurrent laryngeal and internal laryngeal nerve B. Extends from the tip of the epiglottis to the lower border of the cricoid cartilage C. The cricothyroid muscle is innervated by the recurrent laryngeal nerve D. The posterior cricoarytenoid muscle serves to abduct the vocal folds E. It can be divided into supraglottic, glottic and infraglottic areas

All intrinsic muscles of the larynx are innercated by the recurrent laryngeal nerve except for the cricothyroid which is innervated by the external branch of the superior laryngeal nerve

ERIC ROYD TALAVERA, MD (TOP 1 - AUG 2014 MED BOARDS; TOPNOTCH MD)

FINAL EXAM - FEB 2015

TOPNOTCH MEDICAL BOARD PREP ANATOMY SUPEREXAM Page 38 of 94 For inquiries visit www.topnotchboardprep.com.ph or email us at [email protected]

TOPNOTCH MEDICAL BOARD PREP ANATOMY SUPEREXAM For inquiries visit www.topnotchboardprep.com.ph or email us at [email protected] Item # 298

QUESTION

EXPLANATION

AUTHOR ERIC ROYD TALAVERA, MD (TOP 1 - AUG 2014 MED BOARDS; TOPNOTCH MD) ERIC ROYD TALAVERA, MD (TOP 1 - AUG 2014 MED BOARDS; TOPNOTCH MD)

TOPNOTCH EXAM FINAL EXAM - FEB 2015

Lead V1 is placed in what location/position in the chest? A. 5th ICS at the left midclavicular line B. 5th ICS at the left anterior axillary line C. 4th ICS at the left sternal border D. 4th ICS at the right sternal border E. 5th ICS at the right midclavicular line

SIMILAR TO PREVIOUS BOARD EXAM CONCEPT/PRINCIPLE: V1 4th ICS right sternal border, V2 4th ICS left sternal border, V3 halfway between V2 and V4, V4 5th ICS left MCL, V5 5th ICS at left AAL, V6 5th ICS at left MAL, V3R halfway between V1 and V4r, V4r 5th ICS at right MCL

299

Which of the following structures pass anterior to the medial malleolus? A. Tibial nerve B. Saphenous nerve C. Tendon of the flexor digitorum longus D. Tendon of the peroneus longus E. All of the above

Structures passing anterior to the medial malleolus: 1. Saphenous nerve 2. Great saphenous vein; Structures passing posterior to the medial malleolus: 1. Tendon of the FDL, 2. Tendon of the FHL, 3. Tibialis posterior, 4. Tibial nerve, 5. Posterior tibial artery

300

Hassall's corpuscles are histologic features of which of the following lymphoid organs? A. Spleen B. Lymph Nodes C. Tonsils D. Thymus E. All of the above

Hassall's corpuscles (or thymic corpuscles (bodies)) are structures found in the medulla of the human thymus, formed from eosinophilic type VI epithelial reticular cells arranged concentrically

ERIC ROYD TALAVERA, MD (TOP 1 - AUG 2014 MED BOARDS; TOPNOTCH MD)

FINAL EXAM - FEB 2015

301

A 57 y/o hypertensive, diabetic male was rushed to ER secondary to deterioriation of consciousness. Pt was successfully stabilized at the ER. Upon secondary PE, Pt is confused and oriented to place and person. his BP 170/100mmHg, PR 76 bpm, RR 14 cpm. You noted that his (L) eye is fixated in an inferolateral direction with dilated nonresponsive pupils. while his (R) arm showed tremors and incoordination when asked to reach for an object. no paralysis was noted on both UE/LE. CT scan was requested however results were pending. what is the most probable diagnosis? A. Weber syndrome B. Benedikt syndrome C. Claude syndrome D. Foville syndrome E. Wallenberg syndrome

LEAN ANGELO SILVERIO, MD (TOP 4 - AUG 2014 MED BOARDS; TOPNOTCH MD), MD

BACK-UP MIDTERM EXAM - FEB 2015

302

Which of the following neck muscles is not supplied by the ansa cervicalis ? A. geniohyoid B. Omohyoid C. sternohyoid D. Sternothyroid E. None of the above

ipsilateral paralysis of the oculomotor nerve accompanied by contralateral ataxia and intentional tremors is due to a vascular lesion at the central midbrain area known as Claude syndrome. It affects the oculomotor nucleus, red nucleus and the superior cerebellothalamic fibers. Weber syndrome is characterized by ipsilateral oculomotor paralysis and contralateral hemiplegia. Combination of Weber and Claude syndrome constitute the Benedikt syndrome. Foville syndrome is the vascular paramedian pons lesion represented by ipsilateral abducent nerve palsy and contralateral hemiparesis. Wallenberg syndrome is a lateral medullary syndrome secondary to occlusion of posterior inferior cerebellar artery. it manifest with contralateral hemianesthesia, ipsilateral loss of pain and thermal sensation in the face, nystagmus, vertigo, loss of taste from the ipsilateral side of th tonque and hoarseness with dysphagia. Haines 3rd ed pp 175-190 cervical plexus provides motor innervation to most of the neck muscles. Ansa cervicalis is the union of C1-C3 fibers give innervation to sternohyoid, sternothyroid, and omohyoid. C1 fibers via hypoglossal nerve supplies the thyrohyoid and geniohyoid. other cervical plexus branches include the phrenic nerve (C3-C5) innervating the diaphragm and segmental nerves supplying the prevertebral muscles including the levator scapulae. Snell 9th ed p 619.

LEAN ANGELO SILVERIO, MD (TOP 4 - AUG 2014 MED BOARDS; TOPNOTCH MD), MD

BACK-UP MIDTERM EXAM - FEB 2015

303

A 76 y/o male went to your clinic secondary to productive cough, weight loss and voice changes. He was smoker with a 50 pack years and an alcoholic beverage drinker. Xray was done showing a suspicious mass at the left upper lobe of the lung. a follow up CT scan showed a probable lung carcinoma with mediastinal Lymphadenopathies compressing the left bronchial airway. which of the following laryngeal muscles will be spared by this condition? A. thyroarytenoid B. cricothyroid C. posterior cricoarytenoid D. oblique arytenoid E. none of the above

this patient diagnosed with lung cancer on the left upper lobe accompanied by mediastinal lymph nodes can cause compression of the left recurrent laryngeal nerve as it hoops around the aortic arch and left bronchial area. this will result to paralysis of all the intrinsic laryngeal muscles except for cricothyroid. the latter is supplied by external laryngeal nerve. snell 9th ed p 648

LEAN ANGELO SILVERIO, MD (TOP 4 - AUG 2014 MED BOARDS; TOPNOTCH MD), MD

BACK-UP MIDTERM EXAM - FEB 2015

TOPNOTCH MEDICAL BOARD PREP ANATOMY SUPEREXAM Page 39 of 94 For inquiries visit www.topnotchboardprep.com.ph or email us at [email protected]

FINAL EXAM - FEB 2015

TOPNOTCH MEDICAL BOARD PREP ANATOMY SUPEREXAM For inquiries visit www.topnotchboardprep.com.ph or email us at [email protected] Item # 304

QUESTION

EXPLANATION

AUTHOR

TOPNOTCH EXAM BACK-UP MIDTERM EXAM - FEB 2015

A 28 y/o male with a history of poorly controlled asthma and Allergic Rhinitis presented to your clinic with a complain of decrease sense of smell and taste. Upon doing ENT examination, you noted a pale grayish polypoid mass located between inferior and middle nasal concha. which of the following set of sinuses will most likely obstructed? A. frontal, posterior ethmoid, maxillary sinuses B. frontal, middle ethmoid, sphenoid, maxillary sinuses C. frontal anterior, posterior and middle ethmoid, maxillary sinuses D. frontal, anterior ethmoid, maxillary sinuses E. frontal, ant and middle ethmoid, sphenoid, maxillary sinuses

middle meatus is located between inferior and middle nasal concha. It drains the frontal, anterior and middle ethmoid, maxillary sinuses. While sphenoid sinus drains into the sphenoethmoidal recess superior to superior nasal concha. Posterior ethmoidal sinus drains to the superior meatus above the middle concha. nasolacrimal duct drains to the inferior meatus. Snell 9th ed pp 643-644

LEAN ANGELO SILVERIO, MD (TOP 4 - AUG 2014 MED BOARDS; TOPNOTCH MD), MD

305

A 76 y/o female complaining of chronic vague abdominal pain was found to have aortic aneurysm extending from T11 -to L3 vertebra. What are the other structures will most likely be affected by this condition? A. thoracic duct B. vagus nerve C. inferior vena cava D. left gastric artery E. righr phrenic nerve

aortic aneurysm with such length will most likely affect structures that passes the aortic opening of the diaphragm ( T12 vertebrae). This include thoracic duct and azygous veins. Esophageal opening lies more anterior and it is at the level of T10. it transmits vagus nerve, left gastric vessels and lymphatics. Caval opening lies at the 8th thoracic vertebrae transmits IVC and right phrenic nerve. Snell 9th ed p 46.

LEAN ANGELO SILVERIO, MD (TOP 4 - AUG 2014 MED BOARDS; TOPNOTCH MD), MD

BACK-UP MIDTERM EXAM - FEB 2015

306

A 27 y/o male was sent to ER following an MVA. Pt was hypotensive, tachycardic and tachypneic along with neck veins distention and (L) tracheal deviation. There were no breath sounds noted on the left. What landmark will you use inorder to perform needle thoracostomy? A. (L) 2nd ICS 0.5 cm from sternal edge B. (L) 2nd ICS midclavicular line C. (R) 3rd ICS anterior axillary line D. (L) 5th ICS midaxillary line E. none of the above

the patient is suffering from left tension pneumothorax. A needle thoracostomy is needed inorder to alleviate the pressure in the left pleural area. by convention, it is best to insert a needle at the level of the 2nd ICS either at midclavicular or anterior axillary line. Snell 9th ed p 45.

LEAN ANGELO SILVERIO, MD (TOP 4 - AUG 2014 MED BOARDS; TOPNOTCH MD), MD

BACK-UP MIDTERM EXAM - FEB 2015

307

the ductus arteriosus is a derivative of what aortic arch? A. (L) 3rd aortic arch B. ® 4th aortic arch C. (L) 5th aortic arch D. (L) 6th aortic arch E. ® 2nd aortic arch

Aortic arches: 1st - maxillary artery, 2nd- hyoid and stapedial arteries, 3rd - common carotid artery and the 1st part of internal carotid arteries, 4th - (L) arch of aorta between LCCA and LScA, ® - proximal segment of subclavian artery. 5th- obliterated, no derivatives. 6th- (L) ductus arteriosus, (R) proximal segment of (R) pulmonary artery. Langmans embryology 9th ed 255-258.

BACK-UP MIDTERM EXAM - FEB 2015

308

Which of the following is not a tributary of the coronary sinus ? A. Great cardiac vein B. Middle cardiac vein C. Anterior cardiac vein D. Small cardiac vein E. Both B and C.

most blood from the heart wall drains into the right atrium through the coronary sinus, its tributaries are the great, small, and middle cardiac veins. The anterior cardiac vein drains blood directly to the right atrium bypassing the coronary sinus. snell 9th ed p 89

309

Which of the following is not a true statement regarding the inguinal canal? A. The superior wall is made up by the arching fibers of the Internal oblique and transversus abdominis B. The medial side of the posterior wall is reinforced by the conjoint tendon C. the deep inguinal ring is located at 1.3 cm medial to the ASIS D. the superficial inguinal ring gives attachment ot the external spermatic fascia. E. none of the above

the deep inguinal ring, a defect in the fascia transversalis, is located midway between ASIS and pubic tubercle. Snell 9th ed p 127.

LEAN ANGELO SILVERIO, MD (TOP 4 - AUG 2014 MED BOARDS; TOPNOTCH MD), MD LEAN ANGELO SILVERIO, MD (TOP 4 - AUG 2014 MED BOARDS; TOPNOTCH MD), MD LEAN ANGELO SILVERIO, MD (TOP 4 - AUG 2014 MED BOARDS; TOPNOTCH MD), MD

310

A 65 y/o male presenting weightloss, anemia and chronic diarrhea underwent a colonoscopy, a polypoid mass was located at the anterior wall of the colon 40 inches from the anal verge. What part of the colon where the mass is located? A. sigmoid B. descending colon C. transverse colon D. hepatic flexure E. ascending colon

the approximate length of each structures are as follows. Anal canal- 1.5 in, rectum -5 in, sigmoid, 10-15 inch, transverse 15 inch descending colon 10 in. therefore at 40 inches from the anal verge, the mass is most likely located at the transverse colon.

LEAN ANGELO SILVERIO, MD (TOP 4 - AUG 2014 MED BOARDS; TOPNOTCH MD), MD

BACK-UP MIDTERM EXAM - FEB 2015

TOPNOTCH MEDICAL BOARD PREP ANATOMY SUPEREXAM Page 40 of 94 For inquiries visit www.topnotchboardprep.com.ph or email us at [email protected]

BACK-UP MIDTERM EXAM - FEB 2015

BACK-UP MIDTERM EXAM - FEB 2015

TOPNOTCH MEDICAL BOARD PREP ANATOMY SUPEREXAM For inquiries visit www.topnotchboardprep.com.ph or email us at [email protected] Item # 311

QUESTION

EXPLANATION

what is the most dilatable portion of the male urethra? A. Prostatic urethra B. Membranous urethra C. Penile urethra D. Navicular fossa of the urethra E. None of the above

the prostatic urethra is the widest and the most dilatable portion of the entire male urethra. Snell 9th ed p 278

312

the physiological umbilical herniation during embryonic development occur in what age of gestation? A. 5th week B. 6th week C. 7th week D. 8th week E. 10th week

313

Which of the following is a branch of the anterior division of the internal iliac artery? A. Lateral sacral artery B. Superior gluteal artery C. Inferior gluteal artery D. Iliolumbar artery E. None of the above

( SIMILAR TO PREVIOUS BOARD EXAM CONCEPT/PRINCIPLE). Development of the primary intestinal loop is characterized by rapid elongation particularly of the cephalic limb. As a result of the rapid growth and expansion of the liver, the abdominal cavity temporarily becomes too small to contain all the intestinal lipps and they enter the extraembryonic cavity in the umblical cord during 6th week of development. Langmans 9th ed p 307. upon descent of the internal iliac artery at the level of the greater sciatic foramen, it divides into anterior and posterior division. The branches of the anterior division are the umbilical artery, obturator artery, inferior vesical artery, middle rectal arterym internal pudendal aftery, inferior gluteal artery, uterine and vaginal arteries. while the posterior division has only three branches: iliolumbar, lateral sacral and superior gluteal artery. Snell 9th ed Regions p 256-257

314

A 27 y/o female was brought to ER secondary to saddle type injury due to MVA, pelvic CT scan was requested showing hematoma on the superficial perineal pouch. Which of the following structures will most likely be spared in this condition? A. sphincter urethrae B. ischiocavernosus muscle C. bulbospongiosus muscle D. perineal branch of the pudendal nerve E. perineal body

315

316

AUTHOR LEAN ANGELO SILVERIO, MD (TOP 4 - AUG 2014 MED BOARDS; TOPNOTCH MD), MD LEAN ANGELO SILVERIO, MD (TOP 4 - AUG 2014 MED BOARDS; TOPNOTCH MD), MD

TOPNOTCH EXAM BACK-UP MIDTERM EXAM - FEB 2015

BACK-UP MIDTERM EXAM - FEB 2015

LEAN ANGELO SILVERIO, MD (TOP 4 - AUG 2014 MED BOARDS; TOPNOTCH MD), MD

BACK-UP MIDTERM EXAM - FEB 2015

the contents of the superficial perineal pouch are the following: root of the clitoris, bulbospongiosus, ischiocavernosus, superficial transverse perineal muscle, perioneal body and the perineal branch of the pudendal nerve. The contents of the deep perineal pouch includes the followin: sphincter urethrae, deep transverse perineal muscle, internal pudendal vessels, and dorsal nerve of the clitoris. Snell 9th ed regions pp 322- 323

LEAN ANGELO SILVERIO, MD (TOP 4 - AUG 2014 MED BOARDS; TOPNOTCH MD), MD

BACK-UP MIDTERM EXAM - FEB 2015

A 34 y/o female was brought to ER secondary to a diffuse abdominal pain. she was hypotensive, tachycardic at presentation. PE showed diffuse rigidity of the abdomen with both rebound and direct tenderness prominent on the epigastric area. CT scan was done showing blood collection in the lesser sac. she was a smoker and alcoholic beverage drinker. she always skip her meals because she want to loose weight and that she always take mefenamic acid almost everyday for her migraine.prior to onset, she claims to have recurrent history of vague epigastric pain whenever she skip meal and that there were occasions of nocturnal awakening secondary to epigastric pain. what specific vascular structure will most likely be affected in this setting? A. superior pancreaticoduodenal artery B. portal vein C. gastroduodenal artery D. (L) gastroepiploic artery E. all of the above

based on the history, the patient is most likely to have a perforated duodenal ulcer. The most common site for duodenal ulcer is its 1st part which is intraperitoneal structure. CT scan finding of blood in the lesser sac indicates that the perforation is most likely at the posterior wall. gastroduodenal artery is immediately behind the 1st part of the duodenum. affectation of this structure can lead to bleeding in the lesser sac, and sometimes pancreatitis. Snell 9th ed p 176

LEAN ANGELO SILVERIO, MD (TOP 4 - AUG 2014 MED BOARDS; TOPNOTCH MD), MD

BACK-UP MIDTERM EXAM - FEB 2015

which of the following is true about the schneiderian membrane of the nasal cavity? A. Abundant venous plexus in the tunica propria B. Tall pseudostratified columnar epithelium without goblet cells C. No distinct basal lamina D. contains bundles of olfactor y nerves in the lamina propria E. all of the above

the schneiderian membrane refers to the respiratory portion of the nasal cavity. It is characterized by pseudostratified columnar epithelium with goblet cells lining the mucous membrane. It has a relatively thick basal lamina; contains abundant venous plexuses, and serous/ mucous tubuloacinar glands in the tunica propria. it also has characteristic lymphocytic infiltration in the epithelial layer. other choices refers to the olfactory neuroepithelium. cabral histology p 178

LEAN ANGELO SILVERIO, MD (TOP 4 - AUG 2014 MED BOARDS; TOPNOTCH MD), MD

BACK-UP MIDTERM EXAM - FEB 2015

TOPNOTCH MEDICAL BOARD PREP ANATOMY SUPEREXAM Page 41 of 94 For inquiries visit www.topnotchboardprep.com.ph or email us at [email protected]

TOPNOTCH MEDICAL BOARD PREP ANATOMY SUPEREXAM For inquiries visit www.topnotchboardprep.com.ph or email us at [email protected] Item # 317

QUESTION complete transection of the lateral cord of brachial plexus will result to which of the following symptoms? A. Decrease sensation in the lateral dorsal side of the the hand. B. Decrease sensation in the lateral side of the forearm C. inability to supinate the forearm D. inability to flex the forearm E. all of the above.

EXPLANATION

AUTHOR

TOPNOTCH EXAM BACK-UP MIDTERM EXAM - FEB 2015

the lateral cord give rise to lateral pectoral nerve ( pectoralis major), musculocutaneous nerve ( coracobrachialis, biceps brachii, brachialis, sensory to the lateral border of the foreeam) and lateral root of the median nerve. A- is supplied by the radial nerve which is a branch of the posterior cord. C and D- pt will only have weakness but not inability since the supinator muscle is supplied by the radial nerve eventhough the biceps brachii is denervated. on the otherhand, there is still some flexion of the forearm because of the reverse action of other forearm muscle as well as the partial innervation of brachialis by the radial nerve. ( Snell 9th ed Regions p 354) the deep branch of radial nerve is a pure motor nerve supplying all the muscles of the posterior compartment of the forearm except for the ECRL and supinator muscle. There would be no wrist drop since the ECRL is powerful enough to promote wrist extension. Snell 9th ed p 431.

LEAN ANGELO SILVERIO, MD (TOP 4 - AUG 2014 MED BOARDS; TOPNOTCH MD), MD

LEAN ANGELO SILVERIO, MD (TOP 4 - AUG 2014 MED BOARDS; TOPNOTCH MD), MD

BACK-UP MIDTERM EXAM - FEB 2015

LEAN ANGELO SILVERIO, MD (TOP 4 - AUG 2014 MED BOARDS; TOPNOTCH MD), MD LEAN ANGELO SILVERIO, MD (TOP 4 - AUG 2014 MED BOARDS; TOPNOTCH MD), MD DEBBIE ROSE TANENGSY, MD (TOP 5 - AUG 2014 MED BOARDS; TOPNOTCH MD) DEBBIE ROSE TANENGSY, MD (TOP 5 - AUG 2014 MED BOARDS; TOPNOTCH MD) DEBBIE ROSE TANENGSY, MD (TOP 5 - AUG 2014 MED BOARDS; TOPNOTCH MD) DEBBIE ROSE TANENGSY, MD (TOP 5 - AUG 2014 MED BOARDS; TOPNOTCH MD)

BACK-UP MIDTERM EXAM - FEB 2015

318

A 72 y/o male was brought to ER secondary to FOOSH injury ( fall on a outstretched hand). X ray was done showing displaced compression fracture of the radial head. Physical examination of the arm reveals findings that are consistent with an injury to the deep branch of radial nerve. which of the following statement is correct. ? A. MMT 3/5 in forearm supination B. decrease sensation on the dorsal lateral side of the hand C. presence of wristdrop D. all of the above E. none of the above

319

Which of the following structures is not supplied by the lateral plantar nerve? A. Adductor hallucis B. Flexor digitorum brevis C. Quadratus plantae D. Flexor digiti minimi E. None of the above

the lateral plantar nerve supplies all of the intrinsic muscles of the foot except for abductor hallucis, flexor digitorum brevis, flexor hallucis brevis, and the 1st lumbrical muscle ( all of which are supplied by the medial plantar nerve) Snell p 676.

320

Retrolisthesis of the 8th thoracic vertebra will affect what corresponding spinal segment? A. T8 B. T9 C. T10 D. T11 E. T12

relationship of spinal cord segements to vertebral numbers: cervical vertebrae: add 1, upper thoracic vertebra (T1-T6): add 2, lower thoracic (T7-T9): add 3, tenth thoracic: L1 and L2 cord segment, 11th thoracic: L3-L4 cord segment, 12 th thoracic: L5 cord segment; 1st lumbar: sacral and coccygeal segments. Snell 9th ed p 704

321

Epithelial lining of the ovaries: A. simple squamous B. stratified squamous C. simple cuboidal D. simple columnar E. pseudostratified columnar

SIMILAR TO PREVIOUS BOARD EXAM CONCEPT/PRINCIPLE.

322

Tongue papillae with no taste buds and thus serve a mechanical function only: A. fungiform B. circumvallate C. foliate D. filliform

Fungiform - mushroom shaped; circumvallate - largest, located near the sulcus terminalis; foliate - rudimentary in humans; filliform - most abundant papillae

323

Epithelial lining of lymphangiomas: A. simple squamous B. stratified squamous C. simple cuboidal D. simple columnar E. psuedostratified columnar

SIMILAR TO PREVIOUS BOARD EXAM CONCEPT/PRINCIPLE.

324

True of the jejunum but not of the ileum, except: A. makes up the initial 3/5 of the small intestine B. has longer vasa recta C. has 1-2 arterial arcades D. has a thicker wall E. with prominent Peyer's patches



TOPNOTCH MEDICAL BOARD PREP ANATOMY SUPEREXAM Page 42 of 94 For inquiries visit www.topnotchboardprep.com.ph or email us at [email protected]

BACK-UP MIDTERM EXAM - FEB 2015

BACK-UP MIDTERM EXAM - FEB 2015

BACK-UP MIDTERM EXAM - FEB 2015

BACK-UP MIDTERM EXAM - FEB 2015

BACK-UP MIDTERM EXAM - FEB 2015

TOPNOTCH MEDICAL BOARD PREP ANATOMY SUPEREXAM For inquiries visit www.topnotchboardprep.com.ph or email us at [email protected] Item # 325

QUESTION

EXPLANATION

The lower anal canal, as opposed to the upper anal canal: A. is supplied by a branch of the inferior mesenteric artery B. is drained by the deep inguinal nodes C. is derived from the ectoderm D. is lined by simple columnar epithelium



326

The moderator band is located in which chamber of the heart? A. right atrium B. left atrium C. right ventricle D. left ventricle

The moderator band is located in the right ventricle.

327

True of the right atrium: A. smaller than the left atrium B. has thicker walls C. has a moderator band D. with a sinus venarum, a coarse muscular portion of the chamber

The right atrium is larger than the left, has thicker walls, and has a sinus venarum, which is a smooth portion developed from the embryonic sinus venosus. The moderator band is located in the right ventricle.

328

GIST is located in what layer of the gastrointestinal tract? A. mucosa B. submucosa C. muscularis D. adventitia E. serosa



329

The accessory nerve crosses this triangle of the neck: A. submental B. submandibular C. carotid D. muscular E. occipital



330

the lower ureter is supplied by the: A. renal artery B. superior vesical artery C. gonadal artery D. middle rectal E. inferior vesical

Upper ureter - renal artery; middle ureter - gonadal artery; lower ureter - superior vesical artery

331

Found in the superficial perineal space, except: A. bulbospongiosus B. ischiocavernosus C. spongy urethra D. perineal body E. membranous urethra

The membranous urethra is found within the deep perineal space.

332

A patient with an upper motor neuron lesion is expected to exhibit: A. flaccid paralysis B. atrophy C. clonus D. fasciculations E. reduced muscle tone

A person with an upper motor neuron lesion presents with spastic paralysis, increased muscle tone, clonus, (+) Babinski. There is no atrophy or fasciculation in UMNL.

333

Leaf shaped laryngeal cartilage: A. thyroid B. cricoid C. epiglottis D. arytenoid E. corniculate

thyroid - largest laryneal cartilage; cricoid - signet ring shaped; arytenoid - pyramidal; corniculate - conical; cuneiform - club shaped

AUTHOR DEBBIE ROSE TANENGSY, MD (TOP 5 - AUG 2014 MED BOARDS; TOPNOTCH MD) DEBBIE ROSE TANENGSY, MD (TOP 5 - AUG 2014 MED BOARDS; TOPNOTCH MD) DEBBIE ROSE TANENGSY, MD (TOP 5 - AUG 2014 MED BOARDS; TOPNOTCH MD) DEBBIE ROSE TANENGSY, MD (TOP 5 - AUG 2014 MED BOARDS; TOPNOTCH MD) DEBBIE ROSE TANENGSY, MD (TOP 5 - AUG 2014 MED BOARDS; TOPNOTCH MD) DEBBIE ROSE TANENGSY, MD (TOP 5 - AUG 2014 MED BOARDS; TOPNOTCH MD) DEBBIE ROSE TANENGSY, MD (TOP 5 - AUG 2014 MED BOARDS; TOPNOTCH MD) DEBBIE ROSE TANENGSY, MD (TOP 5 - AUG 2014 MED BOARDS; TOPNOTCH MD) DEBBIE ROSE TANENGSY, MD (TOP 5 - AUG 2014 MED BOARDS; TOPNOTCH MD)

TOPNOTCH MEDICAL BOARD PREP ANATOMY SUPEREXAM Page 43 of 94 For inquiries visit www.topnotchboardprep.com.ph or email us at [email protected]

TOPNOTCH EXAM BACK-UP MIDTERM EXAM - FEB 2015

BACK-UP MIDTERM EXAM - FEB 2015

BACK-UP MIDTERM EXAM - FEB 2015

BACK-UP MIDTERM EXAM - FEB 2015

BACK-UP MIDTERM EXAM - FEB 2015

BACK-UP MIDTERM EXAM - FEB 2015

BACK-UP MIDTERM EXAM - FEB 2015

BACK-UP MIDTERM EXAM - FEB 2015

BACK-UP MIDTERM EXAM - FEB 2015

TOPNOTCH MEDICAL BOARD PREP ANATOMY SUPEREXAM For inquiries visit www.topnotchboardprep.com.ph or email us at [email protected] Item # 334

QUESTION

EXPLANATION

Example of a typical rib: A. rib 12 B. rib 11 C. rib 10 D. rib 9

Ribs 12 and 11 have a single facet of the heads, are short, with no neck or tubercle. Rib 10 articulates with the T10 vertebra only. Other atypical ribs: rib 1 - broadest, most curved, prominent scalene tubercle; rib 2 - thinner, less curved, has tuberosity for serratus anterior.

335

The triangle of Calot: A. contains the cystic duct B. is bounded by the liver laterally C. is bounded by the cystic artery inferiorly D. is bounded by the common hepatic duct medially

The triangle of Calot contains the cystic artery, is bounded by the following: superiorly by the liver, inferiorly by the cystic duct, and medially by the common hepatic duct.

336

The female pelvis, compared to the male pelvis, has a/n: A. narrower sacrum B. longer symphysis pubis C. oval shaped superior aperture D. inverted ischial tuberosity E. deeper iliac fossa

Female pelvis: borader sacrum, shorter symphysis pubis, everted ischial tuberosity, shallower iliac fossa.

337

Which part of the adrenals contain pale staining cells in cords with foamy appearances due to lipid droplet accumulations? A. zona glomerulosa B. zona fasciculata C. zona reticularis D. adrenal medulla

Zona glomerulosa - basophilic arranged in cords of columnar or pyramidal cells. Zona reticularis - eosinophilic cells with large nucleus arranged in cords smaller and less spongy than the cells in zona fasciculata.

338

Fibrocartilage is found in: A. larynx B. epiglottis C. auditory tube D. costal cartilage E. intervertebral discs



339

Layer absent in thin skin: A. corneum B. lucidum C. granulosum D. spinosum E. basale



340

Thickest layer in veins: A. tunia intima B. tunica media C. tunica adventitia D. none of the above



341

The carotid sinus is found in the: A. Anterior surface of the bifurcation of the carotid artery B. Posterior surface of the bifurcation of the common carotid artery C. Beginning of the internal carotid artery D. Beginning of the external carotid artery

342

The muscles that form the superior boundary of the carotid triangle: A. Posterior belly of digastric B. Superior belly of omohyoid C. Sternocleidomastoid D. Trapezius

The carotid sinus is a localized dilation at the beginning of the internal carotid artery. The carotid body is a small structure that lies posterior to the bifurcation of the common carotid artery. Reference: Snell. Clinical Anatomy by Regions, 8th ed. p. 747 The boundaries of the carotid triangle are as follows: -Superiorly by the posterior belly of digastric -Posteriorly by the SCM -Anteriorly by the superior belly of the omohyoid.

343

This nerve accompanies the superior thyroid artery: A. Superior laryngeal B. External laryngeal C. Internal laryngeal D. Recurrent laryngeal

The superior thyroid artery, a branch of the ECA, is accompanied by the external laryngeal nerve which supplies the cricothyroid muscle. Reference: Snell. Clinical Anatomy by Regions, 8th ed. P. 749

AUTHOR DEBBIE ROSE TANENGSY, MD (TOP 5 - AUG 2014 MED BOARDS; TOPNOTCH MD) DEBBIE ROSE TANENGSY, MD (TOP 5 - AUG 2014 MED BOARDS; TOPNOTCH MD) DEBBIE ROSE TANENGSY, MD (TOP 5 - AUG 2014 MED BOARDS; TOPNOTCH MD) DEBBIE ROSE TANENGSY, MD (TOP 5 - AUG 2014 MED BOARDS; TOPNOTCH MD) DEBBIE ROSE TANENGSY, MD (TOP 5 - AUG 2014 MED BOARDS; TOPNOTCH MD) DEBBIE ROSE TANENGSY, MD (TOP 5 - AUG 2014 MED BOARDS; TOPNOTCH MD) DEBBIE ROSE TANENGSY, MD (TOP 5 - AUG 2014 MED BOARDS; TOPNOTCH MD) JESSICA MAE SANCHEZ, MD (TOP 4 - AUG 2014 MED BOARDS; TOPNOTCH MD) JESSICA MAE SANCHEZ, MD (TOP 4 - AUG 2014 MED BOARDS; TOPNOTCH MD) JESSICA MAE SANCHEZ, MD (TOP 4 - AUG 2014 MED BOARDS; TOPNOTCH

TOPNOTCH MEDICAL BOARD PREP ANATOMY SUPEREXAM Page 44 of 94 For inquiries visit www.topnotchboardprep.com.ph or email us at [email protected]

TOPNOTCH EXAM BACK-UP MIDTERM EXAM - FEB 2015

BACK-UP MIDTERM EXAM - FEB 2015

BACK-UP MIDTERM EXAM - FEB 2015

BACK-UP MIDTERM EXAM - FEB 2015

BACK-UP MIDTERM EXAM - FEB 2015

BACK-UP MIDTERM EXAM - FEB 2015

BACK-UP MIDTERM EXAM - FEB 2015

BACK-UP MIDTERM EXAM - FEB 2015

BACK-UP MIDTERM EXAM - FEB 2015

BACK-UP MIDTERM EXAM - FEB 2015

TOPNOTCH MEDICAL BOARD PREP ANATOMY SUPEREXAM For inquiries visit www.topnotchboardprep.com.ph or email us at [email protected] Item #

QUESTION

EXPLANATION

AUTHOR

TOPNOTCH EXAM

MD)

344

True of meiosis, except: A. Creates new set of chromosomes by random distribution to daughter cells B. Bestow each germ cell with the diploid number of chromosomes C. Provides gamete variabilities D. Produce daughter cells genetically different from the parent and from each other Which of the following cartilages is characterized by the absence of a perichondrium? A. Adult hyaline cartilage B. Fetal hyaline cartilage C. Fibrocartilage D. Articular cartilage

Meiosis reduces the chromosomes to the haploid number.

346

This structure divides the subclavian artery into 3 parts: A. First rib B. Pectoralis minor C. Scalene anterior D. Teres major

Reference: Snell. Clinical Anatomy by Regions, 8th ed. P.751

347

The deep cervical lymph nodes lie along the: A. Sternocleidomastoid B. Common carotid artery C. External jugular vein D. Internal jugular vein

348

Tracheostomy tube insertion is usually done at the level of the: A. Cricothyroid membrane B. 1st-2nd tracheal rings C. 2nd-3rd tracheal rings D. 4th-5th tracheal rings

The deep cervical nodes form a vertical chain along the course of the internal jugular vein within the carotid sheath. The superficial cervical nodes lie along the course of the external jugular vein on the side of the neck. Reference: Snell. Clinical Anatomy by Regions, 8th ed. p. 755-756 The preferred site for tracheostomy is through the second to third ring, with the thyroid isthmus retracted inferiorly. Reference: Snell. Clinical Anatomy by Regions, 8th ed. P. 814

349

After sustaining a fracture of the mid-humerus following a fall, the patient was unable to extend the wrist. This is consistent with injury to the: A. Axillary nerve B. Median nerve C. Radial nerve D. Ulnar nerve

Reference: Topnotch Anatomy Handouts

350

Which of the following is not a characteristic of epithelium in general? A. Avascular B. Derived from the ectoderm only C. Capable of metaplasia D. Rests on a basal lamina

Epithelium may be derived from any of the 3 germ layers.

351

What vein accompanies the anterior interventricular artery? A. Anterior cardiac B. Great cardiac C. Middle cardiac D. Small cardiac

Great cardiac vein - anterior interventricular artery Middle cardiac vein - posterior interventricular artery Small cardiac vein - marginal branch of the RCA

352

The 6th aortic arch will become the: A. Common carotid artery B. Maxillary artery C. Pulmonary artery D. Right subclavian artery

Aortic arches: I - Maxillary artery II - Hyoid and stapedial arteries III - Common carotid, first part of the ICA, ECA IV - Left: arch of the aorta; Right: proximal segment of the right subclavian artery V - regresses VI - proximal part of the pulmonary artery, and the ductus arteriosus on the left Reference: Langman's Medical Embryology, 9th ed. p. 255-258

345

Reference: Topnotch Histology Handouts

JESSICA MAE SANCHEZ, MD (TOP 4 - AUG 2014 MED BOARDS; TOPNOTCH MD) JESSICA MAE SANCHEZ, MD (TOP 4 - AUG 2014 MED BOARDS; TOPNOTCH MD) JESSICA MAE SANCHEZ, MD (TOP 4 - AUG 2014 MED BOARDS; TOPNOTCH MD) JESSICA MAE SANCHEZ, MD (TOP 4 - AUG 2014 MED BOARDS; TOPNOTCH MD) JESSICA MAE SANCHEZ, MD (TOP 4 - AUG 2014 MED BOARDS; TOPNOTCH MD) JESSICA MAE SANCHEZ, MD (TOP 4 - AUG 2014 MED BOARDS; TOPNOTCH MD) JESSICA MAE SANCHEZ, MD (TOP 4 - AUG 2014 MED BOARDS; TOPNOTCH MD) JESSICA MAE SANCHEZ, MD (TOP 4 - AUG 2014 MED BOARDS; TOPNOTCH MD) JESSICA MAE SANCHEZ, MD (TOP 4 - AUG 2014 MED BOARDS; TOPNOTCH MD)

TOPNOTCH MEDICAL BOARD PREP ANATOMY SUPEREXAM Page 45 of 94 For inquiries visit www.topnotchboardprep.com.ph or email us at [email protected]

BACK-UP MIDTERM EXAM - FEB 2015

BACK-UP MIDTERM EXAM - FEB 2015

BACK-UP MIDTERM EXAM - FEB 2015

BACK-UP MIDTERM EXAM - FEB 2015

BACK-UP MIDTERM EXAM - FEB 2015

BACK-UP MIDTERM EXAM - FEB 2015

BACK-UP MIDTERM EXAM - FEB 2015

BACK-UP MIDTERM EXAM - FEB 2015

BACK-UP MIDTERM EXAM - FEB 2015

TOPNOTCH MEDICAL BOARD PREP ANATOMY SUPEREXAM For inquiries visit www.topnotchboardprep.com.ph or email us at [email protected] Item # 353

QUESTION

EXPLANATION

Melena results from gastrointestinal bleeding found: A. Proximal to the ligament of Treitz B. Distal to the ligament of Treitz C. At the stomach only D. At the esophagogastric area only



354

A 40 year old man sustained a gunshot wound that traversed segments 1 and 4 of the liver. This means that the parts of the liver affected were: A. Quadrate lobe B. Caudate lobe C. Both D. Neither

Reference: Topnotch Anatomy Handouts

355

The extent to which neoplastic cells resemble comparable normal cells is referred to as: A. Differentiation B. Pleomorphism C. Anaplasia D. Dysplasia



356

A patient complains of a boil located on her labia majora. Lymphatic spread of the infection would most likely enlarge which nodes? A. Lumbar nodes B. Superficial inguinal nodes C. External iliac nodes D. Internal iliac nodes

Reference: Topnotch Anatomy Handouts

357

A 63 year old female patient says that she has pain in her groin and upper thigh. Upon examination, you palpate a lump located below the inguinal ligament, lateral to its attachment to the pubic tubercle. You suspect that this may be a hernia passing through the: A. femoral canal B. obturator canal C. adductor hiatus D. superficial inguinal ring The sensory receptor for hearing is located in the: A. Macula B. Ampulla C. Organ of Corti D. Saccule

A femoral hernia is more common in women that in men. The neck of the sac always lies below and lateral to the pubic tubercle, distinguishing it from an inguinal hernia, which lies above and medial to the pubic tubercle. Reference: Snell. Clinical Anatomy by Regions, 8th ed. p. 580

359

A forearm arterio-venous fistula for hemodialysis access is usually done by anastomosing the: A. Radial artery and basilic vein B. Radial artery and cephalic vein C. Radial artery and antecubital vein D. Ulnar artery and basilic vein

Radiocephalic AV fistula is the vascular access of choice for hemodialysis.

360

The aortic valve is best heard clinically at the: A. Right 2nd ICS close to the sternal border B. Left 2nd ICS close to the sternal border C. Left 5th ICS midclavicular line D. Xiphisternal junction



361

Gab is an avid tennis player with a killer serve. In order to do this, he must have a good grip around the racket. Which muscle improves the grip of the palm especially when holding rounded objects? A. Abductor pollicis brevis B. Flexor pollicis brevis C. Palmaris brevis D. Extensor pollicis brevis

palmaris brevis (Fig. 9.55) is a small muscle that arises from the flexor retinaculum and palmar aponeurosis and is inserted into the skin of the palm. It is supplied by the superficial branch of the ulnar nerve. Its function is to corrugate the skin at the base of the hypothenar eminence and so improve the grip of the palm in holding a rounded object. Snell 9th pg 397 APB – Abduction of the thumb FPB - Flexes MCP joint of the thumb EPB – Extends MCP joint of the thumb

358

AUTHOR JESSICA MAE SANCHEZ, MD (TOP 4 - AUG 2014 MED BOARDS; TOPNOTCH MD) JESSICA MAE SANCHEZ, MD (TOP 4 - AUG 2014 MED BOARDS; TOPNOTCH MD) JESSICA MAE SANCHEZ, MD (TOP 4 - AUG 2014 MED BOARDS; TOPNOTCH MD) JESSICA MAE SANCHEZ, MD (TOP 4 - AUG 2014 MED BOARDS; TOPNOTCH MD) JESSICA MAE SANCHEZ, MD (TOP 4 - AUG 2014 MED BOARDS; TOPNOTCH MD) JESSICA MAE SANCHEZ, MD (TOP 4 - AUG 2014 MED BOARDS; TOPNOTCH MD) JESSICA MAE SANCHEZ, MD (TOP 4 - AUG 2014 MED BOARDS; TOPNOTCH MD) JESSICA MAE SANCHEZ, MD (TOP 4 - AUG 2014 MED BOARDS; TOPNOTCH MD) MAIRRE JAMES GADDI, MD (TOP 4 - AUG 2013 MED BOARDS; TOPNOTCH MD)

TOPNOTCH MEDICAL BOARD PREP ANATOMY SUPEREXAM Page 46 of 94 For inquiries visit www.topnotchboardprep.com.ph or email us at [email protected]

TOPNOTCH EXAM BACK-UP MIDTERM EXAM - FEB 2015

BACK-UP MIDTERM EXAM - FEB 2015

BACK-UP MIDTERM EXAM - FEB 2015

BACK-UP MIDTERM EXAM - FEB 2015

BACK-UP MIDTERM EXAM - FEB 2015

BACK-UP MIDTERM EXAM - FEB 2015

BACK-UP MIDTERM EXAM - FEB 2015

BACK-UP MIDTERM EXAM - FEB 2015

BACK-UP MIDTERM EXAM - FEB 2015

TOPNOTCH MEDICAL BOARD PREP ANATOMY SUPEREXAM For inquiries visit www.topnotchboardprep.com.ph or email us at [email protected] Item # 362

QUESTION

EXPLANATION

AUTHOR

TOPNOTCH EXAM BACK-UP MIDTERM EXAM - FEB 2015

Marco Reus of Borussia Dortmund recently damaged his ankle ligaments following a tackle from the outside while on international duty with Germany. Which ligament was most likely involved? A. Anterior talofibular ligament B. Posterior talofibular ligament C. Calcaneofibular ligament D. Anterior tibiotalar ligament

Lateral ligament sprains occur in running and jumping sports. The lateral ligament is injured because it is much weaker than the medial liga- ment, and is the ligament that resists inversion at the talocrural joint. The anterior talofibular ligament is most vulnerable and most commonly torn during ankle sprains, either partially or completely, resulting in instability of the ankle joint. Moore 7th pg 666

MAIRRE JAMES GADDI, MD (TOP 4 - AUG 2013 MED BOARDS; TOPNOTCH MD)

363

In order to achieve anesthesia of an intercostal space, an intercostal nerve block can be done by inserting the needle through: A. the superior margin of the rib above the space B. the inferior margin of the rib above the space C. the superior margin of the rib below the space D. the inferior margin of the rib below the space E. Between the inferior margin of the rib above the space and the superior margin of the rib below the space

In the chest wall, the neurovascular bundles are sheltered by the inferior margins of the overlying ribs. The needle is directed toward the rib near the lower border and the tip comes to rest near the subcostal groove, where the anesthetic is infiltrated around the nerve. Remember that the order of structures from above downward is intercostal vein, artery, and nerve (VAN) Snell 9th pg 43

MAIRRE JAMES GADDI, MD (TOP 4 - AUG 2013 MED BOARDS; TOPNOTCH MD)

BACK-UP MIDTERM EXAM - FEB 2015

364

A patient with a large fungating tongue mass is schedule to undergo operative intervention. The anesthesiologist plans to do nasotracheal intubation. He knows that the distance from the external nares to the carina measures: A. 15 cm B. 20 cm C. 30 cm D. 40 cm

The distance between the incisors to the vocal cords measures 5.9in (15cm), between the incisors to the carina 7.9in (20cm), between the external nares to the carina 11.8in (30cm). Snell 9th pg 720

MAIRRE JAMES GADDI, MD (TOP 4 - AUG 2013 MED BOARDS; TOPNOTCH MD)

BACK-UP MIDTERM EXAM - FEB 2015

365

During tracheostomy it is important to note that the thyroid isthmus is at the level of the: A. Cricoid and 1st tracheal ring B. 1st and 2nd tracheal rings C. 2nd and 3rd tracheal rings D. Cricoid and 2nd tracheal rings

The isthmus is usually anterior to the second and third tracheal rings. Moore 7th pg1018

BACK-UP MIDTERM EXAM - FEB 2015

366

Dr. Wi, the resident on duty received a patient involved in a vehicular crash and upon seeing the patient noted extensive maxillofacial injuries. During the subsequent management imaging was done which showed a fracture line involving the frontozygomatic sutures, the ethmoid and nasal bones, the superior orbital fissures and the greater wings of the sphenoid. His diagnosis would be? A. Le Fort I B. Le Fort II C. Le Fort III D. Le Fort IV

Le Fort I: horizontal fractures of the maxillae, passing superior to the maxillary alveolar process Le Fort II: passes from the posterolateral parts of the maxillary sinuses superomedially through the infraorbital foramina, lacrimals, or ethmoids to the bridge of the nose. Le Fort III: horizontal fracture that passes through the superior orbital fissures and the ethmoid and nasal bones and extends laterally through the greater wings of the sphenoid and the frontozygomatic sutures. There is no Le Fort IV. Moore 7th pg 837

MAIRRE JAMES GADDI, MD (TOP 4 - AUG 2013 MED BOARDS; TOPNOTCH MD) MAIRRE JAMES GADDI, MD (TOP 4 - AUG 2013 MED BOARDS; TOPNOTCH MD)

367

During left radical nephrectomy, the left kidney is removed together with the left suprarenal gland. As part of the procedure, the renal artery is ligated. To prevent bleeding of the vessels supplying the suprarenal gland, arterial branch/branches coming from which of the following should also be ligated? A. Inferior phrenic artery B. Aorta C. Splenic artery D. A and B E. All of the above

The suprarenal glands are supplied by three arteries, inferior phrenic artery (superior), abdominal aorta (middle) and renal artery (inferior) Snell 9th pg 215

MAIRRE JAMES GADDI, MD (TOP 4 - AUG 2013 MED BOARDS; TOPNOTCH MD)

BACK-UP MIDTERM EXAM - FEB 2015

368

True of the rectum: A. The rectum is a purely retroperitoneal organ and begins as a continuation of the sigmoid colon B. It is covered by a part of peritoneum up to its distal third C. The proximal third is covered by peritoneum at its anterior and lateral surfaces D. All of the above

The rectum is primarily retroperitoneal but it is also subperitoneal. The peritoneum covers the anterior and lateral surfaces of the first third of the rectum and only the anterior surface of the middle third, leaving the lower third devoid of peritoneum. Moore 7th pgs 253, 370; Snell 9th pg 265

MAIRRE JAMES GADDI, MD (TOP 4 - AUG 2013 MED BOARDS; TOPNOTCH MD)

BACK-UP MIDTERM EXAM - FEB 2015

369

What structure/s pass/es through the diaphragmatic opening at the level of T12? A. Aorta and Thoracic duct B. Vena cava and phrenic nerve C. Azygos vein D. A and C E. All of the above

The caval opening lies at the level of T8 and transmits the inferior vena cava and the terminal branches of the right phrenic nerve. The esophageal opening lies at the level of T10 and transmits the esophagus, the right and left vagus nerves, the esophageal branches of the left gastric vessels and the lymphatics of the lower third of the esophagus. The aortic opening lies at the level of T12 and transmits the aorta, the thoracic duct

MAIRRE JAMES GADDI, MD (TOP 4 - AUG 2013 MED BOARDS; TOPNOTCH MD)

BACK-UP MIDTERM EXAM - FEB 2015

TOPNOTCH MEDICAL BOARD PREP ANATOMY SUPEREXAM Page 47 of 94 For inquiries visit www.topnotchboardprep.com.ph or email us at [email protected]

BACK-UP MIDTERM EXAM - FEB 2015

TOPNOTCH MEDICAL BOARD PREP ANATOMY SUPEREXAM For inquiries visit www.topnotchboardprep.com.ph or email us at [email protected] Item #

QUESTION

EXPLANATION

AUTHOR

TOPNOTCH EXAM

and the azygos vein. Snell 9th pg 46

370

A 56/F diagnosed with endometrial carcinoma confined to the uterus on TV-UTZ was about to undergo a radical hysterectomy with bilateral salpingo-oophorectomy, the attending OBGYN wanting to avoid post operative complications recalls that the ureter can sustain iatrogenic injury at which of the following locations? A. As it passes immediately inferior to the uterine artery near the lateral part of the fornix of the vagina B. As it crosses the uterine artery approximately 2 cms superior to the ischial spine C. As it crosses the pelvic brim together with the ovarian artery D. All of the above

The ureter can be damaged during ligation of the uterine artery during hysterectomy as it passes immediately inferior to the uterine artery near the lateral part of the fornix of the vagina and as it crosses the uterine artery approximately 2 cms superior to the ischial spine. It can also be damaged during oophorectomy when the ovarian artery is ligated since they are close to each other as they cross the pelvic brim. Moore 7th pg 361

MAIRRE JAMES GADDI, MD (TOP 4 - AUG 2013 MED BOARDS; TOPNOTCH MD)

BACK-UP MIDTERM EXAM - FEB 2015

371

A 34/M was on his way home when he was suddenly mugged and was hit at the side of the head with a lead pipe by unknown assailants. He was briefly unconscious and was awakened by concerned bystanders. He then reported the incident to the police and since there were no obvious external injuries he decided to go home. The following morning, he was found dead by his family. The vessel that was most likely responsible for the death of the patient is a branch of the? A. Middle meningeal artery B. Internal carotid artery C. External carotid artery D. Superior sagittal sinus

The patient most likely suffered from an extradural hematoma which usually presents as a brief concussion followed by a lucid interval for a few hours. This then leads to drowsiness and coma. Extradural hematomas are arterial in origin and are usually caused by extravasation of blood from the torn branches of the middle meningeal artery (specifically the anterior branch which lies immediately on top of the pterion). The middle meningeal artery is a branch of the maxillary artery which in itself is a branch of the external carotid artery. Moore 7th pg 876

MAIRRE JAMES GADDI, MD (TOP 4 - AUG 2013 MED BOARDS; TOPNOTCH MD)

BACK-UP MIDTERM EXAM - FEB 2015

372

A 60/M smoker, diabetic, suddenly experienced severe crushing chest pain. He was brought to the nearest hospital and an ECG was done which showed ST elevation of leads V4-V6. Which vessel was most likely occluded? A. Left circumflex artery B. Left anterior descending artery C. Right marginal artery D. Left main coronary artery

MAIRRE JAMES GADDI, MD (TOP 4 - AUG 2013 MED BOARDS; TOPNOTCH MD)

BACK-UP MIDTERM EXAM - FEB 2015

373

A 32/F underwent total thyroidectomy. Postoperatively the patient complains of hoarseness. This suggests damage to the _________ which is closely related to the___________. A. Recurrent laryngeal nerve: Superior thyroid artery B. Recurrent laryngeal nerve: Inferior thyroid artery C. External laryngeal nerve: Superior thyroid artery D. External laryngeal nerve: Inferior thyroid artery

MAIRRE JAMES GADDI, MD (TOP 4 - AUG 2013 MED BOARDS; TOPNOTCH MD)

BACK-UP MIDTERM EXAM - FEB 2015

Snell 9th pg 89 Near the inferior pole of the thyroid gland, the right recurrent laryngeal nerve is intimately related to the inferior thyroid artery. Because of this the inferior thyroid artery is ligated some distance lateral to the thyroid gland. Although the danger of injuring the left recurrent laryngeal nerve during surgery is not as great, owing to its more vertical ascent from the superior mediastinum, the artery and nerve are also closely associated near the inferior pole of the thyroid gland. Hoarseness is the usual sign of unilateral recurrent nerve injury; however, temporary aphonia or disturbance of phonation and laryngeal spasm may occur. These signs usually result from bruising the recurrent laryngeal nerves during surgery or from the pressure of accumulated blood and serous exudate after the operation. Moore 7th pg 1043

TOPNOTCH MEDICAL BOARD PREP ANATOMY SUPEREXAM Page 48 of 94 For inquiries visit www.topnotchboardprep.com.ph or email us at [email protected]

TOPNOTCH MEDICAL BOARD PREP ANATOMY SUPEREXAM For inquiries visit www.topnotchboardprep.com.ph or email us at [email protected] Item # 374

QUESTION

EXPLANATION

AUTHOR

TOPNOTCH EXAM BACK-UP MIDTERM EXAM - FEB 2015

A 7/M fell while from a tree and landed on his outstretched hand. He was quickly brought to the emergency room and the examining physician noted that the limb is medially rotated with loss of abduction. There was also a step off deformity palpable at the middle third of the clavicle. What nerve was most likely injured? A. Subscapular nerve B. Median nerve C. Axillary nerve D. Suprascapular nerve

The suprascapular nerve is vulnerable to injury in fractures of the middle third of the clavicle. Injury to the suprascapular nerve results in loss of lateral rotation of the humerus at the glenohumeral joint. Consequently the relaxed limb rotates medially into the waiter’s tip position. The ability to initiate abduction of the limb is also affected. Injury to the axillary nerve from fracture of the surgical neck of the humerus or dislocation of the humeral head will produce the same set of deficits. Moore 7th pg 710 & 1009

MAIRRE JAMES GADDI, MD (TOP 4 - AUG 2013 MED BOARDS; TOPNOTCH MD)

375

A 27/F has multiple closed comedones on her nose. She then proceeded to squeeze them individually. After a few days she had a severe headache which was followed by high grade fever, periorbital edema and diplopia. A diagnosis of cavernous sinus thrombosis was made. Which is/are found inside the cavernous sinus? A. Internal carotid artery B. Trochlear nerve C. Mandibular nerve D. A and B E. All of the above

Inside each cavernous sinus is the internal carotid artery with its small branches, surrounded by the carotid plexus of sympathetic nerve(s), and the abducent nerve. The oculomotor (CN III) and trochlear (CN IV) nerves, plus two of the three divisions of the trigeminal nerve (CN V1 and V2) are embedded in the lateral wall of the sinus. Moore 7th pg 869

MAIRRE JAMES GADDI, MD (TOP 4 - AUG 2013 MED BOARDS; TOPNOTCH MD)

BACK-UP MIDTERM EXAM - FEB 2015

376

Which of the following is/are TRUE of the inferior vena cava? A. There are two anterior visceral tributaries B. There are three veins of origin C. There are three visceral tributaries D. All of the above E. A and C A 30/M has extensive metastatic testicular carcinoma. Metastatic foci would most likely be found where? A. Superficial inguinal lymph nodes B. Deep inguinal lymph nodes C. Para-aortic lymph nodes D. Pre-aortic lymph nodes E. All of the above

MAIRRE JAMES GADDI, MD (TOP 4 - AUG 2013 MED BOARDS; TOPNOTCH MD) MAIRRE JAMES GADDI, MD (TOP 4 - AUG 2013 MED BOARDS; TOPNOTCH MD)

BACK-UP MIDTERM EXAM - FEB 2015

377

378

Renal transplantation involves: A. End to side anastomosis of the donor renal artery to the recipient external iliac artery B. End to end anastomosis of the donor renal vein to the recipient internal iliac vein C. Anastomosis of the donor ureter to the recipient bladder through a ureterocystostomy D. All of the above E. A and B

MAIRRE JAMES GADDI, MD (TOP 4 - AUG 2013 MED BOARDS; TOPNOTCH MD)

BACK-UP MIDTERM EXAM - FEB 2015

379

The following are wholly derived from endoderm EXCEPT A. Greater vestibular glands B. Prostate gland C. Mammary glands D. Bulbourethral glands

The inferior vena cava has the following tributaries: 2 anterior visceral tributaries (hepatic veins), 3 lateral visceral tributaries (right suprarenal, right renal and right gonadal vein), 5 lateral abdominal wall tributaries (one inferior phrenic vein and four lumbar veins) and 3 veins of origin (two common iliac veins and the median sacral vein). Snell 9th pg 218 The preaortic lymph nodes lie around the origins of the celiac, superior mesenteric, and inferior mesenteric arteries and are referred to as the celiac, superior mesenteric, and inferior mesenteric lymph nodes, respectively. They drain the lymph from the gastrointestinal tract, extending from the lower one third of the esophagus to halfway down the anal canal, and from the spleen, pancreas, gallbladder, and greater part of the liver. lateral aortic (para-aortic or lumbar) lymph nodes drain lymph from the kidneys and suprarenals; from the testes in the male and from the ovaries, uterine tubes, and fundus of the uterus in the female; from the deep lymph vessels of the abdominal walls; and from the common iliac nodes. Snell 9th pgs 132 & 220 The iliac fossa on the posterior abdominal wall is the usual site chosen for transplantation of the kidney. The fossa is exposed through an incision in the anterior abdominal wall just above the inguinal ligament. The iliac fossa in front of the iliacus muscle is approached retroperitoneally. The renal artery is anastomosed end to end to the internal iliac artery and the renal vein is anastomosed end to side to the external iliac vein. Ureterocystostomy is then performed by opening the bladder and providing a wide entrance of the ureter through the bladder wall. Snell 9th pg 210 Ectoderm derivatives - central nervous system, cornea, retina, and lens, membranous labyrinth of the inner ear, epidermis, epithelial cells of the sebaceous, sweat, and mammary glands; the mucous membrane lining the mouth, nasal cavities, and paranasal sinuses; the pituitary gland and the alveoli and ducts of the parotid salivary glands; the mucous membrane of the lower half of the anal canal; and the terminal parts of the genital tract and the male urinary tract Entoderm: epithelial lining of the alimentary tract from the mouth cavity down to halfway along the anal canal, thyroid, parathyroid, thymus, liver, and pancreas, linings of the respiratory tract, pharyngotympanic tube and middle ear, urinary bladder, parts of the female and male urethras, greater vestibular glands, prostate gland, bulbourethral glands, and vagina.

MAIRRE JAMES GADDI, MD (TOP 4 - AUG 2013 MED BOARDS; TOPNOTCH MD)

BACK-UP MIDTERM EXAM - FEB 2015

TOPNOTCH MEDICAL BOARD PREP ANATOMY SUPEREXAM Page 49 of 94 For inquiries visit www.topnotchboardprep.com.ph or email us at [email protected]

BACK-UP MIDTERM EXAM - FEB 2015

TOPNOTCH MEDICAL BOARD PREP ANATOMY SUPEREXAM For inquiries visit www.topnotchboardprep.com.ph or email us at [email protected] Item # 380

QUESTION

EXPLANATION

Which of the following contains a submucosal layer? A. Gallbladder B. Trachea C. Fallopian tubes D. Larynx

The wall of the gallbladder does not contain a muscularis mucosae or submucosa. diFiore 12th pg 384, the fallopian tubes lack a submucosal layer diFiore 12th pg 520, the lamina propria of the laryngeal mucosa blends with the perichondrium of the thyroid cartilage and there is no distinct submucosa diFiore 12th pg 398.

381

Which of the following best describes the location of the right adrenal gland? A. Inferior to the right lobe of the liver B. Posterolateral to the inferior vena cava C. Anterior to the 1st lumbar vertebra D. Posterior to the right kidney E. Medial to the spleen

SIMILAR TO PREVIOUS BOARD EXAM CONCEPT/PRINCIPLE. The right adrenal gland is posterolateral to the inferior vena cava, lateral to the 1st lumbar vertebrae, posterior to the right lobe of the liver and supeior to the right kidney.

382

Which of the following drains segment I of the liver? A. Middle hepatic vein B. Right hepatic vein C. Direct venous drainage into the inferior vena cava D. Direct venous drainage into the common hepatic vein E. Left gastric vein



383

A 65-year old female who has been bedridden for 3 months presented with symptoms of pneumonia. You suspect an aspiration component for her disease. In which of the following lung segments will you most likely see the infiltrates on her chest radiograph? A. Left superior lingular segment B. Left apicoposterior segment C. Right superior segment D. Right posterobasal segment E. Right medial segment



384

The embryonic right 4th aortic arch develops into which of the following adult structures? A. Arch of the aorta B. Right internal carotid artery C. Right common carotid artery D. Right subclavian artery E. Right brachiocephalic artery

385

AUTHOR MAIRRE JAMES GADDI, MD (TOP 4 - AUG 2013 MED BOARDS; TOPNOTCH MD) SCOTT RILEY ONG, MD (TOP 5 - AUG 2014 MED BOARDS; TOPNOTCH MD)

TOPNOTCH EXAM BACK-UP MIDTERM EXAM - FEB 2015

BACK-UP MIDTERM EXAM - FEB 2015

SCOTT RILEY ONG, MD (TOP 5 - AUG 2014 MED BOARDS; TOPNOTCH MD) SCOTT RILEY ONG, MD (TOP 5 - AUG 2014 MED BOARDS; TOPNOTCH MD)

BACK-UP MIDTERM EXAM - FEB 2015

It contributes to the proximal part of the right subclavian artery. The left 4th aortic arch forms the arch of the aorta.

SCOTT RILEY ONG, MD (TOP 5 - AUG 2014 MED BOARDS; TOPNOTCH MD)

BACK-UP MIDTERM EXAM - FEB 2015

Which of the following is not true about the lesser sac of the abdomen? A. The stomach forms its anterior boundary B. It communicates with the greater sac through the foramen of Winslow C. The gastrosplenic ligament forms its lateral boundary D. The lesser omentum transmits the coronary veins which can dilate as varices during portal hypertension E. The lesser sac is normally filled with fluid and is easily visualized on CT imaging.

The lesser sac is normally collapsed. It can become huge and visible when it is filled with fluid (eg. ascites) during disease states.

SCOTT RILEY ONG, MD (TOP 5 - AUG 2014 MED BOARDS; TOPNOTCH MD)

BACK-UP MIDTERM EXAM - FEB 2015

386

During a 12-lead ECG, where should lead V2 be normally placed? A. 4th intercostal space, left parasternal border B. 5th intercostal space, left parasternal border C. 4th intercostal space, left midclavicular line D. 5th intercostal space, left midclavicular line E. 4th intercostal space, right parasternal border

SIMILAR TO PREVIOUS BOARD EXAM CONCEPT/PRINCIPLE.

SCOTT RILEY ONG, MD (TOP 5 - AUG 2014 MED BOARDS; TOPNOTCH MD)

BACK-UP MIDTERM EXAM - FEB 2015

387

The ureters enter the urinary bladder at which of its aspects? A. Anterolateral B. Lateral C. Posteromedial D. Posterolateral E. Superomedial

SIMILAR TO PREVIOUS BOARD EXAM CONCEPT/PRINCIPLE.

SCOTT RILEY ONG, MD (TOP 5 - AUG 2014 MED BOARDS; TOPNOTCH MD)

BACK-UP MIDTERM EXAM - FEB 2015

TOPNOTCH MEDICAL BOARD PREP ANATOMY SUPEREXAM Page 50 of 94 For inquiries visit www.topnotchboardprep.com.ph or email us at [email protected]

BACK-UP MIDTERM EXAM - FEB 2015

TOPNOTCH MEDICAL BOARD PREP ANATOMY SUPEREXAM For inquiries visit www.topnotchboardprep.com.ph or email us at [email protected] Item # 388

QUESTION

EXPLANATION

AUTHOR

TOPNOTCH EXAM BACK-UP MIDTERM EXAM - FEB 2015

A patient suffered from myocardial infarction and showed signs of hypotension and bradycardia. Which myocardial wall is most likely affected given this clinical picture. A. Anterior wall B. Anteroseptal wall C. Lateral wall D. Inferior wall E. Posterior wall

SIMILAR TO PREVIOUS BOARD EXAM CONCEPT/PRINCIPLE.

SCOTT RILEY ONG, MD (TOP 5 - AUG 2014 MED BOARDS; TOPNOTCH MD)

389

The thyroid isthmus most commonly lies over which tracheal rings? A. 1st and 2nd B. 2nd and 3rd C. 3rd and 4th D. 4th and 5th E. 5th and 6th

SIMILAR TO PREVIOUS BOARD EXAM CONCEPT/PRINCIPLE.

SCOTT RILEY ONG, MD (TOP 5 - AUG 2014 MED BOARDS; TOPNOTCH MD)

BACK-UP MIDTERM EXAM - FEB 2015

390

Which of the following paranasal sinuses is the last to develop? A. Maxillary sinus B. Ethmoid sinus C. Sphenoid sinus D. Frontal sinus E. Both C and D

Present at birth: maxillary and ethmoid sinuses. 3 years old: sphenoid sinus begins to appear. 6 years old: frontal sinus begins to develop.

SCOTT RILEY ONG, MD (TOP 5 - AUG 2014 MED BOARDS; TOPNOTCH MD)

BACK-UP MIDTERM EXAM - FEB 2015

391

The superior mesenteric vein drains directly into which of the following structures? A. Portal vein B. Celiac vein C. Inferior vena cava D. Splenic vein E. Common hepatic vein

The portal vein is formed by the union of the superior mesenteric vein and splennic vein.

SCOTT RILEY ONG, MD (TOP 5 - AUG 2014 MED BOARDS; TOPNOTCH MD)

BACK-UP MIDTERM EXAM - FEB 2015

392

Which of the following forms the posterior border of the heart? A. Right atrium B. Right ventricle C. Left atrium D. Left ventricle E. Apex

The right ventricle forms its anterior and inferior border. The right atrium forms its right border. The left ventricle forms its left border.

SCOTT RILEY ONG, MD (TOP 5 - AUG 2014 MED BOARDS; TOPNOTCH MD)

BACK-UP MIDTERM EXAM - FEB 2015

393

Which of the following structures is not contained within the spermatic cord? A. Vas deferens B. Ilioinguinal nerve C. Genital branch of genitofemoral nerve D. Pampiniform plexus E. Tunica vaginalis

The ilioinguinal nerve runs along with but outside the spermatic cord.

SCOTT RILEY ONG, MD (TOP 5 - AUG 2014 MED BOARDS; TOPNOTCH MD)

BACK-UP MIDTERM EXAM - FEB 2015

394

Which of the following parts of the urinary bladder is most sensitive to stretch? A. Apex B. Base C. Trigone D. Ureteral insertion E. Urethral opening

SIMILAR TO PREVIOUS BOARD EXAM CONCEPT/PRINCIPLE.

SCOTT RILEY ONG, MD (TOP 5 - AUG 2014 MED BOARDS; TOPNOTCH MD)

BACK-UP MIDTERM EXAM - FEB 2015

395

The stylohyoid muscle is innervated by which of the following nerves? A. CN V3 B. CN VII C. CN IX D. CN X E. CN XII

CN V3: masticator muscles, anterior belly of digastric, mylohyoid. CN VII: facial muscles, stapedius, stylohyoid, posterior belly of digastric. CN IX: stylopharyngeus. CN XII: intrinsic and extrinsic tongue muscles (except palatoglossus, which is innervated by CN X)

SCOTT RILEY ONG, MD (TOP 5 - AUG 2014 MED BOARDS; TOPNOTCH MD)

BACK-UP MIDTERM EXAM - FEB 2015

396

An avulsion fracture at the base of the first proximal phalanx is known as: A. Gamekeeper's thumb B. Bennett's fracture C. Boxer's fracture D. Colles fracture E. Lisfranc fracture

Gamekeeper's thumb: base of 1st proximal phalanx. Bennett's fracture: base of 1st metacarpal. Boxer's fracture: neck of 4th and 5th metacarpal. Lisfranc fracture: metatarsal fracture

SCOTT RILEY ONG, MD (TOP 5 - AUG 2014 MED BOARDS; TOPNOTCH MD)

BACK-UP MIDTERM EXAM - FEB 2015

TOPNOTCH MEDICAL BOARD PREP ANATOMY SUPEREXAM Page 51 of 94 For inquiries visit www.topnotchboardprep.com.ph or email us at [email protected]

TOPNOTCH MEDICAL BOARD PREP ANATOMY SUPEREXAM For inquiries visit www.topnotchboardprep.com.ph or email us at [email protected] Item # 397

QUESTION

EXPLANATION

AUTHOR

TOPNOTCH EXAM BACK-UP MIDTERM EXAM - FEB 2015

Which of the following inner ear structures is involved during horizontal linear deceleration? A. Utricle B. Saccule C. Horizontal semicircular canal D. Scala media E. Basilar membrane

SIMILAR TO PREVIOUS BOARD EXAM CONCEPT/PRINCIPLE. Utricle: horizontal linear acceleration. Saccule: vertical linear acceeleration. Semicircular canals: angular acceleration. Scala media and basilar membrane: involved in hearing function.

SCOTT RILEY ONG, MD (TOP 5 - AUG 2014 MED BOARDS; TOPNOTCH MD)

398

Gastrointestinal stromal tumor (GIST), the most common mesenchymal tumor of the stomach, most commonly arises from which of its layers? A. Mucosa B. Submucosa C. Muscularis mucosae D. Muscularis propria E. Serosa

SIMILAR TO PREVIOUS BOARD EXAM CONCEPT/PRINCIPLE.

SCOTT RILEY ONG, MD (TOP 5 - AUG 2014 MED BOARDS; TOPNOTCH MD)

BACK-UP MIDTERM EXAM - FEB 2015

399

What is the lining epithelium of the ovary? A. Simple squamous B. Simple cuboidal C. Simple columnar D. Stratified squamous E. Stratified cuboidal

SIMILAR TO PREVIOUS BOARD EXAM CONCEPT/PRINCIPLE.

BACK-UP MIDTERM EXAM - FEB 2015

400

What is the lining epithelium of the prostatic urethra A. Simple squamous B. Stratified squamous C. Transitional D. Simple cuboidal E. Stratified cuboidal

SIMILAR TO PREVIOUS BOARD EXAM CONCEPT/PRINCIPLE.

401

What is the most common type of Myoma Uteri? A. Submucous B. Pedunculated C. Subserosal D. Intramural E. Polypoid

Yes, gyne questions can be seen in anatomy! SIMILAR TO PREVIOUS BOARD EXAM CONCEPT/PRINCIPLE

402

The Dosalis Pedis Artery is A. A continuation of the posterior tibial artery B. Medial to the adductor longus tendon C. Medial to the medial malleolus D. Lateral to the Extensor digitorum longus E. Lateral to the Extensor hallucis longus

SIMILAR TO PREVIOUS BOARD EXAM CONCEPT/PRINCIPLE

403

Which of the following is derived from the 2nd pharyngeal arch A. Reichter's Cartilage B. Meckel's Cartilage C. Hyoid bone D. A and C E. All of the above

Meckel's Cartilage is derived from the 1st pharyngeal arch, Hyoid which is derived from the Reichter's Cartilage is derived from the 2nd pharyngeal arch

SCOTT RILEY ONG, MD (TOP 5 - AUG 2014 MED BOARDS; TOPNOTCH MD) SCOTT RILEY ONG, MD (TOP 5 - AUG 2014 MED BOARDS; TOPNOTCH MD) JOSE CARLO MASANGKAY III, MD (TOP 8 - FEB 2014 MED BOARDS; TOPNOTCH MD) JOSE CARLO MASANGKAY III, MD (TOP 8 - FEB 2014 MED BOARDS; TOPNOTCH MD) JOSE CARLO MASANGKAY III, MD (TOP 8 - FEB 2014 MED BOARDS; TOPNOTCH MD)

404

The Cremaster Muscle is a continuation of which of the following? A. External Oblique Muscle B. Rectus abdominis muscle C. Internal Oblique muscle D. Transversalis muscle E. Transversalis fascia

Ext spermatic fascia derived from Ext. oblique muscle, Darto's from Superficial fascia, Internal spermatic fascia from Transversalis fascia

JOSE CARLO MASANGKAY III, MD (TOP 8 - FEB 2014 MED BOARDS; TOPNOTCH MD)

DIAGNOSTIC EXAM - AUG 2014

405

Emphysema is a pulmonary disease where expiration is active instead of its passive nature. In a patient with emphysema which of the following is not a muscle used in forced/active expiration? A. Internal Intecostal B. External Intercostal C. Rectus abdominis D. Serratus Posterior Inferior E. Transverse Thoracis



JOSE CARLO MASANGKAY III, MD (TOP 8 - FEB 2014 MED BOARDS; TOPNOTCH MD)

DIAGNOSTIC EXAM - AUG 2014

TOPNOTCH MEDICAL BOARD PREP ANATOMY SUPEREXAM Page 52 of 94 For inquiries visit www.topnotchboardprep.com.ph or email us at [email protected]

BACK-UP MIDTERM EXAM - FEB 2015

DIAGNOSTIC EXAM - AUG 2014

DIAGNOSTIC EXAM - AUG 2014

DIAGNOSTIC EXAM - AUG 2014

TOPNOTCH MEDICAL BOARD PREP ANATOMY SUPEREXAM For inquiries visit www.topnotchboardprep.com.ph or email us at [email protected] Item # 406

QUESTION

EXPLANATION

AUTHOR

TOPNOTCH EXAM DIAGNOSTIC EXAM - AUG 2014

Which of the following is the reason for the higher tendency of the Sigmoid Colon for Volvolus? A. Because it is most redundant B. Because It is intraperitoneal C. Because it is the narrowest D. Because it is hypermotile E. Because of its thin wall

SIMILAR TO PREVIOUS BOARD EXAM CONCEPT/PRINCIPLE

JOSE CARLO MASANGKAY III, MD (TOP 8 - FEB 2014 MED BOARDS; TOPNOTCH MD)

407

Which of the following Sinuses is/are present at birth? A. Frontal B. Maxillary C. Sphenoid D. A and B E. B and C



DIAGNOSTIC EXAM - AUG 2014

408

A patient was rushed to the ED after an automobile accident, patient came in with decreased sensorium and multiple abrasions, after a few hours, the patient became conscious and coherent which was eventually followed by a coma, on physical examination you noted crepitations on the Left Pterion area. As a brilliant doctor you knew that there was an injured vessel which passes in which foramen of the skull A. Foramen Magnun B. Foramen Lacerum C. Foramen Ovale D. Foramen Spinosum E. Foramen Rotundum

A Lucid interval was noted in this patient this is frequently associated with the presence of an epidural hematoma due to a rupture of the middle meningeal artery which passes in proximity to the pterion. The middle meningeal artery passes thru the Foramen Spinosum

JOSE CARLO MASANGKAY III, MD (TOP 8 - FEB 2014 MED BOARDS; TOPNOTCH MD) JOSE CARLO MASANGKAY III, MD (TOP 8 - FEB 2014 MED BOARDS; TOPNOTCH MD)

409

Which of the following structures traverses the Aortic Hiatus of the diaphragm? A. Thoracic Duct B. Hemiazygous vein C. Right Phrenic Nerve D. Vagus Nerve E. Right Lymphatic Duct

The Aorta, Thoracic Duct and Azygous vein enters the aortic hiatus.

JOSE CARLO MASANGKAY III, MD (TOP 8 - FEB 2014 MED BOARDS; TOPNOTCH MD)

DIAGNOSTIC EXAM - AUG 2014

410

A 1-month old female patient was noted to have Ribnotching on X-ray, a chromosomal study was done in this patient revealing a chromosomal count of 45 XO, if this patient would undergo repair of the said cardiac defect what structure may be injured? A. Right Phrenic Nerve B. Right Vagus Nerve C. Left Vagus Nerve D. Left Phrenic Nerve E. Hemiazygous vein

Patient is a case of Turner's syndrome, these patients have a propensity to develop a Patent Ductus Arteriosus which presents with rib notching on X-ray. The Left vagus nerve "recurrs" at the aortic arch to become the left recurrent laryngeal nerve, the Ductus Arteriosus is in proximity to this structure. The right vagus nerve "recurrs" to become the right recurrent at the right subclavian artery.

JOSE CARLO MASANGKAY III, MD (TOP 8 - FEB 2014 MED BOARDS; TOPNOTCH MD)

DIAGNOSTIC EXAM - AUG 2014

411

A male patient was rushed to your trauma center after a massive bleeding secondary to a self-inflicted deep incised wound to the radial aspect of the left wrist, after a recent break-up with his girlfriend. Which of the following structures is most likely preserved? A. median nerve B. flexor carpi radialis tendon C. palmaris longus tendon D. flexor carpi ulnaris tendon E. none of the above

Flexor carpi ulnaris tendon can be transected in a deep laceration of the wrist in the ulnar aspect.

JOSE CARLO MASANGKAY III, MD (TOP 8 - FEB 2014 MED BOARDS; TOPNOTCH MD)

DIAGNOSTIC EXAM - AUG 2014

412

A prison inmate was rushed to your emergency department due to an apparent stab wound at the flank on the left midscapular line at the level of L2 which of the following structures may be least severed? A. Renal vein B. Renal artery C. Renal pelvis D. Psoas Muscle E. Quadratus Lumborum muscle

from anterior to posterior: Renal vein, Renal artery, renal pelvis.

JOSE CARLO MASANGKAY III, MD (TOP 8 - FEB 2014 MED BOARDS; TOPNOTCH MD)

DIAGNOSTIC EXAM - AUG 2014

TOPNOTCH MEDICAL BOARD PREP ANATOMY SUPEREXAM Page 53 of 94 For inquiries visit www.topnotchboardprep.com.ph or email us at [email protected]

DIAGNOSTIC EXAM - AUG 2014

TOPNOTCH MEDICAL BOARD PREP ANATOMY SUPEREXAM For inquiries visit www.topnotchboardprep.com.ph or email us at [email protected] Item # 413

QUESTION

EXPLANATION

AUTHOR

TOPNOTCH EXAM DIAGNOSTIC EXAM - AUG 2014

An opera singer underwent a Near Total Thyroidectomy due to the discovery of an apparent Papillary Thyroid Carcinoma, post-surgery, patient was unable to reach high notes, what could have been the reason? A. Injury to the internal branch of the superior laryngeal nerve B. Unilateral injury to the recurrent laryngeal nerve C. Bilateral injury to the recurrent laryngeal nerve D. Injury to the external branch of the superior laryngeal nerve E. Injury to the rima glottidis

Injury to the external branch of the superior laryngeal nerve causes paralysis of the cricothyroid muscle which tenses and stretches the vocal cords enabling a person to reach high notes.

JOSE CARLO MASANGKAY III, MD (TOP 8 - FEB 2014 MED BOARDS; TOPNOTCH MD)

414

Which of the following is the major blood supply of the parathyroid glands? A. Superior Thyroid Artery B. Inferior Thyroid Artery C. Superior Parathyroid artery D. Inferior Parathyroid Artery E. Middle Thyroid Artery

The Inferior Parathyroid artery supplies both the superior and inferior parathyroid glands

JOSE CARLO MASANGKAY III, MD (TOP 8 - FEB 2014 MED BOARDS; TOPNOTCH MD)

DIAGNOSTIC EXAM - AUG 2014

415

Which of the following cells is responsible for Osteoid formation? A. Osteocyte B. Osteoblast C. Osteoclast D. Langhan's Cell E. Langerhan's Cell

SIMILAR TO PREVIOUS BOARD EXAM CONCEPT/PRINCIPLE

JOSE CARLO MASANGKAY III, MD (TOP 8 - FEB 2014 MED BOARDS; TOPNOTCH MD)

DIAGNOSTIC EXAM - AUG 2014

416

After a prolonged intubation of a COPD patient you decided to perform a tracheostomy to lessen the Dead Space and improve ventilation, on what location are you going to perform the tracheostomy? A. at the level of C4 vertebra B. in between the thyroid and cricoid cartilage C. above the thyoid cartilage D. at the 1st to the 4th tracheal rings E. at the 2nd to the 4th tracheal rings

Tracheostomy is performed at the level of the 2nd and 4th tracheal rings.

JOSE CARLO MASANGKAY III, MD (TOP 8 - FEB 2014 MED BOARDS; TOPNOTCH MD)

DIAGNOSTIC EXAM - AUG 2014

417

After an automobile accident you realized that the occulomotor nerve has been severed due to what manifestation of the patient? A. Pinpoint pupils B. Droopy eyelids C. Weakness looking down D. Loss of Corneal reflex E. Horizontal diplopia

Severance of the Occulomotor nerve causes diplopia, loss of parallel gaze, fixed and dilated pupil, loss of light reflex, loss of accomodation, Ptosis (Droopy eyelids) (SIMILAR TO PREVIOUS BOARD EXAM CONCEPT/PRINCIPLE)

JOSE CARLO MASANGKAY III, MD (TOP 8 - FEB 2014 MED BOARDS; TOPNOTCH MD)

DIAGNOSTIC EXAM - AUG 2014

418

The melanocytes are seen in which layer of the epidermis? A. S. Corneum B. S. Lucidum C. S. Granulosum D. S. Spinosum E. S. Basale

Melanocytes are seen in the basal layer of the epidermis. (SIMILAR TO PREVIOUS BOARD EXAM CONCEPT/PRINCIPLE)

DIAGNOSTIC EXAM - AUG 2014

419

The oxyphil cells are found in which human structure? A. Pineal Gland B. Parathyroid Gland C. Anterior Pituitary Gland D. Posterior Pituitary Gland E. Thyroid Gland

The oxyphil cells are seen in the parathyroid glands still with unknown function. (SIMILAR TO PREVIOUS BOARD EXAM CONCEPT/PRINCIPLE)

420

A 35 year-old female mountain climber went under a Modified Radical Mastectomy after being diagnosed with Breast Cancer. Post-op, patient can move all her limbs, can protract her scapula, can laterally and medially rotate her arm, can extend MP joints of all digits and can flex the wrist BUT she can not elevate her trunk. Which nerve may be injured intra-operatively? A. Long Thoracic nerve B. Thoracodorsal Nerve C. Axillary Nerve D. Lateral Pectoral Nerve E. Muculocutaneous Nerve

The thoracodorsal nerve innervates the Latissimus dorsi muscle which elevates the trunk (as if attempting to climb) this may be a result of a surgical procedure involving the axilla.

JOSE CARLO MASANGKAY III, MD (TOP 8 - FEB 2014 MED BOARDS; TOPNOTCH MD) JOSE CARLO MASANGKAY III, MD (TOP 8 - FEB 2014 MED BOARDS; TOPNOTCH MD) JOSE CARLO MASANGKAY III, MD (TOP 8 - FEB 2014 MED BOARDS; TOPNOTCH MD)

TOPNOTCH MEDICAL BOARD PREP ANATOMY SUPEREXAM Page 54 of 94 For inquiries visit www.topnotchboardprep.com.ph or email us at [email protected]

DIAGNOSTIC EXAM - AUG 2014

DIAGNOSTIC EXAM - AUG 2014

TOPNOTCH MEDICAL BOARD PREP ANATOMY SUPEREXAM For inquiries visit www.topnotchboardprep.com.ph or email us at [email protected] Item # 421

QUESTION The major blood supply to the parathyroid glands comes from: A. Superior thyroid artery B. Inferior thyroid artery C. Superior parathyroid artery D. Inferior parathyroid artery

EXPLANATION

AUTHOR

TOPNOTCH EXAM MIDTERM 1 EXAM - AUG 2014

There is much controversy to this question. But we have to choose, again, the BEST possible answer. Most anatomy books will say that the inferior thyroid artery supplies the parathyroids, but according to Snell 7th edition, the blood comes from both superior and inferior thyroid arteries. However, we deem that the inferior thyroid artery is more consistent, and hence the better answer. Nonetheless, of note, in a study done by Nobori, et al (PubMed) they have found that 45% of their subjects had a distinct anastomosing branch between the superior and inferior thyroid arteries, supplying also the superior parathyroids. The cavernous sinuses are situated in the middle cranial fossa on each side of the body of sphenoid. They extend from the superior orbital fissure in front, to the apex of the petrous part of the temporal bone behind. Found within these sinuses are the internal carotid artery, CN III and IV, and the ophthalmic and maxillary divisions of CN V. They drain posteriorly into the superior and inferior petrosal sinuses and inferiorly into the pterygoid venous plexus. They communicate with the facial vein through the superior ophthalmic vein.

WEBSTER ALINDOG, MD (TOP 3 - FEB 2014 MED BOARDS; TOPNOTCH MD)

WEBSTER ALINDOG, MD (TOP 3 - FEB 2014 MED BOARDS; TOPNOTCH MD)

MIDTERM 1 EXAM - AUG 2014

WEBSTER ALINDOG, MD (TOP 3 - FEB 2014 MED BOARDS; TOPNOTCH MD) WEBSTER ALINDOG, MD (TOP 3 - FEB 2014 MED BOARDS; TOPNOTCH MD)

MIDTERM 1 EXAM - AUG 2014

WEBSTER ALINDOG, MD (TOP 3 - FEB 2014 MED BOARDS; TOPNOTCH MD)

MIDTERM 1 EXAM - AUG 2014

422

A 25-year old male patient was observed to have recurrent pyogenic infections of the nasal sinuses. He later on developed cavernous sinus thrombosis, a potentially serious condition because of the many delicate structures that can be affected. Which of the following is least likely to be involved? A. CN II B. CN III C. CN IV D. Internal carotid artery E. None of the above.

423

Sounds produced by the aortic valve can be best heard at: A. 5th ICS, left midclavicular line B. 2nd ICS, right parasternal C. 2nd ICS, left parasternal D. Right half of the lower end of the body of sternum

Study also some surface anatomy. Pulmonic valve is best heard at 2nd ICS, left parasternal; mitral valve at 5th ICS, left MCL; and tricuspid valve at the right half of the lower end of the body of sternum.

424

A perforating ulcer located at the lesser curvature of the stomach will most likely erode which branch of the celiac artery? A. Splenic artery B. Gastroduodenal artery C. Left gastric artery D. Left gastroepiploic artery

425

The cell organelle containing oxidative enzymes, synthesizing H2O2 and is involved in beta-oxidation of long chain fatty acids: A. Lysosome B. Smooth ER C. Mitochondria D. Peroxisome

Gastric ulcers usually occur within the body of the stomach along the lesser curvature above the incisura angularis. If it perforates, it can erode the left gastric artery (reviewing the blood supply of the stomach: Lesser curvature --- right and left gastric arteries; greater curvature --- right and left gastroepiploic arteries; fundus --- short gastric artery). Both the left gastric artery and the splenic artery are branches of the celiac artery. Gastroduodenal artery comes from the hepatic artery whereas the left gastroepiploic artery originates from the gastroduodenal artery.

426

The most common site of aortic aneurysm: A. Distal to the portion of aorta passing through the diaphragmatic opening B. Below the origin of the renal arteries C. Along the portion contained within the posterior mediastinum D. Just distal to the bifurcation of the common iliacs

Aneurysms are dilatations of the aorta usually resulting from atherosclerosis, which causes arterial wall weakening.

WEBSTER ALINDOG, MD (TOP 3 - FEB 2014 MED BOARDS; TOPNOTCH MD)

MIDTERM 1 EXAM - AUG 2014

427

All of the following muscles are important in plantar flexing the foot at the ankle joint except for: A. Gastrocnemius B. Popliteus C. Soleus D. Plantaris

Gastrocnemius, soleus, and plantaris together serve as a powerful plantar flexor of the ankle joint. They provide the main propulsive force in walking and running. They are supplied by the tibial nerve, with nerve root coming from S1 and S2.

WEBSTER ALINDOG, MD (TOP 3 - FEB 2014 MED BOARDS; TOPNOTCH MD)

MIDTERM 1 EXAM - AUG 2014

TOPNOTCH MEDICAL BOARD PREP ANATOMY SUPEREXAM Page 55 of 94 For inquiries visit www.topnotchboardprep.com.ph or email us at [email protected]

MIDTERM 1 EXAM - AUG 2014

TOPNOTCH MEDICAL BOARD PREP ANATOMY SUPEREXAM For inquiries visit www.topnotchboardprep.com.ph or email us at [email protected] Item # 428

QUESTION

EXPLANATION

AUTHOR

TOPNOTCH EXAM MIDTERM 1 EXAM - AUG 2014

A 22-year old frisbee player was brought to the hospital after sustaining a knee injury. On physical exam, he was noted to demonstrate a positive anterior drawer sign. This suggests that: A. There is injury to the knee ligament that is attached to the lateral condyle of the femur above and to the head of the fibula below. B. There is injury to the knee ligament that is attached to the medial condyle of the femur above and to the medial aspect of the tibial shaft below. C. There is injury to the knee ligament that prevents anterior displacement of the femur on the tibia. D. There is injury to the knee ligament that prevents posterior displacement of the femur on the tibia.

The anterior cruciate ligament prevents the posterior displacement of femur on tibia, or conversely, prevents the anterior displacement of tibia on femur (anterior drawer test). Option A refers to the lateral collateral ligament, which is also more flexible and hence less susceptible to injury compared with the medial collateral ligament (option B). Option C simply refers to the posterior cruciate ligament, whose stability is tested by the posterior drawer test.

WEBSTER ALINDOG, MD (TOP 3 - FEB 2014 MED BOARDS; TOPNOTCH MD)

429

Which of the following is not true about the mucous membrane of the lower half of the anal canal? A. It is sensitive to pain, temperature, touch and pressure (somatic sensory innervation). B. It is thrown into vertical mucous folds called anal columns. C. The lymph drains downward to the medial group of superficial inguinal nodes. D. Its blood supply comes directly from the inferior rectal artery, a branch of the internal pudendal artery. E. None of these.

Anal columns are found only in the mucous membrane of the UPPER half of the anal canal. The rest of the options are true about the lower anal canal. It is the pectinate line that indicates the level where the two halves join together.

WEBSTER ALINDOG, MD (TOP 3 - FEB 2014 MED BOARDS; TOPNOTCH MD)

MIDTERM 1 EXAM - AUG 2014

430

A normal individual will usually have how many parathyroid glands? A. 2 B. 4 C. 6 D. 7

It is a "mortal sin" to get wrong in "give-away" questions like this (chances are, everybody will answer them correctly so please always read CAREFULLY even those questions with obvious answers). Parathyroid glands can be as numerous as 7 in a normal individual.

WEBSTER ALINDOG, MD (TOP 3 - FEB 2014 MED BOARDS; TOPNOTCH MD)

MIDTERM 1 EXAM - AUG 2014

431

Venous drainage of the posterior ventricular walls including the posterior interventricular septum drain directly to: A. Great cardiac vein B. Smallest cardiac vein C. Coronary sinus D. Middle cardiac vein



WEBSTER ALINDOG, MD (TOP 3 - FEB 2014 MED BOARDS; TOPNOTCH MD)

MIDTERM 1 EXAM - AUG 2014

432

A 46-year old male was rushed to the ER after getting involved in a vehicular accident. An anterior abdominal blunt injury was suspected. FAST was performed with focus on 4 areas including the so-called Morrison's pouch. As member of the trauma team you know that this area corresponds to the: A. Splenorenal space B. Suprapubic space C. Hepatorenal space D. Subxiphoid area

Focused assessment with sonography in trauma (FAST) is a quick, reliable, non-invasive procedure that can facilitate a timely diagnosis for patients with blunt abdominal trauma. It includes views of (1) the hepatorenal recess (Morison pouch), (2) the perisplenic view, (3) the subxiphoid pericardial window, and (4) the suprapubic window (Douglas pouch).

WEBSTER ALINDOG, MD (TOP 3 - FEB 2014 MED BOARDS; TOPNOTCH MD)

MIDTERM 1 EXAM - AUG 2014

433

This is one of the five terminal nerves of the brachial plexus which is also the motor innervation of the anterior compartment of the arm, important in flexing the elbow joint and supinating the forearm: A. Musculocutaneous nerve B. Median nerve C. Radial nerve D. Ulnar nerve

WEBSTER ALINDOG, MD (TOP 3 - FEB 2014 MED BOARDS; TOPNOTCH MD)

MIDTERM 1 EXAM - AUG 2014

434

True of utricle and saccule except: A. Housed by a central cavity known as vestibule B. Kinetic labyrinth C. Important for linear acceleration D. Contain otoliths E. None of the above

Must master the brachial (and lumbar) plexus. It is formed in the posterior triangle of the neck by the union of the anterior rami of C5 to C8 and T1. It can be divided into roots, trunks, divisions and cords. It has 5 terminal branches namely, the musculocutaneous nerve (innervating the anterior/flexor compartment of the arm); the axillary nerve (deltoid and teres minor); the radial nerve (posterior/extensor compartment of arm and forearm); the median nerve (anterior/flexor compartment of the forearm except flexor carpi ulnaris and flexor digitorum profundus, medial half); and the ulnar nerve (intrinsic muscles of the hand). The utricle and saccule constitute the static labyrinth which functions during the linear acceleration of the head and with the effects of the pull of gravity. On the other hand, the semicircular canals make up the kinetic labyrinth which is important in angular deceleration and acceleration; it has no otoliths.

WEBSTER ALINDOG, MD (TOP 3 - FEB 2014 MED BOARDS; TOPNOTCH MD)

MIDTERM 1 EXAM - AUG 2014

TOPNOTCH MEDICAL BOARD PREP ANATOMY SUPEREXAM Page 56 of 94 For inquiries visit www.topnotchboardprep.com.ph or email us at [email protected]

TOPNOTCH MEDICAL BOARD PREP ANATOMY SUPEREXAM For inquiries visit www.topnotchboardprep.com.ph or email us at [email protected] Item # 435

QUESTION Which of the following statements about seminal vesicle is not true? A. They are lobulated structures that can store spermatozoa. B. They produce secretions added to the seminal fluid and important in sperm nourishment. C. Their blood supply comes from the superior vesicle and superior rectal arteries. D. They can be palpated during digital rectal exam. E. All of the above.

EXPLANATION

AUTHOR

TOPNOTCH EXAM MIDTERM 1 EXAM - AUG 2014

It is the epididymis that can store mature spermatozoa. Structures that can be palpated during DRE: rectovesical pouch, full bladder, seminal vesicles, displaced or enlarged ductus deferentes, membranous part of urethra when catheterized, and bulbo-urethral glands; ischial tuberosity and spine and sacrotuberous ligament; pelvic surface of sacrum and coccyx. In females, vagina, cervix, ostium uteri, body of uterus when retroverted, recto-uterine fossa, and, pathologically, broad ligaments, uterine tubes, and ovaries. Blood supply of the seminal vesicle comes from the inferior vesicle and middle rectal arteries. This condition corresponds to a distal lesion of the median nerve (true also in carpal tunnel syndrome) in which there is an altered cutaneous sensation on the lateral 3 and 1/2 digits of the hand; a patient may also exhibit weakness in opposition of the thumb which remains adducted and extended as a result, so called "ape" hand. A proximal lesion of the median hand meanwhile, results from supracondylar fracture of the humerus or from compression between the pronator teres leading to the the condition "hand of benediction" - with the index and middle fingers remained extended when attempting to flex.

WEBSTER ALINDOG, MD (TOP 3 - FEB 2014 MED BOARDS; TOPNOTCH MD)

WEBSTER ALINDOG, MD (TOP 3 - FEB 2014 MED BOARDS; TOPNOTCH MD)

MIDTERM 1 EXAM - AUG 2014

436

A 15-year old skateboarder lost balance and fell on his outstretched right hand causing hyperextension of the wrist joint . He then manifest numbness and pain over the palmar aspects of his right thumb, index and middle fingers. Which of the following mechanisms of injury will most likely explain the symptoms? A. A fracture of the base of the metacarpal bones B. A supracondylar fracture of the humerus with compression of the pronator teres muscle C. An oblique fracture of the trapezium bone D. A dislocated lunate bone

437

The inferior thyroid vein which receives tributaries from the lower poles of the gland and the isthmus drains directly to: A. Internal jugular vein B. Brachiocephalic vein C. External jugular vein D. Superior vena cava



WEBSTER ALINDOG, MD (TOP 3 - FEB 2014 MED BOARDS; TOPNOTCH MD)

MIDTERM 1 EXAM - AUG 2014

438

Melanocytes, the pigment-producing cells, are most numerous in this layer of the skin: A. Stratum lucidum B. Stratum granulosum C. Stratum corneum D. Stratum basale

Buzz words: stratum corneum - dead keratinized cells, protection; stratum lucidum - only found in thick skin (palm and soles); stratum granulosum - keratohyaline granules; stratum spinosum - Langerhan cells, SCCA; stratum basale - melanocytes, Merkel cells, basal cell CA.

WEBSTER ALINDOG, MD (TOP 3 - FEB 2014 MED BOARDS; TOPNOTCH MD)

MIDTERM 1 EXAM - AUG 2014

439

Which of the following anatomic relationships will best locate the gallbladder? A. It is medial to the ligamentum teres within the falciform ligament. B. It lies superior to the porta hepatis C. It is lateral to the quadrate lobe of the liver. D. 2/3 of which lies in the left lobe of the liver.



WEBSTER ALINDOG, MD (TOP 3 - FEB 2014 MED BOARDS; TOPNOTCH MD)

MIDTERM 1 EXAM - AUG 2014

440

In performing the ECG, the V5 lead should be placed over the: A. 2nd right ICS, parasternal area B. 4th left ICS, parasternal area C. 4th left ICS, midclavicular area D. 5th left ICS, lateral to the midclavicular area

Must know: V1 - 4th ICS, right parasternal area; V2 - 4th ICS, left para sternal area; V3 - space between V2 and V4; V4 - 5th ICS, left MCL; V5 - 5th ICS, left AAL; V6 - 5th ICS, left MAL.

MIDTERM 1 EXAM - AUG 2014

441

A 45 year-old male painter went up to a ladder to paint the roof, however, he slips and fell suffering a straddle injury. Which of the following structure prevents the spread of urine inferiorly to the thigh in case of ruptured penile urethra: A. rectus sheath B. camper's fascia C. scarpa's fascia D. denonvilier's fascia E. conjoint tendon

Scarpa's or the membranous layer of superficial fascia forms a tubular sheath over the penis and scrotum in males which prevents extravasation of urine inferiorly in cases of urethral trauma. In the male, Camper’s fascia is continued over the penis and outer surface of the spermatic cord to the scrotum, where it helps to form the dartos. This layer is sufficiently complete that fluids escaping from a ruptured vessel or urethra (blood and/or urine) may accumulate deep to it.

WEBSTER ALINDOG, MD (TOP 3 - FEB 2014 MED BOARDS; TOPNOTCH MD) JULIET KRISTINE EVANGELIST A, MD (TOP 9 - FEB 2014 MED BOARDS; TOPNOTCH MD)

442

Which is not included in the drainage of the thyroid: A. Superior thyroid vein B. Middle thyroid vein C. Inferior thyroid vein D. Brachiocephalic E. Thyroidea ima

The main drainage of the thyroid gland are superior thyroid vein and middle thyroid vein draining into internal jugular vein. The inferior thyroid vein which drains isthmus and lower poles of the gland which drains into left brachiocephalic vein in the thorax. Thyroidea ima may arise from the brachiocephalic artery or the arch of the aorta.

JULIET KRISTINE EVANGELIST A, MD (TOP 9 - FEB 2014 MED BOARDS; TOPNOTCH MD)

MIDTERM 2 EXAM - AUG 2014

TOPNOTCH MEDICAL BOARD PREP ANATOMY SUPEREXAM Page 57 of 94 For inquiries visit www.topnotchboardprep.com.ph or email us at [email protected]

MIDTERM 2 EXAM - AUG 2014

TOPNOTCH MEDICAL BOARD PREP ANATOMY SUPEREXAM For inquiries visit www.topnotchboardprep.com.ph or email us at [email protected] Item # 443

QUESTION The non-ciliated secretory cells which randomly interrupt the ciliated epithelial lining of the bronchial mucosa are called: A. Kulchitsky cells B. Clara cells C. Neuroendocrine cells D. Type 2 pneumocytes E. Dust cells

EXPLANATION

AUTHOR

TOPNOTCH EXAM MIDTERM 2 EXAM - AUG 2014

Clara cells are nonciliated, secretory bronchiolar epithelial cells which function as stem cells for repair in the bronchioles and can divide into ciliated or nonciliated bronchiolar cells. Enterochromaffin cells, or Kulchitsky cells, are a type of enteroendocrine and neuroendocrine cell occurring in the epithelial lining the lumen of the digestive tract and the respiratory tract. Type 2 pneumocytes are cells secreting surfactant in the alveoli. Dust cells or alveolar macrophage are in the lungs that reside on respiratory surfaces and clean off particles such as dust or microorganisms. The porous, fragile nature of the ethmoid bone makes it particularly susceptible to fractures. The ethmoid is usually fractured from an upward force to the nose. This could occur by hitting the dashboard in a car crash or landing on the ground after a fall. The ethmoid fracture can produce bone fragments that penetrate the cribriform plate. This trauma can lead to a leak of cerebral spinal fluid into the nasal cavity.

JULIET KRISTINE EVANGELIST A, MD (TOP 9 - FEB 2014 MED BOARDS; TOPNOTCH MD)

JULIET KRISTINE EVANGELIST A, MD (TOP 9 - FEB 2014 MED BOARDS; TOPNOTCH MD)

MIDTERM 2 EXAM - AUG 2014

444

A 52 year-old man was rushed to the ER after a car accident. He was noted to have clear fluid draining from his nose which is apparently the CSF. The bone which is most likely fractured is the: A. nasal B. ethmoid C. frontal D. lacrimal E. zygomatic

445

Which chamber of the heart is most likely enlarged when there is narrowing of the thoracic esophagus on Barium swallow: A. right atrium B. right ventricle C. left atrium D. left ventricle E. left auricle

Left Atrial Enlargement pushes the esophagus toward the spine (as seen on barium swallow) and is the most sensitive indicator of LAE.

JULIET KRISTINE EVANGELIST A, MD (TOP 9 - FEB 2014 MED BOARDS; TOPNOTCH MD)

MIDTERM 2 EXAM - AUG 2014

446

A 23 year-old male patient seen at a clinic cannot focus on near objects but he can move his eyeball normally and see distant objects clearly. This condition may indicate damage to: A. Short ciliary nerve and ciliary ganglion B. Ciliary ganglion and oculomotor nerve C. Oculomotor nerve and long ciliary nerve D. Short and long ciliary nerves E. Long ciliary nerve and superior cervical ganglion

The ciliary ganglion is a parasympathetic ganglion located in the posterior orbit on the lateral side of the optic nerve. The postganglionic axons run in the short ciliary nerves and innervate two eye muscles including the sphincter pupillae which constricts the pupil, and the ciliary muscle which contracts, releasing tension on the Zonular Fibers, and making the lens more convex, resulting to accommodation.

JULIET KRISTINE EVANGELIST A, MD (TOP 9 - FEB 2014 MED BOARDS; TOPNOTCH MD)

MIDTERM 2 EXAM - AUG 2014

447

A 48 year-old woman was diagnosed of Phyllodes tumor. She underwent mastectomy. However, postoperatively, the woman experiences weakness in the ability to protract the scapula with difficulty raising the arm above her head. Damage to which of the following nerves is suggested by this finding: A. Axillary B. Supraclavicular C. Spinal accessory D. Long Thoracic E. Thoracodorsal

Long thoracic nerve supplies serratus anterior. The nerve is most commonly injured as it courses superficial to the serratus anterior causing weakness in the ability to protract the scapula and difficulty in raising the arm above the head. It also causes winging of the scapula.

JULIET KRISTINE EVANGELIST A, MD (TOP 9 - FEB 2014 MED BOARDS; TOPNOTCH MD)

MIDTERM 2 EXAM - AUG 2014

448

A 55 year-old male sustained from a motor vehicular accident resulting to fracture of the 9th to 11th ribs left posterior. BP is 80/50mmHg, PR=125bpm, RR=22cpm. The most likely injured organ is the A. stomach B. pancreas C. liver D. spleen E. small intestine

Although protected under the bony ribcage, the spleen remains the most commonly affected organ in blunt injury to the abdomen in all age groups. While some references occasionally document liver injuries as being more common, blunt injuries to the spleen are documented more frequently as the primary solid organ injury in the abdomen. These injuries are common as a result from motor vehicle crashes, domestic violence, sporting events, and accidents involving bicycle handlebars.

JULIET KRISTINE EVANGELIST A, MD (TOP 9 - FEB 2014 MED BOARDS; TOPNOTCH MD)

MIDTERM 2 EXAM - AUG 2014

449

Massive hemopericardium compromises cardiac and systemic circulation. In emergency evacuation of the blood, the best and safest way to make an incision en route to the pericardial activity is via: A. 5th left ICS lateral to sternum B. 7th left ICS immediately lateral to sternum C. 6th left ICS 8cm from misdternal line D. 7th left ICS 8cm from midsternal line E. immediately subxiphoidal

Cardiac tamponade is a life-threatening condition that requires prompt diagnosis and management. Anatomically, pericardiocentesis is carried out immediately under the xiphoid process (infrasternal), up and leftwards.

JULIET KRISTINE EVANGELIST A, MD (TOP 9 - FEB 2014 MED BOARDS; TOPNOTCH MD)

MIDTERM 2 EXAM - AUG 2014

TOPNOTCH MEDICAL BOARD PREP ANATOMY SUPEREXAM Page 58 of 94 For inquiries visit www.topnotchboardprep.com.ph or email us at [email protected]

TOPNOTCH MEDICAL BOARD PREP ANATOMY SUPEREXAM For inquiries visit www.topnotchboardprep.com.ph or email us at [email protected] Item # 450

QUESTION

EXPLANATION

AUTHOR

TOPNOTCH EXAM MIDTERM 2 EXAM - AUG 2014

A 65 year-old female, hypertensive, diabetic suddenly developed dizziness and subsequently loss consciousness. She was rushed to the emergency room, she regained consciousness. She cannot understand or obey commands, she talks and answers to questions irrelevantly. If an infarct is suspected, what Brodmann Area is affected: A. Brodmann Area 4 B. Brodmann Area 44 and 45 C. Brodmann Area 8 D. Brodmann Area 21, 42 E. Brodmann Area 3,1,2

The lesion is located in Wernicke's area, which is the posterior region of the left superior temporal gyrus or the first gyrus of the temporal lobe. Brodmann's areas 21 and 42 correspond to Wernicke's area. With Wernicke's aphasia there is usually a severe impairment in auditory comprehension. Speech, while fluent, is semantically inappropriate and paraphasic. Comprehension and expression tend to be equally impaired.

JULIET KRISTINE EVANGELIST A, MD (TOP 9 - FEB 2014 MED BOARDS; TOPNOTCH MD)

451

In portal hypertension, which of the following veins will not engorge and is not a collateral circulation: A. Superior rectal B. Median sacral C. Esophageal D. Paraumbilical E. Splenic

The tributaries of portal vein are the splenic vein, inferior mesenteric vein, superior mesenteric vein, left gastric vein, right gastric vein, and cystic veins. Median sacral vein directly drains to left common iliac vein then to inferior vena cava.

MIDTERM 2 EXAM - AUG 2014

452

During a dilatation and curettage in a 23 year-old G1P0 patient after an incomplete abortion at 5 weeks AOG, the instrument accidentally punctured the uterus anteriorly hitting this structure: A. Urinary bladder B. Uterine artery C. Sigmoid colon D. Ureter E. Broad ligament

The body of the uterus is related anteriorly to the uterovesical pouch and the superior surface of the bladder. Perforation is usually caused by a surgical instrument used for scraping and removing material from the uterus. The instrument penetrates through the uterine wall, and rarely, may migrate into the abdominal cavity where the bladder may also be perforated.

JULIET KRISTINE EVANGELIST A, MD (TOP 9 - FEB 2014 MED BOARDS; TOPNOTCH MD) JULIET KRISTINE EVANGELIST A, MD (TOP 9 - FEB 2014 MED BOARDS; TOPNOTCH MD)

453

A 19 year-old male suffered from a stab wound at the anterior abdomen and was rushed to the ER. A segment of the intestine protruded and partially opened. The segment's mucosa had few circular folds. It's mesentery was fatty with many vascular arcades. The segment that is seen protruding the patient's abdomen is: A. duodenal bulb B. distal part of duodenum C. proximal jejunum D. distal jejunum E. distal ileum

The segment that is seen protruding the patient's abdomen is the ileum which is in the lower part of the cavity and in the pelvis. The ileum receives numerous short terminal vessels that form many vascular arcades. At the ileal end of the mesentery, the fat is deposited throughout.

JULIET KRISTINE EVANGELIST A, MD (TOP 9 - FEB 2014 MED BOARDS; TOPNOTCH MD)

MIDTERM 2 EXAM - AUG 2014

454

A 12 year-old boy was brought to the ER because of acute attack of Bronchial Asthma. Symptoms primarily are caused by mucosal inflammation and airway hyperresponsiveness. The specific part of the airway involved in the pathogenesis of Asthma is: A. trachea B. bronchi C. bronchioles D. alveolar ducts E. alveolar sacs

During an asthmatic event the muscles surrounding the bronchioles constrict. The wheezing sound is caused by the contraction of the bronchioles as the air passes through tubes that are almost completely blocked.

JULIET KRISTINE EVANGELIST A, MD (TOP 9 - FEB 2014 MED BOARDS; TOPNOTCH MD)

MIDTERM 2 EXAM - AUG 2014

455

A 48 year-old fish vendor was stabbed by an unknown assailant at the back. He was rushed to the ER with BP 60 palpatory, PR=130s, RR=20s. Which particular structure of the kidney will be stimulated as a result of the above findings: A. juxtaglomerular cells B. cells of PCT C. cells of loop of Henle D. macula densa E. mesangial cells

Macula densa cells sense changes in sodium chloride level, and will trigger an autoregulatory response to increase or decrease reabsorption of ions and water to the blood in order to alter blood volume and return blood pressure to normal. Decreased BP is detected initially by Macula Densa which results to increased renin release from JG cells.

JULIET KRISTINE EVANGELIST A, MD (TOP 9 - FEB 2014 MED BOARDS; TOPNOTCH MD)

MIDTERM 2 EXAM - AUG 2014

456

Direct inguinal hernias are found within the Hesselbach's triangle. The posterior wall of this triangle where hernias protrude is: A. Inguinal ligament B. External oblique aponeurosis C. Transversus abdominis aponeurosis D. Internal oblique aponeurosis E. Transversalis fascia

Direct inguinal hernia protrudes through the inguinal triangle of Hesselbach that lies between the inferior epigastric artery superolaterally, the rectus abdominis medially and the inguinal ligament inferiorly. The posterior wall of this triangle where hernias protrude is formed by the transversalis fascia. The transversalis fascia forms an investing fascial envelope of the abdominal cavity.

JULIET KRISTINE EVANGELIST A, MD (TOP 9 - FEB 2014 MED BOARDS; TOPNOTCH MD)

MIDTERM 2 EXAM - AUG 2014

TOPNOTCH MEDICAL BOARD PREP ANATOMY SUPEREXAM Page 59 of 94 For inquiries visit www.topnotchboardprep.com.ph or email us at [email protected]

MIDTERM 2 EXAM - AUG 2014

TOPNOTCH MEDICAL BOARD PREP ANATOMY SUPEREXAM For inquiries visit www.topnotchboardprep.com.ph or email us at [email protected] Item # 457

QUESTION

EXPLANATION

AUTHOR

TOPNOTCH EXAM MIDTERM 2 EXAM - AUG 2014

A 35 year-old woman has an infected right big toe with swollen lymph nodes. The group of node that is most likely affected is: A. Femoral B. Superficial inguinal C. Deep inguinal D. Internal iliac E. Paravertebral

A patient may present with an enlarged, painful superficial inguinal lymph node that is due to lymphatic spread of pathogenic organisms that entered the body through the legs and big toes. Femoral lymph nodes are ocated in the upper thigh portion along the femoral veins, immediately below the inguinal lymph nodes which drain from some of the genital parts, buttock, thigh and the medial side of the leg.

JULIET KRISTINE EVANGELIST A, MD (TOP 9 - FEB 2014 MED BOARDS; TOPNOTCH MD)

458

A 46 year-old woman had crow's feet at lateral angles of her eyelids. These are due to contraction of: A. Procerus B. Corrugator supercilli C. Frontalis D. Orbicularis oculi E. Levator palpebrae superioris

JULIET KRISTINE EVANGELIST A, MD (TOP 9 - FEB 2014 MED BOARDS; TOPNOTCH MD)

MIDTERM 2 EXAM - AUG 2014

459

A 35 year-old man complains of inability to flex the elbow joint. It involves a nerve supplying the muscle which branches from the brachial plexus as: A. C5 B. C5-C6 C. C5-C7 D. C6, C7, C8 E. C8-T1

Contraction of the orbicularis oculi muscle is primarily responsible for the clinically observed periorbital crow's feet. The orbicularis oculi muscle is bordered superolaterally by fibers of the frontalis muscle and medially by the levator palpebrae muscle. The frontalis muscle and the corrugator muscle are responsible for the appearance of wrinkles and vertical frown lines accordingly. The procerus muscle, which overlies the nasal root, is responsible for a snout-nose appearance and horizontal frown lines. Musculocutaneous nerve supplies the flexor of the arm which branches from the brachial plexus as C5-C7. C5- Dorsal scapular Nerve, C5-C6suprascapular nerve, C6,C7,C8-thoracodorsal nerve, C8-T1-Ulnar nerve and medial root of median.

JULIET KRISTINE EVANGELIST A, MD (TOP 9 - FEB 2014 MED BOARDS; TOPNOTCH MD)

MIDTERM 2 EXAM - AUG 2014

460

Pulsation from an artery which is a continuation of anterior tibial artery on the dorsum of the foot can be easily felt. On the medial side of this artery lies the tendon of: A. Extensor digitorum brevis B. Extensor digitorum longus C. Extensor hallusis brevis D. Extensor hallucis longus E. Inferior extensor retinaculum

Dorsalis pedis artery is a continuation of anterior tibial artery. Medial to this lies the tendon of extensor hallucis longus. On the lateral side lies the terminal part of the deep peroneal nerve and extensor digitorum longus tendons.

JULIET KRISTINE EVANGELIST A, MD (TOP 9 - FEB 2014 MED BOARDS; TOPNOTCH MD)

MIDTERM 2 EXAM - AUG 2014

461

What is the vein that is used in arterio-venous fistula that is located lateral to the biceps brachii? A. Basilic vein B. brachial vein C. Median cubital vein D. Cephalic vein E. Axillary vein

cephalic vein is lateral to the biceps brachii, while basilic vein is medial to the biceps brachii.

LUISA SARANILLO, MD (TOP 6 - FEB 2014 MED BOARDS; TOPNOTCH MD)

BACK-UP MIDTERM EXAM AUG 2014 - FOR INCLUSION IN THE SAMPLEX

462

In the standard 12 lead ECG what is the anatomic placement of the chest electrode lead V2? A. 4th ICS Right parasternal border B. 4th ICS Left parasternal border C. 5th ICS Right MCL D. 5th ICS Left MCL E. None of the choices

4th ICS Right parasternal border = V1: 4th ICS Left parasternal border = V2; 5th ICS Left MCL = V4; 5th ICS Right MCL= none

LUISA SARANILLO, MD (TOP 6 - FEB 2014 MED BOARDS; TOPNOTCH MD)

BACK-UP MIDTERM EXAM AUG 2014 - FOR INCLUSION IN THE SAMPLEX

463

A 55 year old male presents with chest pain. ECG shows ST segment elevation in lead II, III, AVF. What is the most likely affected blood vessel supplying the affected part of the heart? A. Right coronary artery B. Left marginal artery C. left anterior descending artery D. Left circumflex artery E. C and D

Leads II, III, AVF refers the inferior wall, which is supplied by the Right coronary artery. Septal, anterior, and lateral wall are supplied by the left coronary artery.

LUISA SARANILLO, MD (TOP 6 - FEB 2014 MED BOARDS; TOPNOTCH MD)

BACK-UP MIDTERM EXAM AUG 2014 - FOR INCLUSION IN THE SAMPLEX

464

What is the most common type of myoma? A. Submucosal myoma B. Pedunculated subserosal myoma C. Subserosal myoma D. Intramural myoma E. Pedunculated submucosal myoma

Intramural is the most common type, while submucosal is the one frequently associated with heavy, prolonged bleeding.

BACK-UP MIDTERM EXAM AUG 2014 - FOR INCLUSION IN THE SAMPLEX

465

The following are muscles of inspiration except: A. Serratus posterior superior B. External intercostal C. Internal intercostal D. Innermost intercostal E. Subcostal

Internal intercostal depresses the ribs during exhalation. Other choices elevate the ribs during inspiration.

LUISA SARANILLO, MD (TOP 6 - FEB 2014 MED BOARDS; TOPNOTCH MD) LUISA SARANILLO, MD (TOP 6 - FEB 2014 MED BOARDS; TOPNOTCH MD)

TOPNOTCH MEDICAL BOARD PREP ANATOMY SUPEREXAM Page 60 of 94 For inquiries visit www.topnotchboardprep.com.ph or email us at [email protected]

BACK-UP MIDTERM EXAM AUG 2014 - FOR INCLUSION IN THE SAMPLEX

TOPNOTCH MEDICAL BOARD PREP ANATOMY SUPEREXAM For inquiries visit www.topnotchboardprep.com.ph or email us at [email protected] Item # 466

QUESTION

EXPLANATION

AUTHOR

TOPNOTCH EXAM BACK-UP MIDTERM EXAM AUG 2014 - FOR INCLUSION IN THE SAMPLEX

A 25 year old male came in due a stab wound in his abdomen. Upon exploration you noticed a severed blood vessel supplying the lower part of the the muscular tube from the kidney to the posterior surface of the bladder. What blood vessel is most likely involved? A. renal artery B. testicular artery C. superior vesical artery D. iliolumbar artery E. inferior vesical artery

The muscular tube from the kidney to the posterior surface of the bladder is the ureter. It has 3 blood supplies: upper part is supplied by renal artery, the middle part is supplied by the gonadal artery, the lower part is supplied by the superior vesical artery.

LUISA SARANILLO, MD (TOP 6 - FEB 2014 MED BOARDS; TOPNOTCH MD)

467

A 65 year old female fall from a height of approximately 10 step ladder sustaining injuries. Her right leg is shortened and the thigh is laterally rotated. What is the most likely diagnosis. A. Fracture of the neck of the femur, right B. fracture of the tibia, right C. dislocation of the head of the femur, right D. fracture of the fibula, right E. none of the choices

Fracture of the femoral neck will present with shortened leg and laterally rotated thigh while dislocation of the femoral head will present with shortened leg and medially rotated thigh.

LUISA SARANILLO, MD (TOP 6 - FEB 2014 MED BOARDS; TOPNOTCH MD)

BACK-UP MIDTERM EXAM AUG 2014 - FOR INCLUSION IN THE SAMPLEX

468

A surgeon harvested part of the great saphenous vein from the right lower extremity of a 30 year old patient. However, after the procedure, the patient complained of pain and paresthesia in the medial aspect of the leg and foot. What nerve is the most likely affected. A. saphenous nerve B. obturator nerve C. femoral nerve D. sciatic nerve E. A and B

Saphenous nerve supplies the skin of the medial aspect of the leg and foot. Obturator nerve supplies the skin of the medial thigh. Femoral nerve supplies the antero-medial aspect of the thigh. Sciatic nerve supplies the posterior thigh, foot and leg.

LUISA SARANILLO, MD (TOP 6 - FEB 2014 MED BOARDS; TOPNOTCH MD)

BACK-UP MIDTERM EXAM AUG 2014 - FOR INCLUSION IN THE SAMPLEX

469

A 6 month old infant has a long and narrow skull with frontal and occipital expansion was brought in to your clinic by her mother for consultation. You explained to the mother that the most likely reason for this is the premature closure of the cranial sutures. What suture is involved in this case? A. sagittal suture B. coronal suture C. lambdoid suture D. coronal and lambdoid suture E. coronal and sagittal suture

Premature closure of the sagittal suture called scaphocephaly presents with long and narrow skull with frontal and occipital expansion; involvement of the coronal suture forms a tower skull - a short high skull; premature closure of the coronal and lambdoid suture on one side of the skull called plagiocephaly presents with assymetric skull.

LUISA SARANILLO, MD (TOP 6 - FEB 2014 MED BOARDS; TOPNOTCH MD)

BACK-UP MIDTERM EXAM AUG 2014 - FOR INCLUSION IN THE SAMPLEX

470

The following cranial nerves are both motor and sensory except: A. Glossopharyngeal B. Vagus C. facial D. vestibulocochlear E. No exception

Mixed cranial nerves are vagus, glossopharyngeal, facial, and trigeminal nerve. Vestibulocochlear nerve is purely sensory nerve.

LUISA SARANILLO, MD (TOP 6 - FEB 2014 MED BOARDS; TOPNOTCH MD)

BACK-UP MIDTERM EXAM AUG 2014 - FOR INCLUSION IN THE SAMPLEX

471

A 35 year old male sustained facial laceration due to vehicular accident. The wound tend to gape because: A. The face has no deep fascia B. The subcutaneous tissue of the face is loose C. The face has weak muscles D. A and B only E. All of the choices

Facial lacerations tend to gape because the face has no deep fascia and the subcutaneous tissue is loose.

LUISA SARANILLO, MD (TOP 6 - FEB 2014 MED BOARDS; TOPNOTCH MD)

BACK-UP MIDTERM EXAM AUG 2014 - FOR INCLUSION IN THE SAMPLEX

472

The anal canal is divided into 2 parts by the dentate line. What is the lining epithelium of the anal canal above the dentate line? A. Simple columnar epithelium B. Simple cuboidal epithelium C. Stratified squamous epithelium D. transitional epithelium E. simple squamous epithelium



LUISA SARANILLO, MD (TOP 6 - FEB 2014 MED BOARDS; TOPNOTCH MD)

BACK-UP MIDTERM EXAM AUG 2014 - FOR INCLUSION IN THE SAMPLEX

TOPNOTCH MEDICAL BOARD PREP ANATOMY SUPEREXAM Page 61 of 94 For inquiries visit www.topnotchboardprep.com.ph or email us at [email protected]

TOPNOTCH MEDICAL BOARD PREP ANATOMY SUPEREXAM For inquiries visit www.topnotchboardprep.com.ph or email us at [email protected] Item # 473

QUESTION

EXPLANATION

AUTHOR

TOPNOTCH EXAM BACK-UP MIDTERM EXAM AUG 2014 - FOR INCLUSION IN THE SAMPLEX

A 35 year old female singer underwent a subtotal thyroidectomy due to a thyroid mass. After the surgery, she experienced mild hoarseness of voice and she can not reach high notes. What nerve is most likely injured? A. lateral laryngeal nerve B. superior laryngeal nerve C. inferior laryngeal nerve D. phrenic nerve E. glossopharyngeal nerve

Superior laryngeal nerve supplies the cricothyroid muscle which act as a tensor. Inferior laryngeal nerve supplies the rest of the laryngeal muscles.

LUISA SARANILLO, MD (TOP 6 - FEB 2014 MED BOARDS; TOPNOTCH MD)

474

In pericardiocentesis, pericardial fluid is apirated between what layers of the heart? A. Between fibrous and serous pericardium B. Between the parietal layer of the serous pericardium and the fibrous pericardium C. Between the myocardium and the epicardium D. between the parietal layer of the serous pericardium and the visceral layer of the serous pericardium E. none of the choices

Accumulated pericardial fluid is aspirated between the parietal layer of the serous pericardium and the visceral layer of the pericardium.

LUISA SARANILLO, MD (TOP 6 - FEB 2014 MED BOARDS; TOPNOTCH MD)

BACK-UP MIDTERM EXAM AUG 2014 - FOR INCLUSION IN THE SAMPLEX

475

When inserting an NGT, the first esophageal narrowing encountered is between the esophagus and the cricoid cartilage. What is the distance from the external nares up to this point. A. 30cm B. 28cm C. 44 cm D. 12-14cm E. 18cm

From the external nares the following are the approximated distance: up to the first esophageal narrowing which is between the esophagus and the cricoid cartilage is 18cm; up to the 2nd esophageal narrowing between the esophagus and the arch of aorta is 28cm; up to the cardiac orifice of the stomach is 44cm. An additional 12-14cm is added when pushed up to the pylorus.

LUISA SARANILLO, MD (TOP 6 - FEB 2014 MED BOARDS; TOPNOTCH MD)

BACK-UP MIDTERM EXAM AUG 2014 - FOR INCLUSION IN THE SAMPLEX

476

The blood supply of the appendix comes directly from: A. Posterior cecal artery B. Ileocolic artery C. Right middle colic artery D. Left middle colic artery E. Jejunoileal artery



BACK-UP MIDTERM EXAM AUG 2014 - FOR INCLUSION IN THE SAMPLEX

477

Which of the following best characterized the parathyroid glands? A. The inferior parathyroid glands are more variable in location B. Inferior thyroid artery supplies the inferior parathyroid gland only C. Usually, there are 4 parathyroid glands D. All of the above E. A and C only

Usually, there are 4 parathyroid glands. Inferior thyroid artery supplies both the superior and inferior parathyroid glands. The superior glands are constant in location, while the inferior glands are more variable in location.

LUISA SARANILLO, MD (TOP 6 - FEB 2014 MED BOARDS; TOPNOTCH MD) LUISA SARANILLO, MD (TOP 6 - FEB 2014 MED BOARDS; TOPNOTCH MD)

478

An 18 year old boy had anterior dislocation of the shoulder joint while playing basketball. An injury to this nerve is suspected which will present as: A. Inability to abduct the arm B. Inability to adduct the arm C. Weakness in medial rotation of the arm D. altered sensation in the medial aspect of the arm E. all of the above

axillary nerve is injured in anterior dislocation of the shoulder joint. It will presents with inability to abduct the arm, altered sensation in the lateral arm, and weakness in lateral rotation of the arm.

LUISA SARANILLO, MD (TOP 6 - FEB 2014 MED BOARDS; TOPNOTCH MD)

BACK-UP MIDTERM EXAM AUG 2014 - FOR INCLUSION IN THE SAMPLEX

479

The "yes" and "no" joints are synovial joints without intervertebral disc. The "yes" joint is located between: A. Atlas and occipital condyle B. Atlas and axis C. Axis and occipital condyle D. C3 and C4 E. None of the choices

The "yes" joint is the atlanto-occipital joint located between the atlas and occipital condyle, while the "no" joint is the atlanto-axial joint which is between atlas and axis.

LUISA SARANILLO, MD (TOP 6 - FEB 2014 MED BOARDS; TOPNOTCH MD)

BACK-UP MIDTERM EXAM AUG 2014 - FOR INCLUSION IN THE SAMPLEX

480

What is the anatomic landmark of the cricoid cartilage? A. C4 B. C5 C. C6 D. T2 E. C7

C4 =hyoid bone and Common carotid artery bifurcation. C5= thyroid cartilage. C6 = cricoid cartilage, start of trachea and esophagus. T2 = sternal notch.

LUISA SARANILLO, MD (TOP 6 - FEB 2014 MED BOARDS; TOPNOTCH MD)

BACK-UP MIDTERM EXAM AUG 2014 - FOR INCLUSION IN THE SAMPLEX

TOPNOTCH MEDICAL BOARD PREP ANATOMY SUPEREXAM Page 62 of 94 For inquiries visit www.topnotchboardprep.com.ph or email us at [email protected]

BACK-UP MIDTERM EXAM AUG 2014 - FOR INCLUSION IN THE SAMPLEX

TOPNOTCH MEDICAL BOARD PREP ANATOMY SUPEREXAM For inquiries visit www.topnotchboardprep.com.ph or email us at [email protected] Item # 481

QUESTION

EXPLANATION

Which of the following cranial nerves will NOT be affected in cavernous sinus syndrome? A. CN II B. CN III C. CN IV D. CN VI

CN III, IV, V1, V2 and VI and postganglionic fibers en route to the orbit all pass through the cavernous sinus.

482

This type of glia is important for the maintenance of the blood-brain barrier: A. oligodendroglia B. astrocyte C. microglia D. Schwann cell

The astrocyte is important for physical support, repair, K metabolism, removal of excess neurotransmitters and maintenance of bloodbrain barrier.

483

The V4 precordial lead is placed: A. On the 4th ICS to the left of the sternum B. On the 5th ICS to the left of the sternum C. On the 5th ICS at the left midclavicular line D. On the 4th ICS at the left midclavicular line

Several questions on ECG lead placement were asked during our board exam.

484

This is the most common site of ureteral obstruction: A. Ureteropelvic junction B. Ureterovesical junction C. Pelvic brim D. None of the above

The rest are common sites of obstruction as well.

485

Gastrulation happens within week: A. 1 B. 2 C. 3 D. 4

Gastrulation is the phase during which the singlelayered blastula is reorganized into a trilaminar disc. It happens during week 3.

486

The following are embryologic derivatives of ectoderm, EXCEPT: A. Parotid gland B. Retina C. Melanocytes D. Eustachian tube

Eustachian tube is an endodermal derivative.

487

What is the function of Type 2 pneumocytes? A. For gas exchange B. Degrade toxins C. Secrete IgA D. Precursors to type I pneumocytes

Type 2 cells are also the ones which secrete the pulmonary surfactant.

488

A young fetus of 2 months AOG would be expected to be producing RBCs via the: A. Yolk sac B. Liver C. Bone marrow D. Thymus

Erythropoiesis happens in: yolksack for the first 2 months AOG, liver and spleen around 3 months, and bone marrow predominantly from 5 months up to birth.

489

The left circumflex coronary artery supplies the: A. Anterior 2/3 of interventricular septum B. Posterior walls of the left ventricle C. Posterior 1/3 of interventricular septum D. Lateral wall of the right ventricle

SIMILAR TO PREVIOUS BOARD EXAM CONCEPT/PRINCIPLE. LCX supplies the lateral and posterior walls of the left ventricle.

490

Which of the following hormones is secreted by an acidophil? A. FSH B. GH C. TSH D. LH

Acidophils secrete GH and prolactin.

AUTHOR ANGELIS ANDREA COCOS, MD (TOP 1 - FEB 2014 MED BOARDS; TOPNOTCH MD) ANGELIS ANDREA COCOS, MD (TOP 1 - FEB 2014 MED BOARDS; TOPNOTCH MD) ANGELIS ANDREA COCOS, MD (TOP 1 - FEB 2014 MED BOARDS; TOPNOTCH MD) ANGELIS ANDREA COCOS, MD (TOP 1 - FEB 2014 MED BOARDS; TOPNOTCH MD) ANGELIS ANDREA COCOS, MD (TOP 1 - FEB 2014 MED BOARDS; TOPNOTCH MD) ANGELIS ANDREA COCOS, MD (TOP 1 - FEB 2014 MED BOARDS; TOPNOTCH MD) ANGELIS ANDREA COCOS, MD (TOP 1 - FEB 2014 MED BOARDS; TOPNOTCH MD) ANGELIS ANDREA COCOS, MD (TOP 1 - FEB 2014 MED BOARDS; TOPNOTCH MD) ANGELIS ANDREA COCOS, MD (TOP 1 - FEB 2014 MED BOARDS; TOPNOTCH MD) ANGELIS ANDREA COCOS, MD (TOP 1 - FEB 2014 MED BOARDS; TOPNOTCH MD)

TOPNOTCH MEDICAL BOARD PREP ANATOMY SUPEREXAM Page 63 of 94 For inquiries visit www.topnotchboardprep.com.ph or email us at [email protected]

TOPNOTCH EXAM FINAL EXAM - AUG 2014

FINAL EXAM - AUG 2014

FINAL EXAM - AUG 2014

FINAL EXAM - AUG 2014

FINAL EXAM - AUG 2014

FINAL EXAM - AUG 2014

FINAL EXAM - AUG 2014

FINAL EXAM - AUG 2014

FINAL EXAM - AUG 2014

FINAL EXAM - AUG 2014

TOPNOTCH MEDICAL BOARD PREP ANATOMY SUPEREXAM For inquiries visit www.topnotchboardprep.com.ph or email us at [email protected] Item # 491

QUESTION

EXPLANATION

This ligament contains the portal triad: A. gastrohepatic B. hepatosplenic C. hepatoduodenal D. falciform

The hepatoduodenal ligament connects the liver to the duodenum. It contains the portal triad: hepatic artery, portal vein, and common bile duct.

492

This/these segment/s of the small intestine contain/s the Brunner's glands: A. duodenum B. jejunum C. ileum D. Both A and B

Brunner's glands are found in the submucosa of the duodenum. Several histology questions were asked during our board exam.

493

On peripheral blood smear, which cell presents with a large kidney-shaped nucleus? A. Eosinophil B. Monocyte C. B cell D. T cell

SIMILAR TO PREVIOUS BOARD EXAM CONCEPT/PRINCIPLE.

494

Melanocytes are found in the: A. Stratum basale B. Stratum germinativum C. Statum granulosum D. Both A and B

Stratum basale is sometimes referred to as stratum germinativum, the deepest layer of the epidermis. SIMILAR TO PREVIOUS BOARD EXAM CONCEPT/PRINCIPLE.

495

The most commonly injured rotator cuff muscle is: A. Teres minor B. Teres major C. Supraspinatus D. Infraspinatus

Teres major is not part of the rotator cuff muscles.

496

A patient needs a femoral tap for ABG determination. What compartment of the femoral sheath should you aspirate? A. medial B. lateral C. posterior D. anterior

The acronym from lateral to medial is NAVEL: nerve, artery, vein, empty space and lymphatics. The nerve is not included in the femoral sheath.

497

A woman suffered a traumatic injury to the upper limb which results to an inability to spread and extend her fingers, with noted clawing of the ring and little fingers. Which segments of the brachial plexus would have contributed to the nerve that is damaged? A. C5 and C6 B. C6 and C7 C. C7 and C8 D. C8 and T1

Clawing of the ring and little fingers is a keyphrase for lower segment brachial plexus injury. A question on brachial plexus was asked during our exam.

498

A patient suffers an Achilles tendon rupture. Which of the following muscles is NOT affected: A. soleus B. plantaris C. popliteus D. gastrocnemius

The popliteal tendon does not contribute to the formation of the calcaneal or Achilles tendon.

499

During parotidectomy, the main trunk of the facial nerve was accidentally lacerated. Which of the following muscles will be affected? A. masseter B. buccinator C. temporalis D. pterygoids

Buccinator is innervated by the facial nerve. The rest of the choices are innervated by the trigeminal nerve, specifically V3.

500

The nasolacrimal duct drains into which recess? A. Superior meatus B. Middle meatus C. Inferior meatus D. Lateral meatus

SIMILAR TO PREVIOUS BOARD EXAM CONCEPT/PRINCIPLE. There is no lateral meatus.

AUTHOR ANGELIS ANDREA COCOS, MD (TOP 1 - FEB 2014 MED BOARDS; TOPNOTCH MD) ANGELIS ANDREA COCOS, MD (TOP 1 - FEB 2014 MED BOARDS; TOPNOTCH MD) ANGELIS ANDREA COCOS, MD (TOP 1 - FEB 2014 MED BOARDS; TOPNOTCH MD) ANGELIS ANDREA COCOS, MD (TOP 1 - FEB 2014 MED BOARDS; TOPNOTCH MD) ANGELIS ANDREA COCOS, MD (TOP 1 - FEB 2014 MED BOARDS; TOPNOTCH MD) ANGELIS ANDREA COCOS, MD (TOP 1 - FEB 2014 MED BOARDS; TOPNOTCH MD) ANGELIS ANDREA COCOS, MD (TOP 1 - FEB 2014 MED BOARDS; TOPNOTCH MD)

TOPNOTCH EXAM FINAL EXAM - AUG 2014

FINAL EXAM - AUG 2014

FINAL EXAM - AUG 2014

FINAL EXAM - AUG 2014

FINAL EXAM - AUG 2014

FINAL EXAM - AUG 2014

FINAL EXAM - AUG 2014

ANGELIS ANDREA COCOS, MD (TOP 1 - FEB 2014 MED BOARDS; TOPNOTCH MD) ANGELIS ANDREA COCOS, MD (TOP 1 - FEB 2014 MED BOARDS; TOPNOTCH MD)

FINAL EXAM - AUG 2014

ANGELIS ANDREA COCOS, MD (TOP 1 - FEB 2014 MED BOARDS; TOPNOTCH MD)

FINAL EXAM - AUG 2014

TOPNOTCH MEDICAL BOARD PREP ANATOMY SUPEREXAM Page 64 of 94 For inquiries visit www.topnotchboardprep.com.ph or email us at [email protected]

FINAL EXAM - AUG 2014

TOPNOTCH MEDICAL BOARD PREP ANATOMY SUPEREXAM For inquiries visit www.topnotchboardprep.com.ph or email us at [email protected] Item # 501

QUESTION This nuclei is the most important source of norepinephrine in the cerebral cortex: A. Caudate nucleus B. Substantia nigra pars compacta C. Raphe nucleus D. Locus ceruleus E. Basal nucleus of Meynert

502

A 30-year-old man was allegedly stabbed in his right chest by an unknown assailant as he was going to his car parked 2 blocks away from the bar where he sings.On physical examination, patient is unable to raise his right arm above the horizontal with winging of his right scapula. Which of the following nerves is most likely affected? A. Axillary B. Thoracodorsal C. Long thoracic D. Internal intercosta E. Spinal accesory

503

This is also known as the morrison's pouch: A. Hepatorenal recess B. Splenorenal recess C. Renocolic recess D. Costodiagphragmatic recess

504

505

506

507

508

EXPLANATION

TOPNOTCH EXAM SIMILAR TO PREVIOUS BOARD EXAM JAN BACK-UP CONCEPT/PRINCIPLE. CHARMAINE MIDTERM Caudate nucleus-GABA PALOMAR, EXAM AUG Substantia nigra pars compacta-dopamine MD (TOP 9 - 2014 Raphe nucleus-serotonin FEB 2014 Locus ceruleus-norepinephrine MED Basal nucleus of Meynert-acetylcholine BOARDS; TOPNOTCH MD) SIMILAR TO PREVIOUS BOARD EXAM JAN BACK-UP CONCEPT/PRINCIPLE. The long thoracic nerve CHARMAINE MIDTERM innervates the serratus anterior muscle which is PALOMAR, EXAM AUG responsible for drawing the scapula forward MD (TOP 9 - 2014 around the thoracic wall. A winged scapula is FEB 2014 caused by paralysis of the serratus anterior MED muscle.Snells 8th ed., 441, 434. BOARDS; TOPNOTCH MD)

SIMILAR TO PREVIOUS BOARD EXAM CONCEPT/PRINCIPLE. Morrison's pouchhepatorenal recess; the most posterior cavity in the peritoneal cavity. -Medical eponyms by Andrew J. Yee, MD, 2007

AUTHOR

JAN CHARMAINE PALOMAR, MD (TOP 9 - FEB 2014 MED BOARDS; TOPNOTCH MD) The following muscles are responsible for dorsiflexing the SIMILAR TO PREVIOUS BOARD EXAM JAN foot, except: CONCEPT/PRINCIPLE. Anterior leg muscles- CHARMAINE A. Extensor digitorum brevis action: dorsiflexion and extension, nerve: deep PALOMAR, B. Tibialis anterior peroneal nerve, muscles: tibialis anterior, extensor MD (TOP 9 - C. Peroneus longus digitorum longus, peroneus tertius, extensor FEB 2014 D. Extencsor digitorum longus hallucis longus, extensor digitorum brevis. MED E. None of the above Topnotch handouts. BOARDS; Choice C, peroneous longus is found in the lateral TOPNOTCH compartment of the leg and is responsible for MD) plantar flexion and evertion The gastric mucosa is lined by which epithelium: SIMILAR TO PREVIOUS BOARD EXAM JAN A. Simple squamous CONCEPT/PRINCIPLE. -Junquiera and Carneiro, CHARMAINE B. Simple cuboidal Basic Histology 11th ed., p. 290. PALOMAR, C. Simple columnar MD (TOP 9 - D. Stratified squamous FEB 2014 E. Stratified columnar MED BOARDS; TOPNOTCH MD) Melanocytes are found in which layer of the skin: SIMILAR TO PREVIOUS BOARD EXAM JAN A. Stratum germinativum CONCEPT/PRINCIPLE. Eumelanin is a dark brown CHARMAINE B. Stratum granulosum pigment produced by the melanocyte, a PALOMAR, C. Stratum corneum specialized cell of the epidermis found beneath or MD (TOP 9 - D. Stratum basale between the cells of the stratum basale and the FEB 2014 E. Stratum spinosum hair follicles. Melanocytes have rounded bodies MED from which long irregular extensions branch into BOARDS; the epidermis, running between the cells of the TOPNOTCH strata basale and spinosum. -Junquiera and MD) Carneiro, Basic Histology 11th ed., p. 363 The major mechanism of blindness in glaucoma is: SIMILAR TO PREVIOUS BOARD EXAM JAN A. Optic nerve damage CONCEPT/PRINCIPLE. Vaughan and Ausbury's CHARMAINE B. Increased intraocular pressure General Ophthalmology 17th ed., p. 214-215. PALOMAR, C. Optic disk enlargement MD (TOP 9 - D. Corneal inflammation FEB 2014 E. Lens opacification MED BOARDS; TOPNOTCH MD) Which of the folling statements regarding tube thoracostomy B. The skin incision is usually made over the JAN is true? A. intercostal space below the space to be pierced. CHARMAINE The site of insertion of the tube is at the 4th ICS at the C. The incision through the intercostal space is PALOMAR, anterior axillary line. kept close to the upper border of the rib to avoid MD (TOP 9 - B. The skin incision is usually made over the intercostal injuring the intercostal vessels and nerve. Clinical FEB 2014 space above the space to be pierced. Anatomy by Regions, Snell, 8th ed., 59. MED C. The incision through the intercostal space is kept close to BOARDS; the lower border of the rib to avoid injuring the intercostal TOPNOTCH vessels and nerve. MD) D. All of the statement are correct. E. All of the statements are incorrect.

TOPNOTCH MEDICAL BOARD PREP ANATOMY SUPEREXAM Page 65 of 94 For inquiries visit www.topnotchboardprep.com.ph or email us at [email protected]

BACK-UP MIDTERM EXAM AUG 2014

BACK-UP MIDTERM EXAM AUG 2014

BACK-UP MIDTERM EXAM AUG 2014

BACK-UP MIDTERM EXAM AUG 2014

BACK-UP MIDTERM EXAM AUG 2014

BACK-UP MIDTERM EXAM AUG 2014

TOPNOTCH MEDICAL BOARD PREP ANATOMY SUPEREXAM For inquiries visit www.topnotchboardprep.com.ph or email us at [email protected] Item # 509

QUESTION

EXPLANATION

C-shaped organ: A. Esophagus B. Duodenum C. Spleen D. Pancreas E. Rectum

SIMILAR TO PREVIOUS BOARD EXAM CONCEPT/PRINCIPLE

510

Which of the following statements is true regarding the right recurrent laryngeal nerve? A. It hooks around the ligamentum arteriosum and ascends in the groove between the trachea and the esophagus on the right side. B. It supplies the cricothyroid muscle which is responsible for tensing the vocal cord. C. It branches to become the right vagus. D. It loops around the right subclavian artery and ascends between the trachea and esophagus. E. All of the statements are true.

511

Dorsalis pedis pulse can be appreciated: A. Between the the tendons of flexor digitorum longus and flexor hallucis longus B. Between the tendons of extensor hallucis longus and extensor digitorum longus C. Between the anterosuperior iliac spine and the symphysis pubis D. Midway between the medial and lateral maleoli on the front of the ankle E. Both B and D are correct

SIMILAR TO PREVIOUS BOARD EXAM CONCEPT/PRINCIPLE. Snell 8th ed., p. 127. The Right recurrent laryngeal nerve arises from the right vagus in the neck and hooks around the subclavian artery and ascends between the trachea and esophagus. The left recurrent laryngeal nerve arises from the leftvagus trunk as the nerve crosses the arch of the aorta. it hooks around the ligamentum arteriosum and ascends in the groove between the trachea and the esophagus on the on the left side. it suplies all the muscles acting on the left vocal cord (except the cricothyroid muscle, a tensor of the vocal cord, which is supplied by the external laryngeal branch of the vagus. Snell 8th ed., 658

512

A 23 year-old-man was rushed to the hospital after sustaining multiple physical injuries after jumping from a moving bus. X-ray of the humerus revealed midshaft fracture. Which of the following nerves would most likely be injured? A. Ulnar B. Radial C. Median D. Musculocutaneous E. Axillary

513

514

ECG of a 74 year old-male complaining of chest heaviness revealed ST-segment elevation in leads II, III and aVF. This signifies? A. Inferior wall myocaridal infarction B. Inferior wall myocaridal ischemia C. Anterolateral wall infarction D. Anterolateral wall ischemia E. None of the above An orogastric tube was inserted all the way to the 24-inch mark to a 57 year old male who had an episode of hematemesis. Where is the tip of the tube located? A. esophagus B. cardia of stomach C. pylorus D. duodenum E. Ileum

AUTHOR JAN CHARMAINE PALOMAR, MD (TOP 9 - FEB 2014 MED BOARDS; TOPNOTCH MD) JAN CHARMAINE PALOMAR, MD (TOP 9 - FEB 2014 MED BOARDS; TOPNOTCH MD)

JAN CHARMAINE PALOMAR, MD (TOP 9 - FEB 2014 MED BOARDS; TOPNOTCH MD)

TOPNOTCH EXAM BACK-UP MIDTERM EXAM AUG 2014

BACK-UP MIDTERM EXAM AUG 2014

BACK-UP MIDTERM EXAM AUG 2014

Topnotch handout JAN BACK-UP Humeral fracture- and associated nerve injury CHARMAINE MIDTERM 1. Surgical neck - axillary nerve PALOMAR, EXAM AUG 2. midshaft / spiral groove fracture - radial nerve MD (TOP 9 - 2014 3. Supracondylar - Median nerve FEB 2014 4. Medial epicondyle - Ulnar nerve MED BOARDS; TOPNOTCH MD)

SIMILAR TO PREVIOUS BOARD EXAM CONCEPT/PRINCIPLE

JAN CHARMAINE PALOMAR, MD (TOP 9 - FEB 2014 MED BOARDS; TOPNOTCH MD)

BACK-UP MIDTERM EXAM AUG 2014

Snell 8th ed., p. 130

JAN CHARMAINE PALOMAR, MD (TOP 9 - FEB 2014 MED BOARDS; TOPNOTCH MD)

BACK-UP MIDTERM EXAM AUG 2014

TOPNOTCH MEDICAL BOARD PREP ANATOMY SUPEREXAM Page 66 of 94 For inquiries visit www.topnotchboardprep.com.ph or email us at [email protected]

TOPNOTCH MEDICAL BOARD PREP ANATOMY SUPEREXAM For inquiries visit www.topnotchboardprep.com.ph or email us at [email protected] Item # 515

QUESTION

EXPLANATION

AUTHOR

TOPNOTCH EXAM BACK-UP MIDTERM EXAM AUG 2014

Which of the following is true of jejunum but not of ileum? A. Jejunum is longer compared to ileum. B. Jejunum is wider bored, thicker walled and redder than the ileum. C. Aggregations of lymphoid tissue (Peyer's patches) are present in the mucous membrane of the jejunum. D. There are no plicae circulares in the jejunum. E. Jejunal mesenteric vessels receives numerous short terminal vessels that arise from more than 4 arcades.

In the living, the jejunum can be distinguished from the ileum by the following features: 1. The jejunum lies coiled in the upper part of the peritoneal cavity below the left side of the transverse mesocolon; the ileum is in the lower part of the cavity and in the pelvis. 2. The jejunum is wider bored, thicker walled, and redder than the ileum. The jejunal wall feels thicker because the permanent infoldings of the mucous membrane, the plicae circulares, are larger, more numerous, and closely set in the jejunum, whereas in the upper part of the ileum they are smaller and more widely separated and in the lower part they are absent. 3. The jejunal mesentery is attached to the posterior abdominal wall above and to the left of the aorta, whereas the ileal mesentery is attached below and to the right of the aorta. 4. The jejunal mesenteric vessels form only one or two arcades, with long and infrequent branches passing to the intestinal wall. The ileum receives numerous short terminal vessels that arise from a series of three or four or even more arcades. 5. At the jejunal end of the mesentery, the fat is deposited near the root and is scanty near the intestinal wall. At the ileal end of the mesentery the fat is deposited throughout so that it extends from the root to the intestinal wall. 6. Aggregations of lymphoid tissue (Peyer's patches) are present in the mucous membrane of the lower ileum along the antimesenteric border. In the living these may be visible through the wall of the ileum from the outside.

JAN CHARMAINE PALOMAR, MD (TOP 9 - FEB 2014 MED BOARDS; TOPNOTCH MD)

516

The following are boundaries of the site where breath sounds are best heard except: A. Latissimus dorsi B. Trapezius C. Medial border of scapula D. External oblique muscle E. None of the above

Triangle of auscultation: lateral-medial border of scapula, medial- trapezius, inferior- latissimus dorsi -topnotch handout

BACK-UP MIDTERM EXAM AUG 2014

517

The following muscles elevate the ribs and increase the anteroposterior, transverse and vertical diameters of the thoracic cage except: A. Serratus posterior inferior B. External intercostal C. Innermost intercostal D. Subcostal E. None of the above

Muscles of inspiration (elevate the ribs, Increase the AP, transverse and vertical diameters) - Serratus posterior superior, Levator costarum, External intercostal, Innermost intercostal, Subcostal; Muscles of expiration (depress the ribs) - Serratus posterior inferior, Internal intercostal, And Transversus thoracis - Topnotch handout

JAN CHARMAINE PALOMAR, MD (TOP 9 - FEB 2014 MED BOARDS; TOPNOTCH MD) JAN CHARMAINE PALOMAR, MD (TOP 9 - FEB 2014 MED BOARDS; TOPNOTCH MD)

518

The following statements concerning the dermatomes are true except: A. The C5 dermatome includes the tip of the shoulder on the same side. B. The C8 dermatome includes the thumb on the same side. C. The T10 dermatome lies over the skin of the umbilicus. D. The L4 and L5 dermatome runs along the medial side of the foot and big toe. E. The S1 dermatome includes the lateral side of the foot and the small toe.

C6- thumb; C8-small finger

JAN CHARMAINE PALOMAR, MD (TOP 9 - FEB 2014 MED BOARDS; TOPNOTCH MD)

BACK-UP MIDTERM EXAM AUG 2014

519

The cremaster muscle raise the testis and the scrotum upward for warmth and for protection againsts injury. The cremasteric fascia is a derivative of: A. Superficial fascia B. External oblique muscle C. Transversalis fascia D. Internal oblique muscle E. Peritoneum

External spermatic fascia is dervived from the aponeurosis of the external oblique muscle; the cremasteric fascia is derived from the internal oblique muscle, and the internal spermatic fascia is derived from the fascia transversalis. Snell 8th ed., 167.

JAN CHARMAINE PALOMAR, MD (TOP 9 - FEB 2014 MED BOARDS; TOPNOTCH MD)

BACK-UP MIDTERM EXAM AUG 2014

TOPNOTCH MEDICAL BOARD PREP ANATOMY SUPEREXAM Page 67 of 94 For inquiries visit www.topnotchboardprep.com.ph or email us at [email protected]

BACK-UP MIDTERM EXAM AUG 2014

TOPNOTCH MEDICAL BOARD PREP ANATOMY SUPEREXAM For inquiries visit www.topnotchboardprep.com.ph or email us at [email protected] Item # 520

QUESTION

EXPLANATION

AUTHOR

TOPNOTCH EXAM BACK-UP MIDTERM EXAM AUG 2014

A 30 year old man involved in a car accident was rushed to the ER with severe lower abdominal pain and gross hematuria. Radiologic evaluation revealed pelvic fracture. Which of the following statements regarding bladder rupture is not true? A. The bladder may rupture intraperitoneally if the superior wall of a full bladder was injured. B. The bladder may rupture intraperitoneally or extraperitoneally. C. Urine and blood escape freely into the peritoneal cavity in extraperitoneal bladder rupture. D. Extraperitoneal bladder rupture occurs when bony fragments pierce the anterior part of the bladder wall below the level of peritoneal reflection. E. All of the statements are correct.

There were approximately 3 questions on bladder injury and site of blood accumulation in our board exam. ** Urine and blood escape freely into the peritoneal cavity in intraperitoneal bladder rupture which usually occur with injury to the superior wall of a full bladder (that has extended up into the abdomen). -Snell 8th ed., 351.

JAN CHARMAINE PALOMAR, MD (TOP 9 - FEB 2014 MED BOARDS; TOPNOTCH MD)

521

The foremost example of a saddle joint A) 2nd carpometacarpal B) 2nd metatarsophalangeal C) 1st metatarsophalangeal D) 1st carpometacarpal



MIDTERM 1 EXAM - FEB 2013

522

A surgeon tells a medical student to tap the side of the face of a patient who just had thyroid surgery. The surgeon is most worried about damage to which of the following vessels? A) Common carotid artery B) External carotid artery C) Anterior jugular vein D) Superior and inferior thyroid artery



MIGUEL RAFAEL RAMOS, MD (TOP 3 - FEB 2012 MED BOARDS; TOPNOTCH MD) MIGUEL RAFAEL RAMOS, MD (TOP 3 - FEB 2012 MED BOARDS; TOPNOTCH MD)

523

A 45-year-old woman comes into the ER with abdominal pain that has progressively worsened since the previous night. Imaging reveals that a loop of small intesting has passed through the epiploic foramen into the omental bursa and is constricted by the margins of the foramen. This should not be surgically relieved because this procedure would risk cutting the A) Abdominal aorta B) Hepatic artery C) Hepatic vein D) Pancreatic duct



MIGUEL RAFAEL RAMOS, MD (TOP 3 - FEB 2012 MED BOARDS; TOPNOTCH MD)

MIDTERM 1 EXAM - FEB 2013

524

The 2nd rib is atypical because of this feature? A) groove for subclavian vessels B) only 1 facet that articulates with a single vertebrae C) tuberosity for serratus anterior D) contains the scalene tubercle



MIDTERM 1 EXAM - FEB 2013

525

A 16-year-old teenage boy tried to commit suicide by slashing his wrist after his girlfriend of one week broke up with him. He sustained “suicide cuts” on the lateral side of his wrist, most likely sparing the: A) Radial artery B) Median nerve C) Flexor carpi radialis tendon D) Ulnar artery



MIGUEL RAFAEL RAMOS, MD (TOP 3 - FEB 2012 MED BOARDS; TOPNOTCH MD) MIGUEL RAFAEL RAMOS, MD (TOP 3 - FEB 2012 MED BOARDS; TOPNOTCH MD)

526

An IVP is performed on a patient to evaluate the function and structure of her kidneys. Examination of the resulting radiographs reveal that the left kidney is normal but that there is a duplication of the ureter and renal pelvis on the right side. Further testing reveals that kidney function is normal. This variation is a result of abnormal development of which of the following structures? A) Ureteric bud B) Metanephric blastema C) Mesonephric duct D) Mesonephric tubules



MIGUEL RAFAEL RAMOS, MD (TOP 3 - FEB 2012 MED BOARDS; TOPNOTCH MD)

MIDTERM 1 EXAM - FEB 2013

TOPNOTCH MEDICAL BOARD PREP ANATOMY SUPEREXAM Page 68 of 94 For inquiries visit www.topnotchboardprep.com.ph or email us at [email protected]

MIDTERM 1 EXAM - FEB 2013

MIDTERM 1 EXAM - FEB 2013

TOPNOTCH MEDICAL BOARD PREP ANATOMY SUPEREXAM For inquiries visit www.topnotchboardprep.com.ph or email us at [email protected] Item # 527

QUESTION

EXPLANATION

AUTHOR

TOPNOTCH EXAM MIDTERM 1 EXAM - FEB 2013

A 12-year-old boy falls from a tree he is climbing, but catches himself on a branch with his right hand. He swings by his right arm and jumps to the ground. Several hours later he presents to the ER with right hand clumsiness. Which of the following structures has he most likely injured? A) Lower trunk, brachial plexus B) Axillary nerve C) Musculocutaneous nerve D) Radial nerve



MIGUEL RAFAEL RAMOS, MD (TOP 3 - FEB 2012 MED BOARDS; TOPNOTCH MD)

528

A gastric biopsy is performed on a 42-year-old male. The specimen is observed to have numerous cells with apical membrane-bound secretion granules in the gastric glands. From which area of the stomach was the biopsy taken A) Cardiac region B) Columns of Morgagni C) Fundic region D) Pyloric region



MIGUEL RAFAEL RAMOS, MD (TOP 3 - FEB 2012 MED BOARDS; TOPNOTCH MD)

MIDTERM 1 EXAM - FEB 2013

529

A 52 y/o male has had a chronic cough with occasional lowvolume hemoptysis for the past three weeks. He is a two pack-per-day cigarette smoker, and drinks three to four cans of beer on weekends. PE reveals right-sided face and right arm swelling and engorgement of subcutaneous veins on the right side of the neck. Which of the following veins is most likely obstructed in this patient? A) External jugular B) Subclavian C) Brachiocephalic D) Superior vena cava



MIGUEL RAFAEL RAMOS, MD (TOP 3 - FEB 2012 MED BOARDS; TOPNOTCH MD)

MIDTERM 1 EXAM - FEB 2013

530

Most of the muscles of the buttocks insert in the A) Quadrate tubercle B) Greater tuberosity C) Lesser trochanter D) Greater trochanter



MIDTERM 1 EXAM - FEB 2013

531

After a motor vehicle accident, a patient is brought to the emergency room. Xrays reveal that she has fractures of her left ninth and tenth ribs. She has a rapid heart rate and low blood pressure. Peritoneal lavage reveals free blood in the peritoneal cavity. A surgeon is able to stop the bleeding by placing a clamp across which of the following structures? A) Falciform ligament B) Gastrosplenic ligament C) Splenorenal ligament D) Hepatoduodenal ligament



MIGUEL RAFAEL RAMOS, MD (TOP 3 - FEB 2012 MED BOARDS; TOPNOTCH MD) MIGUEL RAFAEL RAMOS, MD (TOP 3 - FEB 2012 MED BOARDS; TOPNOTCH MD)

532

The glossopharyngeal nerve is transected accidentally during a surgical procedure in a 45 y/o male. Which of the following is most likely lost in this patient? A) Taste sensation from the anterior 2/3 of the tongue B) General sensation from the tonsillar lining C) Salivary secretion from the submandibular gland D) Protrusion of the tongue



MIGUEL RAFAEL RAMOS, MD (TOP 3 - FEB 2012 MED BOARDS; TOPNOTCH MD)

MIDTERM 1 EXAM - FEB 2013

533

Laceration of the male urethra just inferior to the urogenital diaphragm would likely result in extravasation of urine into all of the following regions except A) The abdominal wall between Scarpa's fascia and deep fascia B) The anal triangle between superficial fat and deep fascia C) The penis between the superficial fascia and deep (Buck's) fascia D) The urogenital triangle between the deep layer of superficial (Colle's)



MIGUEL RAFAEL RAMOS, MD (TOP 3 - FEB 2012 MED BOARDS; TOPNOTCH MD)

MIDTERM 1 EXAM - FEB 2013

TOPNOTCH MEDICAL BOARD PREP ANATOMY SUPEREXAM Page 69 of 94 For inquiries visit www.topnotchboardprep.com.ph or email us at [email protected]

MIDTERM 1 EXAM - FEB 2013

TOPNOTCH MEDICAL BOARD PREP ANATOMY SUPEREXAM For inquiries visit www.topnotchboardprep.com.ph or email us at [email protected] Item # 534

QUESTION

EXPLANATION

AUTHOR

TOPNOTCH EXAM MIDTERM 1 EXAM - FEB 2013

A 5-year-old male with a bounding pulse has a thrill best palpated over the upper left sternal edge. A continuous murmur is heard over the area on cardiac auscultation. If surgery is planned, the surgeon should intervene on a derivative of which of the following embryologic structures? A) Sinus venosus B) Bulbus cordis C) 4th aortic arch D) 6th aortic arch



MIGUEL RAFAEL RAMOS, MD (TOP 3 - FEB 2012 MED BOARDS; TOPNOTCH MD)

535

In portal hypertension, which of the following veins will not engorge and is not a collateral of the circulation? A) Inferior rectal B) Middle rectal C) Esophageal D) Hepatic



MIGUEL RAFAEL RAMOS, MD (TOP 3 - FEB 2012 MED BOARDS; TOPNOTCH MD)

MIDTERM 1 EXAM - FEB 2013

536

In performing a cricothyrotomy, an incision should be made at which of the following locations? A) Cricothyroid membrane, at the junction of clavicle and sternum B) Cricothyroid membrane, between the thyroid cartilage and cricoid cartilage below C) Thyrohyoid membrane, between the thyroid cartilage and hyoid bone D) Sternal notch, at the junction of the clavicle and sternum



MIGUEL RAFAEL RAMOS, MD (TOP 3 - FEB 2012 MED BOARDS; TOPNOTCH MD)

MIDTERM 1 EXAM - FEB 2013

537

The structure that serves as a landmark in doing pudendal block anesthesia through a transvaginal approach is the A) Ischial spine B) Iliac crest C) Ischial tuberosity D) Sacral promontory



MIGUEL RAFAEL RAMOS, MD (TOP 3 - FEB 2012 MED BOARDS; TOPNOTCH MD)

MIDTERM 1 EXAM - FEB 2013

538

A neurologic exam of a 34-year-old man reveals a direct and consensual light reflex in his left eye; but neither a direct nor consensual reflex in his right eye. The nerve involved is the A) Right optic nerve B) Left optic nerve C) Right oculomotor nerve D) Left oculomotor nerve



MIGUEL RAFAEL RAMOS, MD (TOP 3 - FEB 2012 MED BOARDS; TOPNOTCH MD)

MIDTERM 1 EXAM - FEB 2013

539

A wall-climber's primary back muscle is in the following group of muscles as which of the following A) Serratus posterior B) Trapezius C) Splenus cervicis D) Erector spinae



MIGUEL RAFAEL RAMOS, MD (TOP 3 - FEB 2012 MED BOARDS; TOPNOTCH MD)

MIDTERM 1 EXAM - FEB 2013

540

A 17-year-old football player comes to the emergency department hopping on his right foot. An X-ray of the left leg shows a fracture of the neck of the fibula. Which of the following findings do you most expect on physical examination? A) Loss of sensation on the sole of the left foot B) weakened inversion of left foot C) loss of plantarflexion of the left foot D) Loss of sensation on the dorsum ofthe left foot



MIGUEL RAFAEL RAMOS, MD (TOP 3 - FEB 2012 MED BOARDS; TOPNOTCH MD)

MIDTERM 1 EXAM - FEB 2013

TOPNOTCH MEDICAL BOARD PREP ANATOMY SUPEREXAM Page 70 of 94 For inquiries visit www.topnotchboardprep.com.ph or email us at [email protected]

TOPNOTCH MEDICAL BOARD PREP ANATOMY SUPEREXAM For inquiries visit www.topnotchboardprep.com.ph or email us at [email protected] Item # 541

QUESTION

EXPLANATION

AUTHOR

A 24/M, gang member, is hit over the right side of his head with a baseball bat during a brawl. He loses consciousness for a few minutes, but recovers promptly. One hour later, he is found unconscious at his home. The right pupil is fixed and dilated. When he was rushed to the ER, stat CT done showed lenticular hyperdensity in the temporoparietal area. The blood vessel which most likely bled in this case, is a direct branch of the: A. Ophthalmic artery B. Maxillary artery C. Internal carotid artery D. Middle cerebral artery E. Anterior choroidal artery

A classic case of epidural hematoma (head injury + lucid interval + lens-shaped hemorrhage on CT). The most common artery damaged in epidural hematoma is the anterior division of the middle meningeal artery, a branch of the maxillary artery.

ABDELSIMA R OMAR II, MD (TOP 2 - AUG 2013 MED BOARDS; TOPNOTCH MD - 200 QUESTIONS) AND MARC DENVER TIONGSON, MD (40 QUESTIONS)

542

A 40/F with known tooth infection comes in due to a large, tender, fluctuant mass occupying the left lower side of her face and upper neck including the underside of the mouth. On examination, the mass is seen pushing up the floor of the mouth on the left side. This patient has an acute infection of this fascial space which is usually due to dental infection. A. Visceral B. Retropharyngeal C. Submandibular D. Masticatory E. Sublingual

Patient has Ludwig's angina due to an acute infection of the submandibular fascial space, commonly secondary to a dental infection.

543

You are performing the neurologic examination on a 45/M, diagnosed with a skull base tumor. When you asked him to protrude the tongue, you noted that the tongue deviated to the left; indicating involvement of the: A. Left glossopharyngeal nerve B. Right glossopharyngeal nerve C. Left hypoglossal nerve D. Right hypoglossal nerve E. Right vagus nerve

When the patient asked to put out the tongue, the tongue will deviate toward the paralyzed side. The normal genioglossus muscle will pull the unaffected side of the tongue forward leaving the paralyzed side of the tongue stationary

544

Which of the following would necessitate cricothyroidotomy Section of the ELN causes weakness of the voice. or tracheostomy? Bilateral complete section of the RLN causes loss A. Section of the external laryngeal nerve of speech and impairment of breathing because B. Unilateral partial section of the recurrent laryngeal nerve the glottis is partially closed (vocal fold assumed C. Bilateral partial section of the recurrent laryngeal nerve position midway between abduction and D. Unilateral complete section of the recurrent laryngeal adduction.) A bilateral partial section of the RLN is nerve the worst, causing bilateral paralysis of abductors. E. Bilateral complete section of the recurrent laryngeal nerve The vocal cord is drawn together, leading to acute breathlessness and stridor; cricothyroidotomy or tracheostomy would be necessary.

545

Knowledge of important airway distances would help in the correct performance of procedures such as endotracheal intubation. What is the average distance between the incisor teeth and the carina? A. 15 cm B. 20 cm C. 25 cm D. 30 cm E. 44 cm

Incisor teeth to vocal cords: 15 cm; incisor teeth to carina: 20 cm; external nares to carina: 30 cm

546

A 12/M came in due to epistaxis. Most cases of epistaxis occur on the anterioinferior septum and involve septal branches of the: A. Posterior ethmoidal artery B. Anterior ethmoidal artery C. Facial artery D. Sphenopalatine artery E. Greater palatine artery



ABDELSIMA R OMAR II, MD (TOP 2 - AUG 2013 MED BOARDS; TOPNOTCH MD - 200 QUESTIONS) AND MARC DENVER TIONGSON, MD (40 QUESTIONS) ABDELSIMA R OMAR II, MD (TOP 2 - AUG 2013 MED BOARDS; TOPNOTCH MD - 200 QUESTIONS) AND MARC DENVER TIONGSON, MD (40 QUESTIONS) ABDELSIMA R OMAR II, MD (TOP 2 - AUG 2013 MED BOARDS; TOPNOTCH MD - 200 QUESTIONS) AND MARC DENVER TIONGSON, MD (40 QUESTIONS) ABDELSIMA R OMAR II, MD (TOP 2 - AUG 2013 MED BOARDS; TOPNOTCH MD - 200 QUESTIONS) AND MARC DENVER TIONGSON, MD (40 QUESTIONS) ABDELSIMA R OMAR II, MD (TOP 2 - AUG 2013 MED BOARDS; TOPNOTCH MD - 200 QUESTIONS) AND MARC DENVER TIONGSON, MD (40

TOPNOTCH MEDICAL BOARD PREP ANATOMY SUPEREXAM Page 71 of 94 For inquiries visit www.topnotchboardprep.com.ph or email us at [email protected]

TOPNOTCH EXAM FINAL EXAM - FEB 2014

FINAL EXAM - FEB 2014

FINAL EXAM - FEB 2014

FINAL EXAM - FEB 2014

FINAL EXAM - FEB 2014

FINAL EXAM - FEB 2014

TOPNOTCH MEDICAL BOARD PREP ANATOMY SUPEREXAM For inquiries visit www.topnotchboardprep.com.ph or email us at [email protected] Item #

QUESTION

EXPLANATION

AUTHOR

TOPNOTCH EXAM

QUESTIONS)

547

A 40/M, a known case of coronary artery disease, experiences severe crushing pain in the chest referred to the medial side of the arm and armpit. Which nerve is responsible for the referred pain to the medial side of the arm? A. Anterior intercostal nerve B. Posterior intercostal nerve C. Lateral intercostal nerve D. Intercostobrachial nerve E. Lateral cutaneous nerve

Intercostobrachial nerve communicates with the medial cutaneous nerve of the arm.

548

A patient with carcinoma of the lower third of the esophagus who underwent surgical removal of the lesion also had the stomach, upper half of duodenum, spleen and omenta removed, because all drain commonly into this group of lymph nodes: A. Deep cervical B. Mediastinal C. Celiac D. Superior mesenteric E. Inferior mesenteric

The upper third of the esophagus drains into the deep cervical; the middle third into the mediastinal.

549

A barium swallow can also be used to assess the size of this chamber of the heart: A. Left atrium B. Left ventricle C. Right atrium D. Right ventricle

If you think about it carefully, the question simply asks the posteriormost of the chambers. Remember that the anterior wall of the esophagus and the posterior wall of the left atrium are in close proximity; hence, a barium swallow can be used to assess the size of the LA.

550

How many intervertebral discs are there? A. 21 B. 23 C. 25 D. 27 E. 29

No IV disc between C1 and C2, sacrum and coccyx.

551

A 24/M, a member of the college varsity swimming team, comes in to you complaining of shoulder pain. On examination, you note pain on the shoulder while in the midrange of abduction. You are considering rotator cuff tendinitis, due to this muscle rubbing on the acromion because of a degenerated subacromial bursa. A. Supraspinatus B. Infraspinatus C. Teres minor D. Teres major E. Subscapularis



ABDELSIMA R OMAR II, MD (TOP 2 - AUG 2013 MED BOARDS; TOPNOTCH MD - 200 QUESTIONS) AND MARC DENVER TIONGSON, MD (40 QUESTIONS) ABDELSIMA R OMAR II, MD (TOP 2 - AUG 2013 MED BOARDS; TOPNOTCH MD - 200 QUESTIONS) AND MARC DENVER TIONGSON, MD (40 QUESTIONS) ABDELSIMA R OMAR II, MD (TOP 2 - AUG 2013 MED BOARDS; TOPNOTCH MD - 200 QUESTIONS) AND MARC DENVER TIONGSON, MD (40 QUESTIONS) ABDELSIMA R OMAR II, MD (TOP 2 - AUG 2013 MED BOARDS; TOPNOTCH MD - 200 QUESTIONS) AND MARC DENVER TIONGSON, MD (40 QUESTIONS) ABDELSIMA R OMAR II, MD (TOP 2 - AUG 2013 MED BOARDS; TOPNOTCH MD - 200 QUESTIONS) AND MARC DENVER TIONGSON, MD (40 QUESTIONS)

TOPNOTCH MEDICAL BOARD PREP ANATOMY SUPEREXAM Page 72 of 94 For inquiries visit www.topnotchboardprep.com.ph or email us at [email protected]

FINAL EXAM - FEB 2014

FINAL EXAM - FEB 2014

FINAL EXAM - FEB 2014

FINAL EXAM - FEB 2014

FINAL EXAM - FEB 2014

TOPNOTCH MEDICAL BOARD PREP ANATOMY SUPEREXAM For inquiries visit www.topnotchboardprep.com.ph or email us at [email protected] Item # 552

QUESTION

EXPLANATION

An infant was diagnosed to have Erb-Duchenne palsy as a complication of difficult delivery. The right limb was seen hanging limp, rotated medially and pronated. Which of the following muscles is unaffected in Erb-Duchenne palsy? A. Supraspinatus B. Infraspinatus C. Biceps brachii D. Subclavius E. None of the above

Erb-Duchenne palsy usually affects the (1) suprascapular nerve causing palsy of the supraspinatus (responsible for shoulder abduction) and infraspinatus (lateral rotation shoulder); (2) nerve to subclavius (depression of clavicle); (3) musculocutaneous nerve causing palsy of the biceps, brachialis and coracobrachialis (flex and supinate elbow); and (4) axillary nerve, supplying the deltoid (shoulder abudciton).

553

A 55/F obese, diagnosed case of bilateral knee osteoarthritis, has been using crutches for a year to help her ambulate. She comes in to you complaining of difficulty in abducting her right shoulder. On examination, you note loss of the rounded contour of the right shoulder and loss of sensation on the lower half of the right deltoid. Which nerve must have been affected? A. Medial antebrachial cutaneous nerve B. Medial cutaneous nerve C. Axillary nerve D. Musculocutaneous nerve E. Median nerve

Poorly adjusted crutches may result in axillary nerve palsy presenting as weak abduction of the shoulder and lateral rotation of the arm (actions of the deltoid and teres minor, respectively); loss of sensation on the lower half of the deltoid; and wasting of the deltoid.

554

Ureteral stones are most likely to lodge at the narrowest part of the ureter which is at the: A. Ureteropelvic junction B. Ureterovesical junction C. Point at which the ureter crosses over the iliac vessels D. Pelvic brim

SIMILAR TO PREVIOUS BOARD EXAM CONCEPT/PRINCIPLE.

555

You were tasked to perform urethral catheterization on a patient prior to surgery. You note resistance in passing the catheter at the narrowest part of the urethra which is the: A. External orifice of the glans penis B. Membranous urethra C. Bulbar urethra D. Prostatic urethra

Things to remember. The external orifice of the glans is the narrowest part; the prostatic urethra is the widest and most dilatable; the part within the bulb is subject to chronic inflammation and stricture formation and is the most common site of rupture after a perineal blow.

556

In performing lumbar puncture in an infant, it is important to remember that the spinal cord terminates as low as this vertebral level. A. L1 B. L2 C. L3 D. L4 E. L5

Spinal cord ends at the lower border of L1 in the adult; and at L3 in the infant. The subarachnoid space extends down as far as the lower border of S2.

557

In the adult, the notochord gives rise to the: A. Spinal cord B. Vertebral body C. Annulus fibrosis D. Nucleus pulposus E. It has no adult derivatives

Expect a few (1-2) embryology questions.

AUTHOR ABDELSIMA R OMAR II, MD (TOP 2 - AUG 2013 MED BOARDS; TOPNOTCH MD - 200 QUESTIONS) AND MARC DENVER TIONGSON, MD (40 QUESTIONS) ABDELSIMA R OMAR II, MD (TOP 2 - AUG 2013 MED BOARDS; TOPNOTCH MD - 200 QUESTIONS) AND MARC DENVER TIONGSON, MD (40 QUESTIONS) ABDELSIMA R OMAR II, MD (TOP 2 - AUG 2013 MED BOARDS; TOPNOTCH MD - 200 QUESTIONS) AND MARC DENVER TIONGSON, MD (40 QUESTIONS) ABDELSIMA R OMAR II, MD (TOP 2 - AUG 2013 MED BOARDS; TOPNOTCH MD - 200 QUESTIONS) AND MARC DENVER TIONGSON, MD (40 QUESTIONS) ABDELSIMA R OMAR II, MD (TOP 2 - AUG 2013 MED BOARDS; TOPNOTCH MD - 200 QUESTIONS) AND MARC DENVER TIONGSON, MD (40 QUESTIONS) ABDELSIMA R OMAR II, MD (TOP 2 - AUG 2013 MED BOARDS; TOPNOTCH MD - 200 QUESTIONS) AND MARC DENVER TIONGSON, MD (40 QUESTIONS)

TOPNOTCH MEDICAL BOARD PREP ANATOMY SUPEREXAM Page 73 of 94 For inquiries visit www.topnotchboardprep.com.ph or email us at [email protected]

TOPNOTCH EXAM FINAL EXAM - FEB 2014

FINAL EXAM - FEB 2014

FINAL EXAM - FEB 2014

FINAL EXAM - FEB 2014

FINAL EXAM - FEB 2014

FINAL EXAM - FEB 2014

TOPNOTCH MEDICAL BOARD PREP ANATOMY SUPEREXAM For inquiries visit www.topnotchboardprep.com.ph or email us at [email protected] Item # 558

QUESTION

EXPLANATION

A 24/M basketball player lands awkwardly from a rebound, and feels that his right knee has hyperextended, bent inward and popped. The knee becomes swollen over the next few hours. On PE, you note a positive Lachman test. The patient has suffered a tear in the ligament which prevents: A. Anterior displacement of the tiba on the femur B. Posterior displacement of the tibia on the femur C. Adduction of the leg at the knee D. Abduction fo the leg at the knee

ACL tear.

559

A 35/M, known case of Pott's disease, developed a psoas abscess.. On PE, you noted that he had 1+ patellar tendon reflex on the same side and he had difficulty walking. What nerve is involved? A. Femoral nerve B. Obturator nerve C. Sciatic nerve D. Genitofemoral nerve E. Pudendal nerve



560

The germinal epithelium of the ovary is an example of: A. Simple squamous epithelium B. Stratified squamous epithelium C. Simple columnar epithelium D. Stratified columnar epithelium E. Simple cuboidal epithelium

Other simple cuboidal epithelium include the LENS of the eye, the respiratory bronchioles, thyroid cells, and the pigment layer of the retina.

561

the structure which passes along the borders of the Simons triangle is the: a. recurrent laryngeal nerve b. median nerve c. superficial peroneal nerve d. greater auricular nerve

Answer: Recurrent LN can usually be seen within the Simons triangle bounded by esophagus, ITA, and common carotid.

562

the weight of the thyroid gland in a normal adult is: a. 20g b. 30g c. 40g d. 45g

20g is the average weight of the thyroid gland

563

the esophagus is lined by this epithelium a. stratified squamous b. simple columnar c. stratified cuboidal d. pseudostratified columnar



564

the smooth portion of the ventricles of the heart is derived from a. primitive ventricle b. bulbos cordis c. sinus venosus d. none of the above



AUTHOR ABDELSIMA R OMAR II, MD (TOP 2 - AUG 2013 MED BOARDS; TOPNOTCH MD - 200 QUESTIONS) AND MARC DENVER TIONGSON, MD (40 QUESTIONS) ABDELSIMA R OMAR II, MD (TOP 2 - AUG 2013 MED BOARDS; TOPNOTCH MD - 200 QUESTIONS) AND MARC DENVER TIONGSON, MD (40 QUESTIONS) ABDELSIMA R OMAR II, MD (TOP 2 - AUG 2013 MED BOARDS; TOPNOTCH MD - 200 QUESTIONS) AND MARC DENVER TIONGSON, MD (40 QUESTIONS) BLAKE WARREN ANG, MD (TOP 1 - AUG 2013 MED BOARDS; TOPNOTCH MD) BLAKE WARREN ANG, MD (TOP 1 - AUG 2013 MED BOARDS; TOPNOTCH MD) BLAKE WARREN ANG, MD (TOP 1 - AUG 2013 MED BOARDS; TOPNOTCH MD) BLAKE WARREN ANG, MD (TOP 1 - AUG 2013 MED BOARDS; TOPNOTCH MD)

TOPNOTCH MEDICAL BOARD PREP ANATOMY SUPEREXAM Page 74 of 94 For inquiries visit www.topnotchboardprep.com.ph or email us at [email protected]

TOPNOTCH EXAM FINAL EXAM - FEB 2014

FINAL EXAM - FEB 2014

FINAL EXAM - FEB 2014

MIDTERM 2 EXAM - FEB 2014

MIDTERM 2 EXAM - FEB 2014

MIDTERM 2 EXAM - FEB 2014

MIDTERM 2 EXAM - FEB 2014

TOPNOTCH MEDICAL BOARD PREP ANATOMY SUPEREXAM For inquiries visit www.topnotchboardprep.com.ph or email us at [email protected] Item # 565

QUESTION

EXPLANATION

AUTHOR

TOPNOTCH EXAM MIDTERM 2 EXAM - FEB 2014

a patient presents to the trauma center after a vehicular crash with tachypnea and hypotension. the trachea is deviated to the right with decrease pulses on both arms and legs. as an initial/immediate life saving procedure, the physician should access the thorax through a. right 2nd ICS MCl b. right 5th ICS MAL c. left 2nd ICS MCL d. left 2nd ICS MAL

Left 2nd ICS MCL. Although technically anywhere within the chest can be accessed, it is typically within this area to avoid any major structures.

BLAKE WARREN ANG, MD (TOP 1 - AUG 2013 MED BOARDS; TOPNOTCH MD)

566

blood cell component with the longest half life a. basophil b. rbc c. platelet d. neutrophil

Adult RBC has half life of 90-120 days

MIDTERM 2 EXAM - FEB 2014

567

osteoid is predominantly produced by: a. osteoblast b. osteoclast c. osteocyte d. all of the above



568

branchial arch derivative of the stylopharyngeus is: a. branchial arch 1 b. branchial arch 2 c. branchial arch 3 d. branchial arch 5



569

oxyphil cell is found in: a. parathyroid gland b. pituitary gland c. adrenal gland d. thyroid gland



570

the vein that joins the subcostal vein to form the azygos vein is: a. ascending lumbar b. musculophrenic c. ascending phrenic d. superior epigastric



BLAKE WARREN ANG, MD (TOP 1 - AUG 2013 MED BOARDS; TOPNOTCH MD) BLAKE WARREN ANG, MD (TOP 1 - AUG 2013 MED BOARDS; TOPNOTCH MD) BLAKE WARREN ANG, MD (TOP 1 - AUG 2013 MED BOARDS; TOPNOTCH MD) BLAKE WARREN ANG, MD (TOP 1 - AUG 2013 MED BOARDS; TOPNOTCH MD) BLAKE WARREN ANG, MD (TOP 1 - AUG 2013 MED BOARDS; TOPNOTCH MD)

571

fracture of the humeral midshaft produces this abnormality: a. inability to flex the elbow joint b. numbness of the medial portion of the palm c. inability to extend wrist joint d. inability to abduct the upper arm

Radial nerve injury (know the difference of neuropraxia, axonotmesis, etc.) causes inability to extend the wrist joint.

MIDTERM 2 EXAM - FEB 2014

572

the epithelial covering of the lens of the eye is: a. simple squamous b. simple cuboidal c. simple columnar d. pseudostratified cuboidal



573

the sternal angle of Louis coincides with this vertebral level a. C4 b. T4 c. T8 d. C6



574

the left gastroepiploic vein drains to the a. splenic vein b. SMV c. IMV d. Poral vein



BLAKE WARREN ANG, MD (TOP 1 - AUG 2013 MED BOARDS; TOPNOTCH MD) BLAKE WARREN ANG, MD (TOP 1 - AUG 2013 MED BOARDS; TOPNOTCH MD) BLAKE WARREN ANG, MD (TOP 1 - AUG 2013 MED BOARDS; TOPNOTCH MD) BLAKE WARREN ANG, MD (TOP 1 - AUG 2013 MED BOARDS; TOPNOTCH MD)

TOPNOTCH MEDICAL BOARD PREP ANATOMY SUPEREXAM Page 75 of 94 For inquiries visit www.topnotchboardprep.com.ph or email us at [email protected]

MIDTERM 2 EXAM - FEB 2014

MIDTERM 2 EXAM - FEB 2014

MIDTERM 2 EXAM - FEB 2014

MIDTERM 2 EXAM - FEB 2014

MIDTERM 2 EXAM - FEB 2014

MIDTERM 2 EXAM - FEB 2014

MIDTERM 2 EXAM - FEB 2014

TOPNOTCH MEDICAL BOARD PREP ANATOMY SUPEREXAM For inquiries visit www.topnotchboardprep.com.ph or email us at [email protected] Item # 575

QUESTION

EXPLANATION

the major blood supply to the femoral head is a. medial circumflex artery b. lateral circumflex artery c. feeding artery to the femoral head d. deep femoral artery



576

drainage of the nasolacrimal duct is towards the: a. superior meatus b. middle meatus c. inferior meatus d. oropharynx



577

a person was stabbed at the transpyloric area through the neck of the pancreas. which of the following structures will be least affected: a. SMA b. IMA c. Splenic vein d. Portal vein



578

most common site Of ureteral obstruction: a. ureteropelvic junction b. pelvic brim level c. ureterovesical junction

Answer: Uteropelvic junction , but the narrowest portion is ureterovesical junction

579

the most common hernia in females a. direct b. indirect c. femoral d. obturator

Indirect hernia is the most common type of hernia in females; males are more involved than females.

580

craniophayngioma would present with what visual field defect a. binasal hemianopsia b. bitempora hemianopsia c. right/left homonymous hemianopsia d. right superior quantantanopsia

Bilateral temporal hemianopsia

581

In endocrine histology, the presence of oxyphil cells will tell us that the tissue specimen came from what organ? A. Thyroid gland B. Parathyroid gland C. Pituitary gland D. Adrenal gland E. Gonads

Oxyphil cells are found in the parathyoid gland. There function is still unknown.

582

Which is the most proximal part of the respiratory conducting system that has absent goblet cells and submucous glands? A. Lobar bronchi B. Segmental bronchi C. Terminal bronchioles D. Respiratory bronchioles E. Alveolar ducts

583

A 32 yr old biker fell and sustain lacerations in the right arm with suspicious fracture. In the ER, X-rays were requested which showed fracture in the midshaft of the humerus. Which of the following muscles would exhibit weakness? A. Coracobrachialis B. Brachialis C. Brachioradialis D. Pronator teres E. Flexor digitorum profundus

AUTHOR BLAKE WARREN ANG, MD (TOP 1 - AUG 2013 MED BOARDS; TOPNOTCH MD) BLAKE WARREN ANG, MD (TOP 1 - AUG 2013 MED BOARDS; TOPNOTCH MD) BLAKE WARREN ANG, MD (TOP 1 - AUG 2013 MED BOARDS; TOPNOTCH MD)

TOPNOTCH EXAM MIDTERM 2 EXAM - FEB 2014

MIDTERM 2 EXAM - FEB 2014

MIDTERM 2 EXAM - FEB 2014

BLAKE WARREN ANG, MD (TOP 1 - AUG 2013 MED BOARDS; TOPNOTCH MD) BLAKE WARREN ANG, MD (TOP 1 - AUG 2013 MED BOARDS; TOPNOTCH MD) BLAKE WARREN ANG, MD (TOP 1 - AUG 2013 MED BOARDS; TOPNOTCH MD) TIMOTHY TANG LEE SAY, MD (TOP 4 - AUG 2013 MED BOARDS; TOPNOTCH MD)

MIDTERM 2 EXAM - FEB 2014

The respiratory conducting system in proximal to distal order: trachea, main bronchi, lobar bronchi, terminal bronchioles, respiratory bronchioles, alveolar ducts, alveolar sacs, alveoli. Goblet cells and submucous glands are present up to the segmental bronchi. Smooth muscles are present up to alveolar ducts. Elastic fibers are present up to alveoli sacs.

TIMOTHY TANG LEE SAY, MD (TOP 4 - AUG 2013 MED BOARDS; TOPNOTCH MD)

MIDTERM 1 EXAM - FEB 2014

Brachioradialis is the only flexor muscle that is also innervated by the radial nerve. Midshaft fractures of the humerus usually affect the radial nerve. Coracobrachialis and brachialis are innervated by the musculocutaneous nerve. Pronator teres and flexor digitorum profundus are innervated by the median nerve.

TIMOTHY TANG LEE SAY, MD (TOP 4 - AUG 2013 MED BOARDS; TOPNOTCH MD)

MIDTERM 1 EXAM - FEB 2014

TOPNOTCH MEDICAL BOARD PREP ANATOMY SUPEREXAM Page 76 of 94 For inquiries visit www.topnotchboardprep.com.ph or email us at [email protected]

MIDTERM 2 EXAM - FEB 2014

MIDTERM 2 EXAM - FEB 2014

MIDTERM 1 EXAM - FEB 2014

TOPNOTCH MEDICAL BOARD PREP ANATOMY SUPEREXAM For inquiries visit www.topnotchboardprep.com.ph or email us at [email protected] Item # 584

QUESTION A patient sustain an injury in the posterior cord of the brachial plexus. The following actions are compromised EXCEPT? A. Medial rotation of the shoulder B. Supination of the forearm C. Elevation of the trunk D. Extension of the forearm E. Lateral rotation of the shoulder

585

Which of the following muscles abducts, flexes and laterally rotates the thigh at the knee joint? A. Sartorius B. Tensor fascia lata C. Rectus femoris D. Gracilis E. Pectineus

586

A varsity basketball player had an audible pop in his right knee upon landing from a jump shot. This was accompanied by excruciating pain and swelling of the knee joint. To check for possible ligamental tear, the rehabilitation medicine specialist asked the patient to lie down and perform the anterior drawer test by doing this maneuver? A. Forward sliding of the femur on the tibia due to rupture of either the anterior or posterior cruciate ligament B. Forward sliding of the femur on the tibia due to rupture of the posterior cruciate ligament C. Forward sliding of the tibia on the femur due to rupture of the posterior cruciate ligament D. Forward sliding of the femur on the tibia due to rupture of the anterior cruciate ligament E. Forward sliding of the tibia on the femur due to rupture of the anterior cruciate ligament

587

Which is the final draining vessel of the conventional aqueous humor outflow pathway? A. Orbital veins B. Trabecular meshwork C. Schlemm's canal D. Episcleral veins E. Aqueous veins

588

589

EXPLANATION

AUTHOR

The posterior cord gives rise to the upper and lower subscapular nerves, thoracodorsal, axillary and radial nerves. Supination of the forearm is the main action of biceps brachii which is innervated by the musculocutaneous nerve. Elevation of the trunk is an action of the latissimus dorsi innervated by the thoracodorsal nerve. Radial nerve innervates the posterior compartment of the arm and forearm (extensors). The muscles of the shoulder are innervated by the other nerves mentioned. Actions: Tensor fascia lata: abducts, medially rotates and flexes thigh Rectus femoris: flexes the thigh and extends the leg Gracilis: adducts thigh, flexes and medially rotate the leg Pectineus: adducts, flexes and medially rotate the thigh Anterior drawer sign: Forward sliding of the tibia on the femur due to rupture of the anterior cruciate ligament Posterior drawer sign: Backward sliding of the tibia on the femur due to rupture of the posterior cruciate ligament

TIMOTHY TANG LEE SAY, MD (TOP 4 - AUG 2013 MED BOARDS; TOPNOTCH MD)

TOPNOTCH EXAM MIDTERM 1 EXAM - FEB 2014

TIMOTHY TANG LEE SAY, MD (TOP 4 - AUG 2013 MED BOARDS; TOPNOTCH MD)

MIDTERM 1 EXAM - FEB 2014

TIMOTHY TANG LEE SAY, MD (TOP 4 - AUG 2013 MED BOARDS; TOPNOTCH MD)

MIDTERM 1 EXAM - FEB 2014

The pathway for aqueous humor outflow are the following: trabecular meshwork → Schlemm's canal → Collector channels → Aqueous veins → Episcleral veins → Orbital veins → Intracranial cavernous sinus

TIMOTHY TANG LEE SAY, MD (TOP 4 - AUG 2013 MED BOARDS; TOPNOTCH MD)

MIDTERM 1 EXAM - FEB 2014

A 60 yr old patient with known atherosclerotic heart disease suddenly complain of an acute onset blurring of vision, in which stroke was entertained. MRI shows that the lesion involved the cerebral cortex supplied by the left posterior cerebral artery, what visual field defect is expected? A. Right homonymous hemianopsia without macular sparing B. Left homonymous hemianopsia without macular sparing C. Bitemporal hemianopsia without macular sparing D. Right homonymous hemianopsia with macular sparing E. Left homonymous hemianopsia with macular sparing

Review the visual pathway. Vascular and neoplastic (malignant or benign tumours) lesions from the optic tract, to visual cortex can cause a contralateral homonymous hemianopsia. The difference is that cortical lesions tend to have macular sparing since the optic tract send signals coming from the macula to both cortices and they have dual blood supply from the MCA (aside from the PCA).

TIMOTHY TANG LEE SAY, MD (TOP 4 - AUG 2013 MED BOARDS; TOPNOTCH MD)

MIDTERM 1 EXAM - FEB 2014

An otorhinolaryngologist is examing the tympanic membrane of a 20 yr old patient with chronic otitis media. He knows that the external auditory canal is divided into a? A. 1/2 outer bony portion and 1/2 inner cartilaginous portion B. 1/3 outer cartilaginous portion and 2/3 inner bony portion C. 1/3 outer bony portion and 2/3 inner cartilaginous portion D. 2/3 outer cartilaginous portion and 1/3 inner bony portion E. 2/3 outer bony portion and 1/3 inner cartilaginous portion

The adult external canal is divided into an outer cartilagenous portion in its outer 1/3 and bony portion in its inner 2/3. It measures about 2.5 cms on the whole.

TIMOTHY TANG LEE SAY, MD (TOP 4 - AUG 2013 MED BOARDS; TOPNOTCH MD)

MIDTERM 1 EXAM - FEB 2014

TOPNOTCH MEDICAL BOARD PREP ANATOMY SUPEREXAM Page 77 of 94 For inquiries visit www.topnotchboardprep.com.ph or email us at [email protected]

TOPNOTCH MEDICAL BOARD PREP ANATOMY SUPEREXAM For inquiries visit www.topnotchboardprep.com.ph or email us at [email protected] Item # 590

QUESTION

EXPLANATION

AUTHOR

TOPNOTCH EXAM MIDTERM 1 EXAM - FEB 2014

A patient had been in a car accident wherein he suffered superfical lacerations but the neurologic exam was normal. 6 months later, he complains of loss of pain and temperature sensations in the hands and forearm. This progressed to include mild weakness of hand grip and extension of the sensory loss to the arm and shoulders. An MRI of the spinal cord shows a cyst in a segment of the spinal cord. This pathology is consistent with? A. Anterior cord syndrome B. Central cord syndrome C. Brown-Sequard's syndrome D. Posterior cord syndrome E. Tabes dorsalis

The pathology of a cape-like loss of sensory functions is indicative of syringomyelia or a central cord syndrome. Anterior cord will result to sensory deficits from a dermatome level down up to the lower extremities. Brown-Sequard is a hemisection syndrome. Posterior cord syndrome will show loss of position sense and vibration. Tabes dorsalis is also a posterior cord syndrome.

TIMOTHY TANG LEE SAY, MD (TOP 4 - AUG 2013 MED BOARDS; TOPNOTCH MD)

591

During thyroidectomy, the surgeon accidently ligated the right recurrent laryngeal nerve. Which of the following actions on the vocal cord by the muscles of phonation is SPARED? A. Tenses B. Abducts C. Adducts D. Relaxes E. Opens

All muscles of phonation are innervated by the recurrent larngeal nerve except for the cricothyroid which is responsible for tensing the vocal cords

TIMOTHY TANG LEE SAY, MD (TOP 4 - AUG 2013 MED BOARDS; TOPNOTCH MD)

MIDTERM 1 EXAM - FEB 2014

592

A athlete had just finsihed running a marathon. He has labored breathing. The following muscles are responsible for inspiration EXCEPT? A. External intercostal B. Innermost intercostal C. Serratus posterior superior D. Serratus posterior inferior E. Subcostal

All muscles of the thoracic wall are responsible for inspiration except serratus post inf, internal intercostal and tranverse thoracis muscles which are responsible for labored expiration.

TIMOTHY TANG LEE SAY, MD (TOP 4 - AUG 2013 MED BOARDS; TOPNOTCH MD)

MIDTERM 1 EXAM - FEB 2014

593

In the embryology of the primitive heart, which of the following is INCORRECTLY paired? A. Truncus arteriosus = Pulmonary trunk B. Bulbus cordis = Right atrium C. Sinus venosus = Coronary sinus D. Primitive atrium = Left atrium E. Primitive ventricle = Left Ventricle

Review the fate of the fetal heart. Bulbus cordis becomes the right ventricle. Sinus venosus (right horn) becomes the right atrium.

TIMOTHY TANG LEE SAY, MD (TOP 4 - AUG 2013 MED BOARDS; TOPNOTCH MD)

MIDTERM 1 EXAM - FEB 2014

594

A 12-yr old boy was eating hurriedly when the school bell rang which signaled the end of recess period. He suddenly choked while swallowing the siomai whole without chewing. Heimlick maneuver was done but was unsuccessful to dislodge the food from the airways. Rigid bronchoscopy was done. The siomai is most likely lodge in the? A. Posterior bronchopulmonary segment of the right superior lobe B. Posterior bronchopulmonary segment of the left superior lobe C. Superior bronchopulmonary segment of the right inferior lobe D. Inferior lingular bronchopulmonary segment of the left superior lobe E. Posterior basal bronchopulmonary segment of the right inferior lobe

In sitting and standing positions, foreign bodies tend to lodge in the posterior basal BPS of the right lower lobe. Please review the usual location of foreign body aspiration with different positions of the body.

TIMOTHY TANG LEE SAY, MD (TOP 4 - AUG 2013 MED BOARDS; TOPNOTCH MD)

MIDTERM 1 EXAM - FEB 2014

595

The abdominal cavity is lined by the peritoneum. Which fo the following is TRUE regarding the peritoneum? A. The peritoneum is divided into two continuous layers, the parietal peritoneum investing the organs and the visceral peritoneum lining the internal surface of the the abdominopelvic wall. B. The pain fibers subserving the parietal peritoneum is primarily sensitive to stretching. C. The duodenum, pancreas, and transverse colon are retroperitoneal organs. D. Pain in the visceral peritoneum is generally localized. E. The peritoneal cavity is the space between the parietal and visceral peritoneum that contains 50 mL of serous fluid to allow sliding between the layers.

For A, the parietal peritonuem should line the internal surface of the wall and the visceral peritoneum should invest the organs. Parietal is sensitive to pressure, pain and heat, with pain generally localized. Visceral is sensitive to stretching and pain is poorly localized. The ascending and descending colons are retroperitoneal not the transverse colon.

TIMOTHY TANG LEE SAY, MD (TOP 4 - AUG 2013 MED BOARDS; TOPNOTCH MD)

MIDTERM 1 EXAM - FEB 2014

TOPNOTCH MEDICAL BOARD PREP ANATOMY SUPEREXAM Page 78 of 94 For inquiries visit www.topnotchboardprep.com.ph or email us at [email protected]

TOPNOTCH MEDICAL BOARD PREP ANATOMY SUPEREXAM For inquiries visit www.topnotchboardprep.com.ph or email us at [email protected] Item # 596

QUESTION

EXPLANATION

AUTHOR

TOPNOTCH EXAM MIDTERM 1 EXAM - FEB 2014

A patient undergone upper GI endoscopy for recurrent peptic ulcer disease. At what level of the gastrointestinal tract is the scope in at 40 cm from the central incisors? A. Lower esophageal sphincter B. Cardia of the stomach C. Body of the stomach D. Antrum of the stomach E. Pylorus

The esophagus is 25 cm long but from the incisor, 15 cm - UES, 18 cm - thoracic inlet, 25 cm - arch of the aorta, 40 cm - LES

TIMOTHY TANG LEE SAY, MD (TOP 4 - AUG 2013 MED BOARDS; TOPNOTCH MD)

597

A trauma surgeon operated on a patient who had a gunshot wound. In running the bowel, the surgeon differentiates the small intestines because he knows that the jejunum has the following characteristics EXCEPT? A. Deeper red color B. Thinner walls C. Fewer arcades D. Longer vasa recta E. Larger plica circularis

Jejunum has deeper red color, thick and heavy wall, 2-4 cm in caliber, greater vascularity, long vasa recta, few and large arcades, less fat, tall and large plica, and few lymphoid nodules.\

TIMOTHY TANG LEE SAY, MD (TOP 4 - AUG 2013 MED BOARDS; TOPNOTCH MD)

MIDTERM 1 EXAM - FEB 2014

598

What is TRUE regarding the ligametum teres? A. It forms a border of between the quadrate lobe and the left lobe of the liver. B. It is a remnant of the right umbilical vein. C. It contains the portal triad that enters the liver at the porta hepatis. D. It divides the liver into a left and right lobe. E. It is a remnant of the ductus venosus

The ligamentum teres is a remnant of the left umbilical vein. The right umbilical vein obliterates during the fetal period. It is empty and does not conduct any vessels. The true division of the left and right lobes of the liver is the imaginary line passing through the gallbladder and IVC.

TIMOTHY TANG LEE SAY, MD (TOP 4 - AUG 2013 MED BOARDS; TOPNOTCH MD)

MIDTERM 1 EXAM - FEB 2014

599

In doing pelvic surgery, you noticed that there is a nerve found anterior to the psoas muscle, this is probably the? A. Iliohypogastric nerve B. Ilioinguinal nerve C. Genitofemoral nerve D. Femoral nerve E. Obturator nerve

Lateral border of the psoas - iliohypogastric, ilioinguinal and femoral nerve Anterior - genitofemoral Medial - obturator

TIMOTHY TANG LEE SAY, MD (TOP 4 - AUG 2013 MED BOARDS; TOPNOTCH MD)

MIDTERM 1 EXAM - FEB 2014

600

Which of the following is NOT located in the deep perineal space in males? A. Root of the penis B. Bulbourethral gland C. Dorsal nerve of the penis D. Sphincter urethra E. Deep transverse perineal muscle

Root of the penis is found in the superficial perineal space.

TIMOTHY TANG LEE SAY, MD (TOP 4 - AUG 2013 MED BOARDS; TOPNOTCH MD)

MIDTERM 1 EXAM - FEB 2014

601

Adult venous system is derived from the following embryonic structures, EXCEPT: A. Vitelline vein B. Umbilical vein C. Cardinal vein D. Sinus venosus E. None of the above

Three Pairs of Veins are present in embryo: vitelline veins, umbilical veins, common cardinals veins. Common Cardinal Veins collect the deoxygenated blood from the body wall. Umbilical Veins carrying the oxygenated blood from the Placenta. Vitelline veins bring the deoxygenated blood from the Yolk Sac & gut

RACHELLE MENDOZA, MD (TOP 9 - AUG 2012 MED BOARDS; TOPNOTCH MD)

FINAL EXAM - FEB 2013

602

EG, 47-year old female, known hypertensive and diabetic, went to your clinic due to sudden onset of left-sided facial weakness. On examination, you noted that there was asymmetry in facial expression with lagging noted on the left side, blunting of left nasolabial fold and inability to close the left eyelid. If you are considering Bell's palsy, which of the following is consistent with this finding? A. Loss of sensation on left side of the face B. Loss of sensation on right side of the face C. Unable to wrinkle left side of the forehead D. Able to wrinkle both side of the forehead E. A and C

Bell's palsy is a peripheral facial nerve palsy, usually presenting with inability to wrinkle forehead, inability to close eyelid and lagging of facial expression on the side affected. Sensation is typically intact, since facial sensation comes from CN V (trigeminal).

RACHELLE MENDOZA, MD (TOP 9 - AUG 2012 MED BOARDS; TOPNOTCH MD)

FINAL EXAM - FEB 2013

603

A 25-year old male presents with severe colicky lumbar pain on the right. KUB was done, revealing a radioluscent stone, that is most likely lodged where? A. Mid-ureter B. Point where ureter enters the pelvic brim C. Ureteropelvic junction D. Point where ureter enters the bladder E. Any of the above

Since the pain is felt on the lumbar area, the highest area of anatomic ureteral constriction should be considered in this patient.

RACHELLE MENDOZA, MD (TOP 9 - AUG 2012 MED BOARDS; TOPNOTCH MD)

FINAL EXAM - FEB 2013

TOPNOTCH MEDICAL BOARD PREP ANATOMY SUPEREXAM Page 79 of 94 For inquiries visit www.topnotchboardprep.com.ph or email us at [email protected]

TOPNOTCH MEDICAL BOARD PREP ANATOMY SUPEREXAM For inquiries visit www.topnotchboardprep.com.ph or email us at [email protected] Item # 604

QUESTION

EXPLANATION

AUTHOR RACHELLE MENDOZA, MD (TOP 9 - AUG 2012 MED BOARDS; TOPNOTCH MD) RACHELLE MENDOZA, MD (TOP 9 - AUG 2012 MED BOARDS; TOPNOTCH MD) RACHELLE MENDOZA, MD (TOP 9 - AUG 2012 MED BOARDS; TOPNOTCH MD)

TOPNOTCH EXAM FINAL EXAM - FEB 2013

Pringle maneuver involves compression of which of the following? A. Portal vein B. Bile duct C. Hepatic artery D. All of the above

Pringle manuever is used to control bleeding during a liver procedure without compromising too much of its blood supply. It involves compression of the heaptic arty, as most of the liver's blood supply would come from the portal vein.

605

Femoral sheath encloses the following, EXCEPT: A. Femoral nerve B. Femoral artery C. Femoral vein D. Lymph vessels E. None of the above

The femoral sheath does not enclose the femoral nerve.

606

The following structures pass through the esophageal perture of the diaphragm, EXCEPT: A. Right vagus B. Left gastric vesses C. Esophagus D. Lymphatics from lower 3rd of esophagus E. None of the above

All of these pass through the esophageal aperture, alongside esophagus (T10 level)

607

A 26-year old male came from a fist fight and was brought to your clinic due to difficulty moving the eyeball. MRI revealed isolated rupture of annulus of zinn. Which of the following eye muscles whose origin would remain unaffected? A. Lateral rectus B. Superior rectus C. Inferior rectus D. Superior oblique E. None of the above

All EOMs have a common origin at the annulus of zinn except for 2: superior oblique (roof) and inferior oblique (floor)

RACHELLE MENDOZA, MD (TOP 9 - AUG 2012 MED BOARDS; TOPNOTCH MD)

FINAL EXAM - FEB 2013

608

During thyroidectomy, the first major blood vessel to be encountered is: A. Anterior jugular vein B. External jugular vein C. Internal jjugular vein D. Subclavian vein E. A and B

The anterior jugular vein descends through the midline and joins the opposite vein via the jugular notch just above the sternum. This will then join the external jugular vein deep to the SCM.

RACHELLE MENDOZA, MD (TOP 9 - AUG 2012 MED BOARDS; TOPNOTCH MD)

FINAL EXAM - FEB 2013

609

Most common blood supply of the SA node comes from the: A. Left coronary artery B. Right coronary artery C. Left circumflex artery D. Right marginal artery E. A and C

90% of the time, the SA node is supplied with a nodal branch coming from the right coronary artery.

FINAL EXAM - FEB 2013

610

If the 3rd branchial arch fails to form or mature, which of the following clinical manifestation would be observed? A. No facial expression would be appreciated B. Deafness C. Hoarseness D. Decreased secretion of parotid gland E. All of the above

The 3rd branchial arch gives rise to the greater horn of the hyoid, stylopharyngeus muscle and CN IX. Therefore, if it fails to form or mature to adult structures, patient would be expected to present with deficiency of the greater horn of the hyoid and lack of stylopharynegus muscle. And since CN IX would be absent, there would be also be poor gag reflex and decreased parotid gland secretion.

RACHELLE MENDOZA, MD (TOP 9 - AUG 2012 MED BOARDS; TOPNOTCH MD) RACHELLE MENDOZA, MD (TOP 9 - AUG 2012 MED BOARDS; TOPNOTCH MD)

611

The following layers of the anterior abdominal wall give rise to an equivalent covering of scrotal and spermatic fascia, EXCEPT: A. Subcutaneous fascia B. External oblique C. Internal oblique D. Transversus abdominis E. A and C

External oblique gives rise to external spermatic fascia. Internal oblique gives rise to cremasteric muscle and fascia. Transversalis fascia gives rise to internal spermatic fascia. Transversus abdominis does not contributw to any portion of the scrotum.

RACHELLE MENDOZA, MD (TOP 9 - AUG 2012 MED BOARDS; TOPNOTCH MD)

FINAL EXAM - FEB 2013

612

A 35-year old male presented with enlarged scrotum on the left. Examination of the scrotal content revealed appearance of "bag of worms." Which of the following clinical condition/s is/are considered? A. Varicocoele B. Orchitis C. Renal carcinoma D. B and C E. A and C

Varicocole is the engorgement of the venous drainage of the testis, the pampiniform plexus. It usually presents with enlargement of the scrotum and examination would reveal "bag of worms" appearance. Varicocoel is more commonly observed in the left scrotum, which drains to the left renal vein and is subjected to higher pressure than the right. Hence, varicocoele is usually associated with renal tumors or other pathologies that may increase renal vein pressure.

RACHELLE MENDOZA, MD (TOP 9 - AUG 2012 MED BOARDS; TOPNOTCH MD)

FINAL EXAM - FEB 2013

TOPNOTCH MEDICAL BOARD PREP ANATOMY SUPEREXAM Page 80 of 94 For inquiries visit www.topnotchboardprep.com.ph or email us at [email protected]

FINAL EXAM - FEB 2013

FINAL EXAM - FEB 2013

FINAL EXAM - FEB 2013

TOPNOTCH MEDICAL BOARD PREP ANATOMY SUPEREXAM For inquiries visit www.topnotchboardprep.com.ph or email us at [email protected] Item # 613

QUESTION

EXPLANATION

AUTHOR

TOPNOTCH EXAM FINAL EXAM - FEB 2013

Which enteroendocrine cells produce a compound that promotes insulin secretion and inhibits glucagon production? A. L cells B. S cells C. K cells D. I cells E. None of the above

I cells produce cholecystokinin, S cells produce secretin, K cells produce gastrin inhinitory peptide and L cells produce glucagon-like peptide 1 (GLP 1). The latter stimulate insulin production and inhibits glucagon secretion.

RACHELLE MENDOZA, MD (TOP 9 - AUG 2012 MED BOARDS; TOPNOTCH MD)

614

An athlete suffered from fracture of his left medial malleolus, transecting all structures behind it. Which of the following will NOT be affected? A. Tibialis posterior B. Posterior tibial artery C. Posterior tibial nerve D. Flexor hallucis longus E. Deep peroneal nerve

Structures running behind the medial malleolus: tibialis poetrior, flexor digitorum longus, posterior tibial artery, posterior tibial nerve and flexor hallocis longus.

RACHELLE MENDOZA, MD (TOP 9 - AUG 2012 MED BOARDS; TOPNOTCH MD)

FINAL EXAM - FEB 2013

615

Tranverse fracture of the humeral mid-shaft would result to: A. Loss of sensation on anterior forearm B. Claw hand C. Ape hand D. Inability to extend wrist E. A and C

Tranverse fracture of the mid-shaft of the humerus would injure the deep brachial artery and the radial nerve. The radial nerve typically supplies posterior compartments of arm and forearm and thereby resposible for allowing extension of elbow and wrist.

FINAL EXAM - FEB 2013

616

A stroke patient was found to have a necrotic focus on the brain, specifically identified as Brodmann areas 44 and 45. The patient is then expected to: A. have difficulty in speaking but can comprehend B. have difficulty understanding but can speak C. not be able to hear and interpret sounds D. not be able to associate visual input to other information E. none of the above

Brodmann areas 44 and 45 correspond to Broca's speech area.

RACHELLE MENDOZA, MD (TOP 9 - AUG 2012 MED BOARDS; TOPNOTCH MD) RACHELLE MENDOZA, MD (TOP 9 - AUG 2012 MED BOARDS; TOPNOTCH MD)

617

A trauma patient was found to have total obliteration of foramen rotundum. Which of the following clinical condition would be a finding in this patient? A. Loss of sensation along the maxillary area of the face B. Loss of sensation along the mandibular area of the face C. Epidural hemorrhage D. A and C E. B and C

CN V-2 (maxillary branch of the trigeminal nerve) passes through foramen rotundum. The maxillary branch is resposnible for somatic sensory supply of the maxillary area of the face and mouth. CN V3 passes through foramen ovale, while the middle meningeak artery passes through foramen spinosum.

RACHELLE MENDOZA, MD (TOP 9 - AUG 2012 MED BOARDS; TOPNOTCH MD)

FINAL EXAM - FEB 2013

618

The widest and most dilatable portion of the male urethra: A. Membranous B. Spongy C. Penile D. Prostatic E. B and C

Prostatic urethra is the widest and most dilatable. Membranous part is the shortest and least dilatable. The penile/spongy urethra is the narrowest part of the entire urethre.

FINAL EXAM - FEB 2013

619

Arthrocentesis is most difficult to carry out in which joint? A. Knee B. Elbow C. Ankle D. Wrist E. A and C

Arthrocentesis of the ankle is more difficult than that of the other joints. The knee is the most common and the easiest joint for the physician to aspirate

620

Histopath cross-section of the gallbladder shows the following features, EXCEPT: A. Tall columnar epithelium with microvilli B. Muscularis mucosae C. Muscularis D. Adventitia E. A and C

The gallbladder is lined with tall columnar cells with microvilli, except the terminal ducts (cuboidal epithelium). No muscularis mucosae can be found.

RACHELLE MENDOZA, MD (TOP 9 - AUG 2012 MED BOARDS; TOPNOTCH MD) RACHELLE MENDOZA, MD (TOP 9 - AUG 2012 MED BOARDS; TOPNOTCH MD) RACHELLE MENDOZA, MD (TOP 9 - AUG 2012 MED BOARDS; TOPNOTCH MD)

621

A part of anterior pituitary gland which has a small collar cells around infundibular stalk that mainly contains the portal venules. A. Pars Distalis B. Pars Intermedia C. Pars Tuberalis D. Pars nervosa E. None of the above



VON ANDRE MEDINA, MD (TOP 4 - FEB 2012 MED BOARDS; TOPNOTCH MD)

DIAGNOSTIC EXAM - AUG 2012

TOPNOTCH MEDICAL BOARD PREP ANATOMY SUPEREXAM Page 81 of 94 For inquiries visit www.topnotchboardprep.com.ph or email us at [email protected]

FINAL EXAM - FEB 2013

FINAL EXAM - FEB 2013

FINAL EXAM - FEB 2013

TOPNOTCH MEDICAL BOARD PREP ANATOMY SUPEREXAM For inquiries visit www.topnotchboardprep.com.ph or email us at [email protected] Item # 622

QUESTION

EXPLANATION

AUTHOR

TOPNOTCH EXAM DIAGNOSTIC EXAM - AUG 2012

Which portion or zone of the liver acinus is exposed to blood high in oxygen content and nutrients , and is primarily involved in the pathology of preeclampsia /eclampsia? A. Centrilobular B. Periportal C. Midzonal D. Space of Disse E. Sinusoids

periportal= exposed to blood high in O2, involved in eclampsia Midzonal= intermediate, involved in yellow fever centrilobular= exposed to blood low in O2, involved in ischemic injury, right sided heart failure

VON ANDRE MEDINA, MD (TOP 4 - FEB 2012 MED BOARDS; TOPNOTCH MD)

623

The inferior parathyroid glands are derived from which branchial pouch? A. 1st B. 2nd C. 3rd D. 4th E. A and B

In humans, the superior parathyroid glands are derived from the 4th branchial pouch while the inferior parathyroid glands come from 3rd branchial pouch.

VON ANDRE MEDINA, MD (TOP 4 - FEB 2012 MED BOARDS; TOPNOTCH MD)

624

Diaphragm is the most important muscle of inspiration. Which embryonic structure gives rise to the central tendon of the diaphragm which is fused partially with the fibrous pericardium of the heart? A. Pleuroperitoneal membrane B. Dorsal mesentery of the esophagus C. Septum transversum D. Body wall mesenchyme E. All of the above

embryonic origin VON ANDRE DIAGNOSTIC Septum transversum= central tendon MEDINA, MD EXAM - AUG Pleuroperitoneal membrane= bulk of muscular (TOP 4 - FEB 2012 part of diaphragm Body wall mesenchyme= 2012 MED peripheral muscular part of the diaphragm BOARDS; dorsal mesentery= diphragmatic crura TOPNOTCH MD)

625

This structure encloses the kidney and suprarenal gland and blends with the fascia of IVC and aorta. A. Fibrous capsule B. Perirenal fat C. Gerota's fascia D. Renal fascia E. C and D

renal fascia is also known as gerota's fascia

VON ANDRE MEDINA, MD (TOP 4 - FEB 2012 MED BOARDS; TOPNOTCH MD)

DIAGNOSTIC EXAM - AUG 2012

626

A 46 year old patient came to emergency department due to gunshot wound. During physical examination, it was found out that the bullet entered the sternal area. Which chamber of the heart will most probably be injured in this case? A. Right atrium B. Right ventricle C. Left ventricle D. Left atrium E. Diaphragm

sternal border= right ventricle

VON ANDRE MEDINA, MD (TOP 4 - FEB 2012 MED BOARDS; TOPNOTCH MD)

DIAGNOSTIC EXAM - AUG 2012

627

An aneurysm of the abdominal aorta at the aortic hiatus of the diaphragm is most likely to result in the compression of which of the following pairs of structures? A. Vagus nerve and azygous vein B. Esophagus and vagus nerve C. Azygous vein and thoracic duct D. Thoracic duct and vagus nerve E. Inferior vena cava and phrenic nerve

the aortic hiatus of the diaphragm transmits the azygous vein and thoracic duct.

VON ANDRE MEDINA, MD (TOP 4 - FEB 2012 MED BOARDS; TOPNOTCH MD)

DIAGNOSTIC EXAM - AUG 2012

628

During appendectomy performed at McBurney's point, which of the following structures is most likely to be injured? A. Deep circumflex femoral artery B. Inferior epigastric artery C. Iliohypogastric nerve D. Genitofemoral nerve E. Spermatic cord

Iliohypogastric nerve runs medially and inferiorly between the internal oblique and transverse abdominal muscles at Mcburney's point.

VON ANDRE MEDINA, MD (TOP 4 - FEB 2012 MED BOARDS; TOPNOTCH MD)

DIAGNOSTIC EXAM - AUG 2012

629

As the uterine artery passes from the internal iliac artery to the uterus, it crosses which of the following structures that is sometimes mistakenly ligated during pelvic surgery? A. Ovarian artery B. Ovarian ligament C. Uterine tube D. Ureter E. Round ligament of the uterus

the ureter runs under the uterine artery near the cervix; thus, the ureter is sometimes mistakenly ligated during pelvic surgery

VON ANDRE MEDINA, MD (TOP 4 - FEB 2012 MED BOARDS; TOPNOTCH MD)

DIAGNOSTIC EXAM - AUG 2012

TOPNOTCH MEDICAL BOARD PREP ANATOMY SUPEREXAM Page 82 of 94 For inquiries visit www.topnotchboardprep.com.ph or email us at [email protected]

DIAGNOSTIC EXAM - AUG 2012

TOPNOTCH MEDICAL BOARD PREP ANATOMY SUPEREXAM For inquiries visit www.topnotchboardprep.com.ph or email us at [email protected] Item # 630

QUESTION

EXPLANATION

AUTHOR

TOPNOTCH EXAM DIAGNOSTIC EXAM - AUG 2012

During a game, a 25 year old baseball player receives a severe blow to the head that fractures the optic canal. Which of the following pairs of structures is most likely to be damaged? A. Optic nerve and ophthalmic vein B. Ophthalmic vein and ophthalmic nerve C. Ophthalmic artery and and optic nerve D. Ophthalmic nerve and optic nerve E. Ophthalmic artery and ophthalmic vein



VON ANDRE MEDINA, MD (TOP 4 - FEB 2012 MED BOARDS; TOPNOTCH MD)

631

What is the lining epithelium of the endocervical canal? A. Simple columnar B. Simple squamous C. Stratified squamous D. Cuboidal E. Pseudostratified columnar

the cervix has a simple columnar epithelium, except for the external portion which is continous with the vagina which has a stritified squamous epithelium

VON ANDRE MEDINA, MD (TOP 4 - FEB 2012 MED BOARDS; TOPNOTCH MD)

DIAGNOSTIC EXAM - AUG 2012

632

Communication of the scala vestibuli and scala tympani occurs at the: A. Oval window B. Round window C. Helicotrema D. Endolymphatic sac E. None of the above

the scala vestibuli and the scala tympani are actually one perilymphatic space separated by the cochlear duct (scala media). The scala vestibuli and tympani communicate with each other at helicotrema.

VON ANDRE MEDINA, MD (TOP 4 - FEB 2012 MED BOARDS; TOPNOTCH MD)

DIAGNOSTIC EXAM - AUG 2012

633

Which of the following is a characteristic of the cornea? A. It represents the anterior portion of the tunica vasculosa B. It is the anterior transparent portion of the tunica fibrosa C. It forms the anterior boundary of the posterior chamber of the eye D. It is devoid of nerve endings E. All of the above

the cornea is the transparent anterior portion of the tunica fibrosa, the outer covering of the eye; thus it forms the anterior wall of the anterior chamber of the eye.

VON ANDRE MEDINA, MD (TOP 4 - FEB 2012 MED BOARDS; TOPNOTCH MD)

DIAGNOSTIC EXAM - AUG 2012

634

In a surgical procedure in which you are to remove the gallbladder and therefore you need to ligate the cystic artery. This artery arises from which of the following? A. Proper hepatic artery B. Right hepatic artery C. Left hepatic artery D. Right gastric artery E. Common hepatic artery

Cystic artery arises mainly from right hepatic artery

VON ANDRE MEDINA, MD (TOP 4 - FEB 2012 MED BOARDS; TOPNOTCH MD)

DIAGNOSTIC EXAM - AUG 2012

635

Mrs. Succor suffered from an embolic infarct. She came in to the emergency department complaining of difficulty in speaking and left arm weakness. Which of the following arteries is most likely affected? A. Middle cerebral artery B. Anterior cerebral artery C. Vertebral artery D. Posterior cerebal artery E. Basilar artery



VON ANDRE MEDINA, MD (TOP 4 - FEB 2012 MED BOARDS; TOPNOTCH MD)

DIAGNOSTIC EXAM - AUG 2012

636

The following veins drain into the coronary sinus EXCEPT: A. Great cardiac vein B. Small cardiac vein C. Middle cardiac vein D. Anterior cardiac vein E. None of the above

great cardiac vein, small cardiac vein and middle cardiac vein drain into the coronary sinus.

VON ANDRE MEDINA, MD (TOP 4 - FEB 2012 MED BOARDS; TOPNOTCH MD)

DIAGNOSTIC EXAM - AUG 2012

637

Mrs. Jennifer Marikina is suffering from an ovarian malignancy with hepatic metastasis. After 3 days, you noticed that her abdomen is getting bigger. On physical examination, there is a positive fluid wave which is highly suggestive that the patient has ascites. Her attending physician decided to do a paracentesis. The PREFERRED entry site for paracentesis is? A. Halfway between the umbilicus and the pubic symphysis B. Between umbilicus and ASIS, lateral to rectus abdominis muscle, Left C. Between umbilicus and ASIS, lateral to the restus abdominis muscle, Right D. Between umbilicus and ASIS, medial to the Rectus abdominis E. Any of the above



VON ANDRE MEDINA, MD (TOP 4 - FEB 2012 MED BOARDS; TOPNOTCH MD)

DIAGNOSTIC EXAM - AUG 2012

TOPNOTCH MEDICAL BOARD PREP ANATOMY SUPEREXAM Page 83 of 94 For inquiries visit www.topnotchboardprep.com.ph or email us at [email protected]

TOPNOTCH MEDICAL BOARD PREP ANATOMY SUPEREXAM For inquiries visit www.topnotchboardprep.com.ph or email us at [email protected] Item # 638

QUESTION

EXPLANATION

AUTHOR

TOPNOTCH EXAM DIAGNOSTIC EXAM - AUG 2012

A 58 year old G8P8 underwent TAHBSO with bilateral lymph node dissection for the high grade cervical cancer. Few hours post-surgery, she complained that she cannot feel her right inner leg. On your physical examination, impaired right thigh adduction was observed. The iatrogenically damaged structure is most likely the: A. Femoral nerve B. Sciatic nerve C. Obturator nerve D. Pudendal nerve E. Superior gluteal nerve



VON ANDRE MEDINA, MD (TOP 4 - FEB 2012 MED BOARDS; TOPNOTCH MD)

639

The following structures are parts of the osteon, EXCEPT: A. Osteocytes B. Concentric Lamellae C. Canaliculi D. Volkmann's canal E. A and B



VON ANDRE MEDINA, MD (TOP 4 - FEB 2012 MED BOARDS; TOPNOTCH MD)

DIAGNOSTIC EXAM - AUG 2012

640

The structure which has a head and a base that are united by two limbs and is connected to the oval window is the? A. Incus B. Malleus C. Stapes D. Stapedius E. Tensor Tympani

SIMILAR TO PREVIOUS BOARD EXAM CONCEPT/PRINCIPLE

VON ANDRE MEDINA, MD (TOP 4 - FEB 2012 MED BOARDS; TOPNOTCH MD)

DIAGNOSTIC EXAM - AUG 2012

641

All of the following are contained in the vertebral canal, EXCEPT? A. Spinal cord B. Spinal nerve C. Dorsal nerve roots D. Ventral nerve roots E. Meninges

Spinal Nerve is located OUTSIDE the vertebral canal (exit throught intervertebral foramen)

LITO JAY MACARAIG, MD (TOP 8 - FEB 2013 MED BOARDS; TOPNOTCH MD)

DIAGNOSTIC EXAM - AUG 2013

642

Which of the following is/are true regarding the "yes" joint? A. Atlanto-occipital joint B. Atlanto-axial joint C. Joint between C1 and occipital condyles D. Joint between C1 and C2 E. choices A and C are both correct F. choices B and D are both correct

The atlanto-occipital joint is found between the C1 and occipital condyles. It is also called the "yes joint" because it facilitates extension and flexion of the neck.

LITO JAY MACARAIG, MD (TOP 8 - FEB 2013 MED BOARDS; TOPNOTCH MD)

DIAGNOSTIC EXAM - AUG 2013

643

The vertebral levels are used as reference points for locating anatomical landmarks. Which among the following choices is/are found at the level of C4 vertebra? A. Hyoid bone B. Start of trachea C. Cricoid cartilage D. choices A and B are both correct E. choices B and C are both correct

At the level of C4 vertebra, hyoid bone and bifurcation of common carotid artery are found. The start of trachea and Cricoid cartilage are found at the level of C6 vertebra.

LITO JAY MACARAIG, MD (TOP 8 - FEB 2013 MED BOARDS; TOPNOTCH MD)

DIAGNOSTIC EXAM - AUG 2013

644

This is a condition which resulted from chronic stress fracture of Pars interarticularis seen as "collar around the neck of Scottie dog" on Xray. Usually at the L5 vertebra of adolescent athletes. A. Spondylosis B. Spondylolisthesis C. Ankylosing spondylitis D. Osteoyelitis E. Herniation of Nucleus Pulposus

Spondylolisthesis involves degeneration of pedicles of lumbar vertebra. Osteomyelitis involves infection within vertebral bodies. AS involves the bamboo spine deformity. HNP is the protrusion of Nucleus Pulposus due to a break on the annulus fibrosus.

LITO JAY MACARAIG, MD (TOP 8 - FEB 2013 MED BOARDS; TOPNOTCH MD)

DIAGNOSTIC EXAM - AUG 2013

645

During an ER duty, a 24 y/o male patient was brought in due to seizure. History revealed severe ear pain 2 weeks prior to consult, followed by occasional headaches. Four days PTC, patient complained of fever 38-39 degrees Celsius. Upon PE, patient had (+) meningeal signs. You are entertaining bacterial meningitis. What vertebral level is ideal to insert the needle for Lumbar Tap? A. between L1 and L2 B. between L2 and L3 C. between L3 and L4 D. choices A and B are both correct E. choices B and C are both correct

The ideal site for lumbar puncture is 1 level above or below the spinous process of L4 vertebra (between L3-L4 or between L4-L5).

LITO JAY MACARAIG, MD (TOP 8 - FEB 2013 MED BOARDS; TOPNOTCH MD)

DIAGNOSTIC EXAM - AUG 2013

TOPNOTCH MEDICAL BOARD PREP ANATOMY SUPEREXAM Page 84 of 94 For inquiries visit www.topnotchboardprep.com.ph or email us at [email protected]

TOPNOTCH MEDICAL BOARD PREP ANATOMY SUPEREXAM For inquiries visit www.topnotchboardprep.com.ph or email us at [email protected] Item # 646

QUESTION

EXPLANATION

AUTHOR

TOPNOTCH EXAM DIAGNOSTIC EXAM - AUG 2013

A 32 year-old male patient was brought in the ER due to stabbing. Patient was hypotensive and bleeding profusely from the chest. Despite all efforts of resuscitation, patient subsequently expired. If the wound was located between the 4th and 5th rib Left parasternal line, the chamber of the heart that is most likely affected was? A. Left Atrium B. Left Ventricle C. Right Atrium D. Right Ventricle

The Right Ventricle is the chamber of the heart that lies directly behind the sternum.

LITO JAY MACARAIG, MD (TOP 8 - FEB 2013 MED BOARDS; TOPNOTCH MD)

647

Volksmann Ischemic Contracture is characterized by severe flexion of the affected arm, wrist and fingers. The following structures are affected EXCEPT? A. Midshaft of humerus B. Brachial artery C. Median nerve D. A and B E. B and C

In a Volksmann ischemic contracture, the fracture is found at the supracondylar area of humerus and NOT at the midshaft.

LITO JAY MACARAIG, MD (TOP 8 - FEB 2013 MED BOARDS; TOPNOTCH MD)

DIAGNOSTIC EXAM - AUG 2013

648

The following are the boundaries of the Anatomical Snuffbox, EXCEPT? A. Extensor Pollicis Longus B. Extensor Pollicis Brevis C. Abductor Pollicis Longus D. Abductor Pollicis Brevis E. There is NO exception

The first three choices are the ONLY boundaries of the Anatomical Snuffbox.

LITO JAY MACARAIG, MD (TOP 8 - FEB 2013 MED BOARDS; TOPNOTCH MD)

DIAGNOSTIC EXAM - AUG 2013

649

These granulocytes are described with "bilobulated nucleus" and plays roles in fighting viral infections, as proved by the RNAses they contain. A. Basophil B. Eosinophil C. Neutrophil D. NK cells E. Lymphocytes

Eosinophils are characterized by their bilobulated nucleus with numerous pink-orange granules. They are highly involved in Asthma pathophysiology and Helminthic infections. They contain RNAses which proves their effectivity against viral infections.

LITO JAY MACARAIG, MD (TOP 8 - FEB 2013 MED BOARDS; TOPNOTCH MD)

DIAGNOSTIC EXAM - AUG 2013

650

While jogging, a 40 y/o male was bumped by a tricycle on the lateral side of his left leg. Patient was unable to evert his left foot, with weakness in plantar flexion but normal dorsiflexion. What nerve was affected? A. Common Peroneal Nerve B. Superficial Peroneal Nerve C. Deep Peroneal Nerve D. Tibial Nerve E. Common Fibular Nerve

The common peroneal nerve is the root of superficial and deep peroneal nerve. If it was affected, signs of deep peroneal nerve lesion must also be present (like weak or no dorsiflexion). The superficial peroneal nerve is responsible for eversion and plantar flexion. Tibial nerve is for plantar flexion only. Common fibular nerve is the other term for common peroneal nerve.

LITO JAY MACARAIG, MD (TOP 8 - FEB 2013 MED BOARDS; TOPNOTCH MD)

DIAGNOSTIC EXAM - AUG 2013

651

You are assigned in the Neuro-OPD and saw a patient who presented with gait instability. Patient was NOT able to perform the "heel-to-shin" movement. You know that Cerebellum is involved. But what lobe? A. Anterior lobe B. Flocculo-nodular lobe C. Posterior lobe D. Archi-cerebellum E. Neo-cerebellum

There are 3 anatomical lobes of the cerebellum. The anterior lobe (paleo-cerebellum), Flocculonodular lobe (archi-cerebellum), and the posterior lobe (neo-cerebellum). Anterior lobe lesions will result to gait instability and patient will be unable to perform the heel-to-shin movement.

LITO JAY MACARAIG, MD (TOP 8 - FEB 2013 MED BOARDS; TOPNOTCH MD)

DIAGNOSTIC EXAM - AUG 2013

652

Which among the following spinal cord lesions will present as both upper and lower motor neuron lesion? A. Poliomyelitis B. Amyotrophic Lateral Sclerosis C. Brown-sequard D. Choices A and B are both correct E. Choices B and C are both correct

In ALS and Brown-Sequard (hemisection), both the cortico-spinal tract (Upper MN) and the anterior horn cells (Lower MN) are affected.

LITO JAY MACARAIG, MD (TOP 8 - FEB 2013 MED BOARDS; TOPNOTCH MD)

DIAGNOSTIC EXAM - AUG 2013

653

The muscles of facial expression are derived from? A. 1st Pharyngeal Pouch B. 1st Pharyngeal Arch C. 2nd Pharyngeal Pouch D. 2nd Pharyngeal Arch E. None of the above

Derivatives of pharyngeal Arch: 1. Muscles of Mastication, 2. Muscles pf Facial expression, 3. Muscles of deglutition, 4-6. Muscles for Phonation

LITO JAY MACARAIG, MD (TOP 8 - FEB 2013 MED BOARDS; TOPNOTCH MD)

DIAGNOSTIC EXAM - AUG 2013

TOPNOTCH MEDICAL BOARD PREP ANATOMY SUPEREXAM Page 85 of 94 For inquiries visit www.topnotchboardprep.com.ph or email us at [email protected]

TOPNOTCH MEDICAL BOARD PREP ANATOMY SUPEREXAM For inquiries visit www.topnotchboardprep.com.ph or email us at [email protected] Item # 654

QUESTION

EXPLANATION

AUTHOR

TOPNOTCH EXAM DIAGNOSTIC EXAM - AUG 2013

The internal carotid artery (ICA) supplies the anterior part of the brain through the Circle of Willis. The ICA is a derivative of? A. Proximal Part of the 3rd Aortic arch B. Distal Part of the 3rd Aortic Arch C. Right part of the 4th Aortic Arch D. Left part of the 4th Aortic Arch E. None of the above

ICA is a derivative of the distal part of the 3rd Aortic Arch. The proximal part will give rise to the Common carotid artey.

LITO JAY MACARAIG, MD (TOP 8 - FEB 2013 MED BOARDS; TOPNOTCH MD)

655

The genioglossus is an extrinsic muscle of the tongue and is responsible for what movement? A. elevation B. depression C. protrusion D. retraction E. None of the above

The genioglossus is innervated by CN XII and protrudes the tongue.

LITO JAY MACARAIG, MD (TOP 8 - FEB 2013 MED BOARDS; TOPNOTCH MD)

DIAGNOSTIC EXAM - AUG 2013

656

A patient with Mallory Weiss tear came to ER due to severe hematemesis, pulsating in character. Knowing the diagnosis, you already know that the bleeding blood vessel is? A. Middle esophageal artery B. Middle esophageal vein C. Left gastric artery D. Left gastric vein E. Descending aorta

The Mallory Weiss tear is most commonly located at the gastro-esophageal junction. The bleeding is described as "pulsating", hence arterial. And the blood supply of the distal 3rd of esophagus is the Left gstric artery which is a branch of the celiac trunk.

LITO JAY MACARAIG, MD (TOP 8 - FEB 2013 MED BOARDS; TOPNOTCH MD)

DIAGNOSTIC EXAM - AUG 2013

657

Which among the following statements is/are true regarding the difference between Jejunum and Ileum? A. Jejunum is shorter B. Ileum has more fat C. Jejunum is thicker D. B and C only E. All of the above

Jejunum is shorter, thicker, more vascular and with long vasa recta. Ileum has more fat, arcades, and lymph nodes.

LITO JAY MACARAIG, MD (TOP 8 - FEB 2013 MED BOARDS; TOPNOTCH MD)

DIAGNOSTIC EXAM - AUG 2013

658

Which among the following statements is/are true regarding the round ligament of the liver? A. AKA falciform ligament B. AKA ligamentum teres C. From the Left umbilical vein D. A and C only E. B and C only

The round ligament of the liver is AKA the Ligamentum Teres. It is a derivative from the left umbilical vein. The right umbilical vein obliterates without any derivative in the later stage of life.

LITO JAY MACARAIG, MD (TOP 8 - FEB 2013 MED BOARDS; TOPNOTCH MD)

DIAGNOSTIC EXAM - AUG 2013

659

The rectum is divided into three (3) anatomical divisions. And its inferior part (distal 3rd) is being supplied by a direct branch of? A. Inferior Mesenteric Artery B. Sigmoidal artery C. Internal pudendal artery D. A and C only E. All of the above

the superior rectal artery is from IMA, the middle rectal artery is from the Internal iliac artery, and the inferior rectal artery is from the the internal pudendal artery which is also a branch of the internal iliac artery

LITO JAY MACARAIG, MD (TOP 8 - FEB 2013 MED BOARDS; TOPNOTCH MD)

DIAGNOSTIC EXAM - AUG 2013

660

Which among the following statements is/are true regarding the anatomy of the kidney? A. 12 pyramids B. 2 major calyces C. 3 minor calyces D. A and C only E. All of the above

A normal adult kidney contains 12 pyramids. 2 major calyces and 3 minor calyces.

LITO JAY MACARAIG, MD (TOP 8 - FEB 2013 MED BOARDS; TOPNOTCH MD)

DIAGNOSTIC EXAM - AUG 2013

661

Which of the following is bilobed? A. neutrophils B. basophils C. lymphocytes D. monocytes E. Platelets

Basophils and Eosinophils have nuclei that are bilobed. Neutrophils are multinucleated.

HAZEL KAREN RAZ, MD (TOP 6 - FEB 2013 MED BOARDS; TOPNOTCH MD)

MIDTERM 2 - AUG 2013

TOPNOTCH MEDICAL BOARD PREP ANATOMY SUPEREXAM Page 86 of 94 For inquiries visit www.topnotchboardprep.com.ph or email us at [email protected]

TOPNOTCH MEDICAL BOARD PREP ANATOMY SUPEREXAM For inquiries visit www.topnotchboardprep.com.ph or email us at [email protected] Item # 662

QUESTION

EXPLANATION

AUTHOR

TOPNOTCH EXAM MIDTERM 2 - AUG 2013

Which of the following statements is true? A. Bronchioles have cartilage plates similar to those of bronchi. B. Bronchioles have greater amount of smooth muscle than bronchi. C. Bronchioles are formed when the airways are reduced to a diameter of < 1 cm. D. Bronchioles have greater number of mucous glands than bronchi. E. None of the above.

Bronchioles have lesser cartilage and mucous glands than bronchi. However, smooth muscle cells are greater in the distal airways. Bronchioles are formed when the airways are reduced to < 1 mm.

HAZEL KAREN RAZ, MD (TOP 6 - FEB 2013 MED BOARDS; TOPNOTCH MD)

663

The following are found inside the lacunae, except? A. Osteocytes B. Osteoclast C. Osteoblast D. Chrondrocyte E. None of the above

Osteoblast are found outside of lacunar cells in bone. They are responsible for bone formation.Osteoblasts that become trapped in the bone matrix and remain isolated in lacunae become osteocytes.

MIDTERM 2 - AUG 2013

664

Protein production for extracellular distribution are produced in the: A. Smooth ER B. Rough ER C. Free ribosomes D. Nucleus E. Lysozyme

Protein produced from the RER are transported out of the cell for ectracellular use. Free ribosomes produce protein for intracellular use. Smooth ER functions in cholesterol synthesis and all other food products except complex carbohydrates. Lysozyme produces enzymes for self - desctruction.

HAZEL KAREN RAZ, MD (TOP 6 - FEB 2013 MED BOARDS; TOPNOTCH MD) HAZEL KAREN RAZ, MD (TOP 6 - FEB 2013 MED BOARDS; TOPNOTCH MD)

665

In the respiratory tract, what differentiates the terminal bronchioles from the respiratory bronchioles? A. The presence of alveoli in respiratory bronchioles B. Presence of greater number of smooth muscle in respiratory bronchioles C. Presence of greater number of cilia in respiratory bronchioles D. Lesser elastic fibers in respiratory bronchioles E. All of the above

Respiratory bronchioles begin where alveoli start to appear. There are lesser smooth muscle, cilia and greater amount of elastic fiber.

HAZEL KAREN RAZ, MD (TOP 6 - FEB 2013 MED BOARDS; TOPNOTCH MD)

MIDTERM 2 - AUG 2013

666

Cancer of the esophagus presents with early metastasis due to the absence of which layer? A. epithelium B. submucosa C. Muscularis propria D. Lamina propria E. Serosa

Esophagus lacks serosal layer, that is why esophageal cancer when diagnosed is usually in the advanced stages due to early dissemination and metastasis.

HAZEL KAREN RAZ, MD (TOP 6 - FEB 2013 MED BOARDS; TOPNOTCH MD)

MIDTERM 2 - AUG 2013

667

A 15- year old triathlethe complains of low back pain which is aggravated by activity. Physical examination was normal, no neurological symptoms were noted. After x-ray of his lumbar spine, he was diagnosed to have spondylolysis. What could be the finding which led to this diagnosis? A. Degeneration of pedicles of lumbar vertebrae B. Formation of bone spurs and osteophytes C. Ossification of annulus fibrosus D. Scottie dog collar lesion on L5 E. None of the above

a - spondylolisthesis, b - spondylosis, c - ankylosing spondylitis

HAZEL KAREN RAZ, MD (TOP 6 - FEB 2013 MED BOARDS; TOPNOTCH MD)

MIDTERM 2 - AUG 2013

668

A patient came to you presenting with fever, nuchal rigidity and changes in sensorium. Meningitis was among the differentials. Lumbar tap was contemplated. At what level is lumbar tap usually performed? A. T12 - L1 B. L1 - L2 C. L3 D. L4 - L5 E. None of the above

A lumbar puncture is a puncture into the subarachnoid space of the spinal cord to obtain cerebrospinal fluid (CSF) for clinical investigation, to remove excess fluid or inject medication. Lumbar tap is performed between L3 - L4 or L4 and L-5.

HAZEL KAREN RAZ, MD (TOP 6 - FEB 2013 MED BOARDS; TOPNOTCH MD)

MIDTERM 2 - AUG 2013

669

A trauma patient presented with difficulty of breathing. CXR showed blunting of the right costophrenic angle. You decided to do a CTT insertion to drain the fliud. At what level would you insert the CTT tube? A. 4th - 5th ICS R Anterior Axillary Line directed superiorly B. 4th -5th ICS R Mid - axillary line directed inferiorly. C. 5th - 6th ICS R Mid- axillary line directed superiorly D. 5th - 6th ICS R Anterior Axillary Line directed inferiorly E. None of the above

CTT insertion is done at the 5th - 6th anterior axillary line and directed inferiorly to drain blood or fluid from the thoracic cavity. The tube is directed superiorly to drain pneumothorax.

HAZEL KAREN RAZ, MD (TOP 6 - FEB 2013 MED BOARDS; TOPNOTCH MD)

MIDTERM 2 - AUG 2013

TOPNOTCH MEDICAL BOARD PREP ANATOMY SUPEREXAM Page 87 of 94 For inquiries visit www.topnotchboardprep.com.ph or email us at [email protected]

MIDTERM 2 - AUG 2013

TOPNOTCH MEDICAL BOARD PREP ANATOMY SUPEREXAM For inquiries visit www.topnotchboardprep.com.ph or email us at [email protected] Item # 670

QUESTION

EXPLANATION

AUTHOR

Amphiarthrosis: A. Symphysis pubis B. Radio - ulnar joint C. Atlanto - axial joint D. wrist E. Distal interphalangeal joints

Joints are classified based on their mobility and composition. They are classified as synarthrosis (fibrous) - immovable, amphiarthrosis (cartilagenous) - slightly movable, diarthrosis (synovial) - freely movable. Symphysis pubis is a fibrocartilagenous joint that is slightly movable.

671

This muscle is considered the main flexor of the forearm and its innervation. A. Coracobrachialis muscle, musculocutaneous nerve B. Biceps brachii muscle, musculocuteneous nerve C. Brachialis muscle, musculocutaneous nerve D. Deltoid muscle, musculocutaneous nerve E. None of the above

The muscles innervated by musculocutaneous nerve includes the following: coracobrachialis (flexes and adducts ARM), biceps brachii (flexes and main supinator of FOREARM), brachialis (main flexor of the FOREARM). Deltoid muscle is innervated by axillary nerve, a part of the rotator cuff muscles.

672

Psoas sign is considered positive if there is_____________? A. Increased tenderness at the RLQ with palpation of the LLQ B. Pain at the RLQ with hip flexion and internal rotation of the leg C. Pain on passive extension of the right thigh D. Pain on slow compression of the abdominal wall with rapid release E. All of the above

psoas sign is a medical sign that indicates irritation to the iliopsoas group of hip flexors in the abdomen, and consequently indicates that the inflamed appendix is retrocaecal in orientation (as the iliopsoas muscle is retroperitoneal). It is elicited by performing the psoas test by passively extending the thigh of a patient lying on his side with knees extended, or asking the patient to actively flex his thigh at the hip.

HAZEL KAREN RAZ, MD (TOP 6 - FEB 2013 MED BOARDS; TOPNOTCH MD)

MIDTERM 2 - AUG 2013

673

A complication of radical mastectomy which presents as difficulty in horizontal extension of the upper extermity involves injury to the? A. Long thoracic nerve B. Intercostobrachial nerve C. Medial pectoral nerve D. Thoracodorsal nerve E. Axillary nerve

Injury to the long thoracic nerve produces a winged scapula due to paralysis of serratus anterior. Intercostobrachial nerve involvement causes loss of sensation of the upper inner arm. Paralysis of the pectoralis major muscle with weakness of abduction is due to damaged medial pectoral nerve.

HAZEL KAREN RAZ, MD (TOP 6 - FEB 2013 MED BOARDS; TOPNOTCH MD)

MIDTERM 2 - AUG 2013

674

A 60 year old smoker presents with hemoptysis, weight loss, weakness of the upper limbs, fatigue and difficulty in swallowing. Electrolyte levels show hyponatremia. What could be the possible diagnosis for this patient? A. Oat cell CA of the lung B. Adenocarcinoma of the lung C. Squamous cell CA D. PTB E. None of the above

The signs and symptoms presented are due to the paraneoplastic syndromes accompanying oat cell/small cell carcinoma of the lung. Lambert - Eaton Syndrome presents with muscle weakness, fatigue and difficulty in swallowing. SIADH is manifested by hyponatremia (dilutional), and ACTH production. Smoking is a major predisposing factor in small cell and squamous cell CA.

HAZEL KAREN RAZ, MD (TOP 6 - FEB 2013 MED BOARDS; TOPNOTCH MD)

MIDTERM 2 - AUG 2013

675

Lingula is the embryonic counterpart of which lobe of the lung? A. Right lower lobe B. Right upper lobe C. Right middle lobe D. Left Lower lobe E. Left Upper lobe

Lingula is used to denote a projection of the upper lobe of the left lung that serves as the homologue of the right middle lobe.

HAZEL KAREN RAZ, MD (TOP 6 - FEB 2013 MED BOARDS; TOPNOTCH MD)

MIDTERM 2 - AUG 2013

676

The inferior diaphragmatic surface of the heart is composed mainly of the______? A. Left atrium B. Left ventricle C. Right atrium D. Right Ventricle E. Base of the heart

Base/Posterior surface - Left atrium; Apex: LV at 5th ICS MCL; Sternal border: RV; Diaphragmatic surface: LV

HAZEL KAREN RAZ, MD (TOP 6 - FEB 2013 MED BOARDS; TOPNOTCH MD)

MIDTERM 2 - AUG 2013

677

Leaflets of the valves which separates the right atrium form the right ventricle except: A. anterior B. septal C. medial D. posterior E. None of the above

leaflets of the tricuspid valve: anterior, posterior, septal.

HAZEL KAREN RAZ, MD (TOP 6 - FEB 2013 MED BOARDS; TOPNOTCH MD)

MIDTERM 2 - AUG 2013

Paracentesis sites: 2 cms below the umbillicus, lateral border of the rectus abdominis muscle superior and medial to the ASIS

HAZEL KAREN RAZ, MD (TOP 6 - FEB 2013 MED BOARDS; TOPNOTCH MD)

MIDTERM 2 - AUG 2013

678

HAZEL KAREN RAZ, MD (TOP 6 - FEB 2013 MED BOARDS; TOPNOTCH MD) HAZEL KAREN RAZ, MD (TOP 6 - FEB 2013 MED BOARDS; TOPNOTCH MD)

TOPNOTCH EXAM MIDTERM 2 - AUG 2013

Most ideal site for paracentesis?

TOPNOTCH MEDICAL BOARD PREP ANATOMY SUPEREXAM Page 88 of 94 For inquiries visit www.topnotchboardprep.com.ph or email us at [email protected]

MIDTERM 2 - AUG 2013

TOPNOTCH MEDICAL BOARD PREP ANATOMY SUPEREXAM For inquiries visit www.topnotchboardprep.com.ph or email us at [email protected] Item #

QUESTION

EXPLANATION

AUTHOR

TOPNOTCH EXAM

A. midway between symphysis and umbillicus B. 1 cm to the left of the lateral border of the rectus abdominis at the level of the umbillicus C. Immediately above the symphysis pubis D. Mc Burney's point E. above the umbillicus

679

Which of the following associations is correct? A. Superficialis fascia : external spermatic fascia B. Transversalis fascia : Dartos muscle C. Internal Oblique muscle : Cremasteric muscle D. Peritoneum : none E. None of the above



HAZEL KAREN RAZ, MD (TOP 6 - FEB 2013 MED BOARDS; TOPNOTCH MD)

MIDTERM 2 - AUG 2013

680

Which of the ff structures of the renal hilus is most posterior? A. Renal artery B. Renal vein C. Gerotas fascia D. Perirenal fat E. Renal pelvis

Order from anterior to posterior: Vein, Artery, Pelvis

MIDTERM 2 - AUG 2013

681

A 15 yo boy complaining of pain in the lower right part of the anterior abdominal wall was seen by a physician. Upon examination, he was found to have a temperature of 38.3OC. He had a furred tongue and was extremely tender in the lower right quadrant. The abdominal muscles in that area were found to be firm on palpation and became more spastic when increased pressure was applied. The organ most likely affected in this case has the following contributory factors why it is predisposed to infection, except; A. It is a long, narrow, blind-ended tube, which encourages stasis of large-bowel contents. B. It is supplied by a long small artery that does not anastomose with other arteries. C. It has a large amount of lymphoid tissue in its wall. D. The lumen has a tendency to become obstructed by hardened intestinal contents, which leads to further stagnation of its contents.

Answer: B. it is supplied by a long small artery that does not anastomose with other arteries (pp. 234, Snell’s Clinical Anatomy By Regions, 8th edition.) Notes: All choices are factors that contribute to the appendix’s predilection to infection except choice B which is a factor for predisposition to perforation.

HAZEL KAREN RAZ, MD (TOP 6 - FEB 2013 MED BOARDS; TOPNOTCH MD) MICHELLE JAY FRANCISCO, MD (TOP 9 - FEB 2013 MED BOARDS; TOPNOTCH MD)

682

The following statements concerning the pancreas are correct except which? A. The pancreas receives part of the arterial supply from the splenic artery. B. The main pancreatic duct opens into the third part of the duodenum. C. The uncinate process of the pancreas projects from the head of the pancreas. D. The bile duct (CBD) lies posterior to the head of the pancreas.

Answer: B. The main pancreatic duct opens into the third part of the duodenum. Notes: The main pancreatic duct opens into the second part of the duodenum, at about its middle, with the bile duct on the major duodenal papilla. Sometimes, the main duct drains separately into the duodenum.

MICHELLE JAY FRANCISCO, MD (TOP 9 - FEB 2013 MED BOARDS; TOPNOTCH MD)

MIDTERM 1 - AUG 2013

683

What part of the ciliary body does the aqueous humor is formed? A. Pars optica B. Pars nervosa C. Pars pigmentosa D. Pars plicata

Answer: D. Pars plicata (Robbins and Cotran Pathologic Basis of Disease, 8th ed.)

MIDTERM 1 - AUG 2013

684

The intercellular spaces in the stratum spinosum of the epidermis contain lipid-containing sheets that are impermeable to water. This material is released from A. Membrane-coating granules B. Keratohyalin granules C. Langerhans cells D. Sebaceous glands

Answer: A. Membrane-coating granules Notes: Membrane-coating granules are present in keratinocytes in the stratum spinosum (and stratum granulosum). The contents of these granules are released into the intercellular spaces to help waterproof the skin. Keratinocytes in the stratum granulosum also possess keratohyalin granules; these contain proteins that bind keratin filaments together.

MICHELLE JAY FRANCISCO, MD (TOP 9 - FEB 2013 MED BOARDS; TOPNOTCH MD) MICHELLE JAY FRANCISCO, MD (TOP 9 - FEB 2013 MED BOARDS; TOPNOTCH MD)

TOPNOTCH MEDICAL BOARD PREP ANATOMY SUPEREXAM Page 89 of 94 For inquiries visit www.topnotchboardprep.com.ph or email us at [email protected]

MIDTERM 1 - AUG 2013

MIDTERM 1 - AUG 2013

TOPNOTCH MEDICAL BOARD PREP ANATOMY SUPEREXAM For inquiries visit www.topnotchboardprep.com.ph or email us at [email protected] Item # 685

QUESTION

EXPLANATION

AUTHOR MICHELLE JAY FRANCISCO, MD (TOP 9 - FEB 2013 MED BOARDS; TOPNOTCH MD) MICHELLE JAY FRANCISCO, MD (TOP 9 - FEB 2013 MED BOARDS; TOPNOTCH MD) MICHELLE JAY FRANCISCO, MD (TOP 9 - FEB 2013 MED BOARDS; TOPNOTCH MD)

TOPNOTCH EXAM MIDTERM 1 - AUG 2013

Which of the following is lined by an epithelium containing ciliated cells and Clara cells? A. Trachea B. Terminal bronchiole C. Intrapulmonary bronchi D. Alveolar duct

Answer: B. Terminal bronchiole Notes: Terminal bronchioles are lined by a simple cuboidal epithelium containing ciliated cells and Clara cells. Clara cells can divide and regenerate both cell types.

686

Which of the following cells in the inner ear are involved in detecting movements of the head? A. Hair cells in the maculae B. Cells of Hensen C. Hair cells in the organ of Corti D. Inner pillar cells

Answer: A. Hair cells in the maculae Notes: Neuroepithelial hair cells in the maculae of the saccule and the utricule detect linear movement of the head. These cells are connected to the vestibular portion of the acoustic nerve.

687

The following statements concerning the stomach are true; except A. The splenic artery runs along the upper border of the pancreas and lies behind the stomach B. The lesser curvature of the stomach receives its blood supply from the right and left gastroepiploic arteries C. The lymph drainage from the gastroesophageal junction passess to the celiac lymph nodes D. The lesser sac lies behind the stomach

Answer: B. The lesser curvature of the stomach receives its blood supply from the right and left gastroepiploic arteries Notes: The greater curvature of the stomach receives its blood supply from the right and left gastroepiploic arteries.

688

Ductus/vas deferens is the main content of the spermatic cord. Among these choices, which is not found in the spermatic cord? A. Hernia sac B. Genital branch of the genitofemoral nerve C. Cremasteric muscle fibers D. Ilioinguinal nerve

Answer: D. Ilioinguinal nerve Notes: Ilioinguinal nerve travels along/on the superficial part of the spermatic cord. Hernia sac may or may not be found inside the spermatic cord. Mnemonics for contents of the spermatic cord: Piles Don’t Contribute To A Good Sex Life • Pampiniform venous plexus • Ductus deferens • Cremasteric muscle • Testicular Artery • Artery of the ductus deferens • Genital branch of the genitofemoral nerve • Sympathetic nerve fibers • Lymphatic vessels

MICHELLE JAY FRANCISCO, MD (TOP 9 - FEB 2013 MED BOARDS; TOPNOTCH MD)

MIDTERM 1 - AUG 2013

689

You asked a patient to look to the left side during a neurologic examination. Patient’s right eye cannot look to his left, with (+) left eye nystagmus, but convergence is still intact. In order to produce these symptoms, you suspect that the lesion is most probably located in the _____________. A. Right medial longitudinal fasciculus B. Left abducens nucleus C. Right cerebral cortex D. Left abducens nerve

Answer: A. Right medial longitudinal fasciculus Notes: • Left abducens nucleus or right cerebral cortex – neither eye can look left with a slow drift to the right • Left abducens nerve – left eye can’t look to the left • Right medial longitudinal fasciculus – right eye can’t look left, left eye nystagmus, and convergence is intact

MICHELLE JAY FRANCISCO, MD (TOP 9 - FEB 2013 MED BOARDS; TOPNOTCH MD)

MIDTERM 1 - AUG 2013

690

Name the ocular defect if the lesion is found in the right lateral geniculate body in the thalamus A. Left homonymous hemianopsia B. Left nasal and temporal hemianopsia C. Bitemporal heteronymous hemianopsia D. Right nasal hemianopsia

MICHELLE JAY FRANCISCO, MD (TOP 9 - FEB 2013 MED BOARDS; TOPNOTCH MD)

MIDTERM 1 - AUG 2013

691

What is true regarding lymphatic anatomy? A. The limb lymphatics are valveless. B. The lymphatic system begins just below the dermis as a network of fine capillaries C. RBC and bacteria do not enter the lymphatic capillaries D. Extrinsic factors (muscle contraction, arterial pulsations, respiratory movement and massage)aid in the movement of lymph flow. E. All of the above.

Answer: A. Left homonymous hemianopsia Notes: • Left optic nerve lesion – left eye anopsia (left nasal and temporal hemianopsia) • Right calcarine cortex lesion – left homonymous hemianopsia • Right LGB lesion – left homonymous hemianopsia • Optic chiasm lesion – bitemporal heteronymous hemianopsia • Right lateral compression of the optic chiasm (as in aneurysms of the internal carotid artery) – right nasal hemianopsia Lymphatic system begins as network of valveless capillaries in the superficial dermis. RBC & bact. enter lymph vessels by separating endothelial cells at their junctions.

MICHELLE JAY FRANCISCO, MD (TOP 9 - FEB 2013 MED BOARDS; TOPNOTCH MD)

MIDTERM 1 - AUG 2013

TOPNOTCH MEDICAL BOARD PREP ANATOMY SUPEREXAM Page 90 of 94 For inquiries visit www.topnotchboardprep.com.ph or email us at [email protected]

MIDTERM 1 - AUG 2013

MIDTERM 1 - AUG 2013

TOPNOTCH MEDICAL BOARD PREP ANATOMY SUPEREXAM For inquiries visit www.topnotchboardprep.com.ph or email us at [email protected] Item # 692

QUESTION

EXPLANATION

AUTHOR

TOPNOTCH EXAM MIDTERM 1 - AUG 2013

A 10-yr old girl is brought to her doctor because of hypoplasia of her right breast. On closer examination, she is found to have asymmetry of the chest wall itself. Which of the following is true regarding this syndrome? A. This affects i in every 10,000 live births. B. Deformity of the thoracoacromial joint is characteristic of this syndrome. C. Absence of the sternum is characteristic of this syndrome D. Absence of the sterna head of the pectoralis muscle is a characteristic of this syndrome. E. Pectus excavatum is part of this syndrome..

Poland syndrome is a congenital defect of about 1 in 30,000 births. Hallmark is chest wall anomaly: partial absence of sternal head of pec.major m., hypo/aplasia of breast & SC tissue and possible complete absence of pec.minor m.

MICHELLE JAY FRANCISCO, MD (TOP 9 - FEB 2013 MED BOARDS; TOPNOTCH MD)

693

Which of the following statements is most accurate regarding branchial cleft anomalies? A. Third arch anomalies are common B. Type II 1st branchial anomalies are the most common over-all. C. The glossopharyngeal nerve is asso. with the 3rd branchial arch. D. Second arch anomalies end in pyriform sinus.

Third arch anomalies are rare & appear in lower neck. Second branchial arch anomalies are the most common & ends at the tonsillar fossa.

MICHELLE JAY FRANCISCO, MD (TOP 9 - FEB 2013 MED BOARDS; TOPNOTCH MD)

MIDTERM 1 - AUG 2013

694

True of liver anatomy: A. The right lobe extends to the umbilical fissure and falciform ligament. B. The left lobe end at the falciform ligament. C. The quadrate lobe is a portion of the medial segment of the right lobe. D. The lateral segments of the left lobe in the American system consists of segments II & III.

The right lobe consists of posterior and anterior segments. The left lobe consists of medial (quadrate) and lateral segment divided by falciform ligament. The caudate can be considered anatomically independent from both R/L lobes because it receives portal & arterial blood supply from both sides.

MICHELLE JAY FRANCISCO, MD (TOP 9 - FEB 2013 MED BOARDS; TOPNOTCH MD)

MIDTERM 1 - AUG 2013

695

All except one drains into the right atrium through the coronary sinus: A. small cardiac B. middle cardiac C. anterior cardiac D. great cardiac

The anterior cardiac vein together w/ smallest cardiac opens directly into the right atrium.

MIDTERM 1 - AUG 2013

696

The muscle and central tendon of the diaphragm develop from which of the following embryonic structures? A. Septum transversum B. Pleuroperitoneal membranes C. Dorsal mesentery of esophagus D. Ventral mesentery of esophagus

Pleuroperitoneal membranes-peripheral area of diaphragmatic pleura & upper/lower pleural surface; Dorsal mesentery - crura

697

The blood supply to the thoracic wall mainly comes from the _________ artery. A. Lateral thoracic B. Internal thoracic C. Posterior intercostals D. Subcostal

These 9 pairs of post. intercostal arteries arise from the thoracic aorta, the right arteries, having longer course than the left ones.

698

I.M. injections should be given in the upper outer quadrant of the buttocks to prevent damage to which of the following nerves? A. Sciatic B. Obturator C. Superior gluteal D. Lateral femoral cutaneous

*review ana nerve distribution of lower limbs

699

This ligament prevents posterior dislocation of the femur at the knee joint. A. Anterior cruciate B. Ischiofemoral C. Lateral collateral D. Posterior cruciate

The rectus femoris as part of quadriceps crosses the hip joint anteriorly and its tendon crosses the knee joint anteriorly resulting in flexion of the thigh and extension of the leg.

MICHELLE JAY FRANCISCO, MD (TOP 9 - FEB 2013 MED BOARDS; TOPNOTCH MD) MICHELLE JAY FRANCISCO, MD (TOP 9 - FEB 2013 MED BOARDS; TOPNOTCH MD) MICHELLE JAY FRANCISCO, MD (TOP 9 - FEB 2013 MED BOARDS; TOPNOTCH MD) MICHELLE JAY FRANCISCO, MD (TOP 9 - FEB 2013 MED BOARDS; TOPNOTCH MD) MICHELLE JAY FRANCISCO, MD (TOP 9 - FEB 2013 MED BOARDS; TOPNOTCH MD)

TOPNOTCH MEDICAL BOARD PREP ANATOMY SUPEREXAM Page 91 of 94 For inquiries visit www.topnotchboardprep.com.ph or email us at [email protected]

MIDTERM 1 - AUG 2013

MIDTERM 1 - AUG 2013

MIDTERM 1 - AUG 2013

MIDTERM 1 - AUG 2013

TOPNOTCH MEDICAL BOARD PREP ANATOMY SUPEREXAM For inquiries visit www.topnotchboardprep.com.ph or email us at [email protected] Item # 700

QUESTION A neurologic exam of a 34 yr-old man reveals a direct and consensual light reflex in his left eye but neither in his right eye. The nerve involved is the: A. right optic nerve B. left optic nerve C. right oculomotor D. left oculomotor

EXPLANATION

AUTHOR MICHELLE JAY FRANCISCO, MD (TOP 9 - FEB 2013 MED BOARDS; TOPNOTCH MD)



TOPNOTCH MEDICAL BOARD PREP ANATOMY SUPEREXAM Page 92 of 94 For inquiries visit www.topnotchboardprep.com.ph or email us at [email protected]

TOPNOTCH EXAM MIDTERM 1 - AUG 2013

TOPNOTCH MEDICAL BOARD PREP ANATOMY SUPEREXAM For inquiries visit www.topnotchboardprep.com.ph or email us at [email protected] Item # ANSWER 1 2 3 4 5 6 7 8 9 10 11 12 13 14 15 16 17 18 19 20 21 22 23 24 25 26 27 28 29 30 31 32 33 34 35 36 37 38 39 40 41 42 43 44 45 46 47 48 49 50 51 52 53 54 55 56 57 58 59 60 61 62 63 64 65 66 67 68 69 70 71 72 73 74 75 76 77 78 79 80 81 82 83 84 85 86

KEY E E B A C B E D B A C C A E C C D E C D A C D B A B D D C D C C B B C E E A B C C B A B A C A E E C D C C A B E B B C B C A D B B BONUS E D B C B B D E B D C A D C A D B B A D

87 88 89 90 91 92 93 94 95 96 97 98 99 100 101 102 103 104 105 106 107 108 109 110 111 112 113 114 115 116 117 118 119 120 121 122 123 124 125 126 127 128 129 130 131 132 133 134 135 136 137 138 139 140 141 142 143 144 145 146 147 148 149 150 151 152 153 154 155 156 157 158 159 160 161 162 163 164 165 166 167 168 169 170 171 172 173 174

B A A A B B C D B C B E C C D D A C B D A B A C E D B C D A B E A C C A E E C B A B C D B A B C A B D D D E B A A E B B D B A A E B E A B B B A C D D A B B E A,B C A E B A B C B

175 176 177 178 179 180 181 182 183 184 185 186 187 188 189 190 191 192 193 194 195 196 197 198 199 200 201 202 203 204 205 206 207 208 209 210 211 212 213 214 215 216 217 218 219 220 221 222 223 224 225 226 227 228 229 230 231 232 233 234 235 236 237 238 239 240 241 242 243 244 245 246 247 248 249 250 251 252 253 254 255 256 257 258 259 260 261 262

D E C D A C A D C D B C B A C B E D C B B A B D C C C A A C B D B B B A B D C A D C B A D B C A C C B A A C C C B A C D C C D C A B A C A C C D C D B B B B C C D D C B C C E D

263 264 265 266 267 268 269 270 271 272 273 274 275 276 277 278 279 280 281 282 283 284 285 286 287 288 289 290 291 292 293 294 295 296 297 298 299 300 301 302 303 304 305 306 307 308 309 310 311 312 313 314 315 316 317 318 319 320 321 322 323 324 325 326 327 328 329 330 331 332 333 334 335 336 337 338 339 340 341 342 343 344 345 346 347 348 349 350

B D B C B D B B B A B A A A D D C B A D B B E C C B E A E C E D C A C D B D C A D A B D C C C A B C A C A B E B D C D A E C C B B E B E C C D D C B E B C C A B B C C D C C B

TOPNOTCH MEDICAL BOARD PREP ANATOMY SUPEREXAM Page 93 of 94 For inquiries visit www.topnotchboardprep.com.ph or email us at [email protected]

351 352 353 354 355 356 357 358 359 360 361 362 363 364 365 366 367 368 369 370 371 372 373 374 375 376 377 378 379 380 381 382 383 384 385 386 387 388 389 390 391 392 393 394 395 396 397 398 399 400 401 402 403 404 405 406 407 408 409 410 411 412 413 414 415 416 417 418 419 420 421 422 423 424 425 426 427 428 429 430 431 432 433 434 435 436 437 438

B C A C A B A C B A C A B C C C D C D D A A B D D D C C C B B C C D E A D D B D A C B C B A B B B C D E D C B A B D A C D A D B A E B E B B B A B C D B B D B B D C A B A D B D

TOPNOTCH MEDICAL BOARD PREP ANATOMY SUPEREXAM For inquiries visit www.topnotchboardprep.com.ph or email us at [email protected] 439 440 441 442 443 444 445 446 447 448 449 450 451 452 453 454 455 456 457 458 459 460 461 462 463 464 465 466 467 468 469 470 471 472 473 474 475 476 477 478 479 480 481 482 483 484 485 486 487 488 489 490 491 492 493 494 495 496 497 498 499 500 501 502 503 504 505 506 507 508 509 510 511 512 513 514 515 516 517 518 519 520 521 522 523 524 525 526 527

C D C E B B C A D D E D B A E C D E B D C D D B A D C C A A A D D A B D E A E A A C A B C A C D D A B B C A B D C B D C B C D C A C C D A A B D E B A D B D A B D C D D B C D A A

528 529 530 531 532 533 534 535 536 537 538 539 540 541 542 543 544 545 546 547 548 549 550 551 552 553 554 555 556 557 558 559 560 561 562 563 564 565 566 567 568 569 570 571 572 573 574 575 576 577 578 579 580 581 582 583 584 585 586 587 588 589 590 591 592 593 594 595 596 597 598 599 600 601 602 603 604 605 606 607 608 609 610 611 612 613 614 615 616

C C D C B B D D B A C B D B C C C B D D C A B A E C B A C D A A E A A A B C B A C A A C B B A A C B A B B B C C B A E A D B B A D B E E A B A C A D C C C A E D A B D D E A E D A

617 618 619 620 621 622 623 624 625 626 627 628 629 630 631 632 633 634 635 636 637 638 639 640 641 642 643 644 645 646 647 648 649 650 651 652 653 654 655 656 657 658 659 660 661 662 663 664 665 666 667 668 669 670 671 672 673 674 675 676 677 678 679 680 681 682 683 684 685 686 687 688 689 690 691 692 693 694 695 696 697 698 699 700

A D C B C B C C E B C C D C A C B B A D B C D C B E A A C D A D B B A E D B C C E E C E B B C B A E D D D A C C D A C B C A C E B B D A B A B D A A D D C D C A C A A C

TOPNOTCH MEDICAL BOARD PREP ANATOMY SUPEREXAM Page 94 of 94 For inquiries visit www.topnotchboardprep.com.ph or email us at [email protected]

View more...

Comments

Copyright ©2017 KUPDF Inc.
SUPPORT KUPDF